OSCE Test

अब Quizwiz के साथ अपने होमवर्क और परीक्षाओं को एस करें!

Obstetrics and Gynaecology 1 - You are an RMO in the emergency centre at KEMH and your next patient is Rachael, a 24 year old primigravida at about 9 weeks gestation. She has presented with vaginal bleeding. Take a focused history, state your differential diagnoses and outline a management plan.

Current pregnancy Hx [0.5] Pregnancy Sx [0.5] Bleeding Hx [0.5] LMP, regular/irregular, heavy/light [0.5] Pain Hx [0.5] Gyn Hx (inc Pap smear, contraception use) [0.5] Associated Sx (dizziness/fainting, N/V/D, pain) [0.5] Miscarriage / Spontaneous abortion [0.5] Idiopathic bleeding in a viable pregnancy [0.5] Ectopic pregnancy [0.5] Molar pregnancy [0.5] Bleeding from cervix or vagina (e.g. cancer) [0.5] Find underlying cause, e.g. ectopic, miscarriage or viable pregnancy. [1] Examination: Assess vital signs and resuscitate if needed [0.5] Abdominal & PV/SPEC examination (if OS open - inevitable miscarriage) [0.5] Bloods: Blood group & cross-match & Rh antibodies (Rh issues & blood loss) [1] FBC [0.5] Quantative BHCG [0.5] Imaging: Transvaginal ultrasound [1] Urgent gynaecology referral [1]Empathy score: 2

Paediatrics 1 - GP working in Kalgoorlie. Tim is 2 yo admitted overnight following febrile convulsion. How do you know it's not an epileptic fit?

Explain febrile convulsion appropriately and as destinct from epilepsy[3]and chance of repeated episode 30-40%[1]Likelihood of epilepsy in future 2-7%[1](given no family history)[1]Explain Mx fever in future: keep cool, paracetamol does not reduce risk[2]Explains how to give first aid for convulsion (lateral position, move over, rectal diazepam if>10 min)[4]Empathy score: 2

General Practice 2 - Patient presents to GP practice with a diffuse maculopapular rash, they think it might be an allergic reaction

Hx of allergy, arthralgia, rash onset + distribution, fever/recent illness, new foods, recent meds, contacts-plants/chemicals[4]Angioedema symptoms - wheeze, cough, face swelling[1]Urticaria, explain most likely viral, unlikely to be allergy, explain further tests not usually helpful[2]Non-sedating antihistamine[0.5]Calamine/other topical[0.5]Tepid baths[0.5]Likely to resolve in 2/52[0.5]Return if symptoms persist[1]May Ix if rash persists[1]Suggest diary to see if pattern develops[1]Empathy score: 2

"Paediatrics 3 - Simon is a 4 year old boy who has presented with an 8 hour history of being unwell with 3 non-bilious vomits and 2 loose stools. You feel that he looks reasonably well, that he is not dehydrated and that you can "wait and see" how he

...

General Medicine 1 - A 24-year-old woman had a metal prosthetic heart valve inserted 4 years ago after a severe episode of bacterial endocarditis that degenerated her mitral valve. She has done very well since the surgery and has been on warfarin, regularly attending an anticoagulation clinic. She has come to see you today because she wants to have a family but has read on the internet that warfarin can damage the baby. Can she stop her warfarin?

...

General Medicine 1 - A 70-year-old woman presents with sharp shooting pains in the region of the left cheek.The pains last a few seconds and occur several times a day. She finds that yawning or touching the left cheek provokes attacks of the pain. ❶: What is the likely differential diagnosis? ❷: What in the given history supports the diagnosis? ❸: What additional features in the history would you seek to support a particular diagnosis? ❹: What clinical examination would you perform and why? ❺: What investigations would be most helpful and why? ❻: What treatment options are appropriate?

...

General Medicine 1 - A 75-year-old man who lives in a nursing home is confined to his bed or chair as a result of a previous left-sided stroke. He is a large man and his care is made even more challenging as a result of the presence of dysphasia, dysphagia and mild cognitive impairment. The patient's GP periodically visits the nursing home to review his care and general condition. List five important areas that the GP should enquire about or assess when visiting. Give a brief explanation as to why each of these areas needs review.

...

General Medicine 1 - A pregnant 35-year-old woman presents with a 4-month history of tingling in her left hand. This is worse at night and often wakes her. She finds that hanging her left arm down outside the bed or shaking the left hand may help. She was previously well. ❶: What is the likely differential diagnosis? ❷: What in the given history supports the diagnosis? ❸: What additional features in the history would you seek to support a particular diagnosis? ❹: What clinical examination would you perform and why? ❺: What investigations would be most helpful and why? ❻: What treatment options are appropriate?

...

General Medicine 1 - An elderly housebound woman has fallen at home on several occasions and has recently attended A&E with a Colles' (wrist) fracture. Her daughter is concerned about the presence of osteoporosis and seeks your advice at the surgery about the need for calcium tablets in order to prevent further fractures.

...

General Medicine 2 - A 29-year-old woman presents with a 6-month history of pain and swelling, which started in the fingers and wrists and then spread to her shoulders, knees and ankles. She was treated for pleurisy twice 4 and 3 months ago and since then says she feels unwell. Most recently she says that she has developed a rash on her cheeks and forehead. ❶: What is the likely differential diagnosis? ❷: How would you investigate this case? ❸: How would you confirm the likely diagnosis? ❹: How would you initially manage the case? ❺: What are the principles of long-term management? ❻: What is the prognosis? ❼: Will having lupus give me problems should I want to become pregnant?

...

General Medicine 2 - A 68-year-old woman, a known CRF patient, has presented to A&E with acute shortness of breath. Computer records suggest that her most recent serum creatinine from about a month earlier was 256 μmol/L (GFR 21 mL/min) and her regular medication includes furosemide 80 mg daily. She has recently been commenced on an NSAID for arthritis. On examination, she has oedema up to her mid-thighs, her jugular venous pressure (JVP) is raised at 6 cm, and there are crackles in both lung bases up to the midzones. Her admission ECG does not show any ischaemic changes, and her urea and electrolytes (U&Es) show: Na+ 138 mmol/L, K+ 5.6 mmol/L, urea 38 mmol/L, creatinine 446 μmol/L (GFR 12 mL/min). ❶: What is the likely diagnosis? ❷: How would you investigate this case? ❸: What would be the initial management? ❹: What would be the long-term management?

...

General Medicine 2 - A 72-year-old man who suffers from cryptogenic fibrosing alveolitis attends the clinic with his daughter. They both feel that he requires oxygen to help him with his daily activities. ❶: What criteria should you assess before prescribing oxygen? ❷: What advice would you give to him about the use of oxygen?

...

General Medicine 3 - 'I hate doing injections and painful blood tests for my diabetes. Can I try an insulin pump instead? ❶: Please counsel the patient.

...

General Medicine 3 - 'My friend has diabetes and she says that all people with diabetes should go to a podiatrist. Is this true?' ❶: Please counsel the patient.

...

General Medicine 3 - A 19-year-old student presents with an 18-month history of progressive tiredness, weight loss and lethargy. One year before presentation she had a common cold that forced her to stay away from lectures for 2 days. Three months before presentation, another common cold laid her low for almost 2 weeks, during which time she lost 3 kg in weight. The local student health doctors had assured her that the symptoms were the result of either being pregnant or taking drugs. On examination, she was noted to have darker skin than her mother and pigmented palmar creases. ❶: What do you think the likely diagnosis is? ❷: What tests could you do to confirm this? ❸: What tests would help establish the cause? ❹: What is the initial management? ❺: What are the long-term complications/sequelae? ❻: Why did this happen to me? ❼: How will it affect my life?

...

General Medicine 3 - A 20-year-old woman with ulcerative colitis has had symptoms for several weeks so she presents to the accident and emergency department (A&E). ❶: What specific questions would you ask the patient? ❷: What is the most likely diagnosis? ❸: What examination would you perform? ❹: What would be the initial management? ❺: What investigations would you request to confirm a diagnosis? ❻: What other issues should be addressed? ❼: Will I need to have the bag?

...

General Medicine 3 - A 21-year-old man with diarrhoea and a perianal abscess. He has noted episodes of colicky lower abdominal pain over several months and it had been suggested that he may have IBD. ❶: What specific questions would you ask the patient? ❷: What is the most likely diagnosis? ❸: What examination would you perform? ❹: What would be the initial management? ❺: What investigations would you request to confirm a diagnosis? ❻: What other issues should be addressed?

...

General Medicine 3 - A 21-year-old woman who has always been fit and well visits you shortly after her honeymoon. She has been profoundly thirsty and noticing blurring of her vision. A finger-prick test in the surgery shows a blood glucose level of 18.9 mmol/L and a fresh urine specimen has both glycosuria and heavy ketones (+++) ❶: What specific questions would you ask the patient? ❷: What investigations would you request to confirm a diagnosis? ❸: What examination would you perform? ❹: What would be the initial management? ❺: What are the long-term sequelae? ❻: What issues need to be addressed apart from blood glucose control?

...

General Medicine 3 - A 21-year-old woman with type 1 diabetes of 15 years' standing has an annual review of her diabetes. She injects insulin using a basal bolus regimen and does not smoke. Her BMI is 22 kg/m2, BP 120/60 mmHg and she has good pedal pulses. She has minimal background diabetic retinopathy and slightly diminished sensation in her feet. Random TC is 5.7 mmol/L, triglycerides 1.1 mmol/L. and HbA1c 6.5 per cent (normal range < 5 per cent). Analysis of a random urine shows an ACR of 2.7 which is reported as 'microalbuminuria'. Previously urinalysis had been negative for dipstick proteinuria. ❶: Are further assessments required before the italicized finding provokes a drug intervention? ❷: If drug therapy were indicated, which would be the most appropriate choice of agent? ❸: What sort of follow-up is indicated? ❹: What other issues need to be addressed as regards cardiovascular risk?

...

General Medicine 3 - A 23-year-old woman presents with impaired fertility and loose stools. She had loose stools throughout her teenage years. She is slightly built and has noted that it is difficult to increase her weight. ❶: What is the likely differential diagnosis? ❷: What in the given history supports the diagnosis? ❸: What additional features in the history would you seek to support a particular diagnosis? ❹: What clinical examination would you perform and why? ❺: What investigations would be most helpful and why? ❻: What treatment options are appropriate?

...

General Medicine 3 - A 24-year-old housewife complains of abdominal bloating, which gets progressively worse during the day. ❶: What is the likely differential diagnosis? ❷: What in the given history supports the diagnosis? ❸: What additional features in the history would you seek to support a particular diagnosis? ❹: What clinical examination would you perform and why? ❺: What investigations would be most helpful and why? ❻: What treatment options are appropriate?

...

General Medicine 3 - A 26 year-old-receptionist presents with palpitations and weight loss. Although pleased about the latter, she is concerned that her menstrual periods have stopped and it is the anxiety related to this, she believes, that is responsible for disturbing her sleep. Her partner has insisted that she seek a medical opinion. ❶: What do you think the likely diagnosis is? ❷: What tests could you do to confirm this? ❸: What tests would help establish the cause? ❹: What is the initial management? ❺: What are the long-term complications/sequelae? ❻: Do I need to have surgery? ❼: Will I be able to have children after radioiodine treatment?

...

General Medicine 3 - A 26-year-old woman presents with a 3-week history of bloody diarrhoea. The blood is fresh and mixed in the liquid stool. Initially, she thought that she might have food poisoning, but could not recall eating anything unusual and had eaten the same food as her unaffected partner. ❶: What specific questions would you ask the patient? ❷: What is the most likely diagnosis? ❸: What examination would you perform? ❹: What would be the initial management? ❺: What investigations would you request to confirm a diagnosis? ❻: What other issues should be addressed? ❼: Is it infectious? ❽: Will my children get it?

...

General Medicine 3 - A 42-year-old social worker presents with a story of straining to pass stool, passage of mucus and perhaps blood on the tissue paper. ❶: What is the likely differential diagnosis? ❷: What in the given history supports the diagnosis? ❸: What additional features in the history would you seek to support a particular diagnosis? ❹: What clinical examination would you perform and why? ❺: What investigations would be most helpful and why? ❻: What treatment options are appropriate?

...

General Medicine 3 - A 44-year-old woman presents with lethargy, breathlessness on exertion and a little chest tightness on walking uphill. She was turned down as a blood donor because of anaemia. ❶: What is the likely differential diagnosis? ❷: What in the given history supports the diagnosis? ❸: What additional features in the history would you seek to support a particular diagnosis? ❹: What clinical examination would you perform and why? ❺: What investigations would be most helpful and why? ❻: What treatment options are appropriate?

...

General Medicine 3 - A 46-year-old man with alcohol problems presents with haematemesis and shock. ❶: What specific questions would you ask the patient? ❷: What is the most likely diagnosis? ❸: What examination would you perform? ❹: What would be the initial management? ❺: What investigations would you request to confirm a diagnosis? ❻: What other issues should be addressed?

...

General Medicine 3 - A 50-year-old man with squamous carcinoma of the bronchus and bony mestastases is referred complaining of abdominal distension. He has not opened his bowels for 8 days. His abdominal pain has worsened and he has increased his morphine sulphate tablets (MST), which previously he just took for bone pain. ❶: What is the likely differential diagnosis? ❷: What in the given history supports the diagnosis? ❸: What additional features in the history would you seek to support a particular diagnosis? ❹: What clinical examination would you perform and why? ❺: What investigations would be most helpful and why? ❻: What treatment options are appropriate? ❼: Doctor, what can be done to stop my husband who needs to take pain-killers becoming constipated again?

...

General Medicine 3 - A 70-year-old man with arthritis and ischaemic heart disease presents with indigestion. He has epigastric pain radiating to his back. He has been taking aspirin for his heart and ibuprofen for his arthritis. ❶: What is the likely differential diagnosis? ❷: What in the given history supports the diagnosis? ❸: What additional features in the history would you seek to support a particular diagnosis? ❹: What clinical examination would you perform and why? ❺: What investigations would be most helpful and why? ❻: What treatment options are appropriate?

...

General Medicine 3 - An ophthalmologist recommends a 26-year-old man to seek an opinion about his long history of back pain and stiffness after developing a red and painful left eye. ❶: What is the likely differential diagnosis? ❷: How would you investigate this case? ❸: How would you confirm the likely diagnosis? ❹: How would you initially manage the case? ❺: What are the principles of long-term management? ❻: What is the prognosis?

...

General Medicine 3 - As someone with diabetes, should I be taking aspirin? ❶: Please counsel the patient.

...

General Medicine 3 - I have been diagnosed as having maturity-onset diabetes. Does this mean that I will never need go on to injections?' ❶: Please counsel the patient.

...

Obstetrics and Gynaecology 2 - Mrs V presents with hot flushes and night sweats. She has not had a period for 12 months. ❶ Please manage her problem. ❷ Please advise Mrs Von the use of hormone therapy (HT). ❸ Please advise Mrs Von the benefits and risks of using hormone therapy.

...

Obstetrics and Gynaecology 2 - You are seeing Mrs Prouse, a 70-year-old woman with three episodes of vaginal bleeding over the last 6 weeks. ❶ Please take a history from Mrs Prouse. ❷ What examination findings would you like to know? ❸ What are the most appropriate investigations? ❹ What features are you looking for in the ultrasound scan? ❺ The curettings show a grade 1 endometrioid adenocarcinoma. What is the most appropriate treatment for Mrs Prouse's cancer?

...

Obstetrics and Gynaecology 3 - A 20-year-old woman, Anna, presents for her first Pap smear. She has had an episode of post-coital bleeding in the last week. ❶ Please take a history of the presenting complaint from Anna. ❷ What investigations would you like to perform at the time of your examination? ❸ The results of your investigations are: • Pap smear: normal • Colposcopy: normal • HVS: - Wet prep- trichomonas absent Gram stain -leukocytes++, normal vaginal flora - Culture- heavy growth of normal vaginal flora • Chlamydia DNA- positive • Gonorrhoea DNA- negative. What is your management now?

...

Obstetrics and Gynaecology 3 - Mrs Tran has been referred to you by her GP with the following letter: Dear Dr, I am referring Mrs Tran to you, who is 10 weeks pregnant with recent onset of jaundice. Please manage her pregnancy. Regards. You are the doctor. The examiner is playing the role of Mrs Tran. Results show: • Blood group A+, no antibodies • FBE Hb 130 WCC 6.0 Pits 200 • Rubella immune, TPHA negative, MSU negative, HIV negative, declines Down syndrome screening test • LFT: AST 250, ALT 500, ALP 50, bilirubin 180, albumin 35 • Hep A negative • Hep B +ve (HBcAg +ve, anti-HBclgM +ve, HBsAg +ve, anti-HBslg -ve, HBeAg-ve) • Hep C negative • Upper abdominal ultrasound: no gallstones detected. ❶ Please explain to Mrs Tran how her diagnosis alters your management of her pregnancy.

...

Obstetrics and Gynaecology 3 - Ms B presents for an abortion. She is 10 weeks pregnant based on an ultrasound. ❶ Please take an appropriate history. ❷ Please inform Ms B on the process of pregnancy termination and the risks involved. ❸ What should be advised for Ms B for the future?

...

Obstetrics and Gynaecology 3 - You are in antenatal clinic. Susan Thompson is a 30-year-old primigravida at 38 weeks gestation. You have just examined her and noted her BP to be 160/100 with a FWT 2+ protein. ❶ "Is there something wrong, doctor?" ❷ What happens now, doctor? Please advise on management of this patient.

...

Obstetrics and Gynaecology 3 - You are seeing Ms Veronica Hilton, a 23-year-old woman who wishes to try to have a baby in the next few months. She smokes 20 cigarettes a day and drinks 2-3 glasses of wine a night. You are speaking to the patient. ❶ Please counsel her about the risks of smoking and alcohol, with particular reference to obstetric and gynaecological problems. ❷ Ms Hilton is now 12 weeks pregnant, and has stopped drinking alcohol. However, she still smokes 10 cigarettes a day. "Can I use nicotine patches to help me stop smoking, doctor?" ❸ Ms Hilton has a friend who doesn't smoke or drink, but uses cocaine at parties and is 20 weeks pregnant. "Can cocaine harm her pregnancy, doctor?"

...

Obstetrics and Gynaecology 4 - Mrs Drake is 40 years old, and presents at 9 weeks gestation in her third pregnancy. She wishes to discuss the risk of Down syndrome. ❶ Please take a brief history and counsel her. ❷ Will an ultrasound pick up Down syndrome, doctor? I would not want to have a Down syndrome child. ❸ Mrs Drake has a high-risk screening test for Down syndrome (1 in 40). Explain this to her and counsel appropriately. "What do I need to do now, doctor?"

...

Ophthalmology - 'Will laser treatment make my vision worse? My neighbour, who also has diabetes, is now blind following laser treatment.' This patient has had type 2 diabetes for 10 years. The ophthalmologist has recommended laser treatment for her right eye. ❶: What factors would you consider important when counselling this patient?

...

Ophthalmology - A 50-year-old patient complains of intermittent loss of vision in one eye like a 'cloud' or 'curtain' descending over the vision, which clears after a few seconds. ❶: What is the likely differential diagnosis? ❷: What in the given history supports the diagnosis? ❸: What additional features in the history would you seek to support a particular diagnosis? ❹: What clinical examination would you perform and why? ❺: What investigations would be most helpful and why? ❻: What treatment options are appropriate?

...

Ophthalmology - A 50-year-old patient presents with a 1-day history of sudden painless loss of vision. The patient has diabetes and hypertension. ❶: What is the likely differential diagnosis? ❷: What in the given history supports the diagnosis? ❸: What additional features in the history would you seek to support a particular diagnosis? ❹: What clinical examination would you perform and why? ❺: What investigations would be most helpful and why? ❻: What treatment options are appropriate?

...

Ophthalmology - A 60-year-old patient complains of a gradual deterioration in vision in both eyes over 1 year. ❶: What is the likely differential diagnosis? ❷: What in the given history supports the diagnosis? ❸: What additional features in the history would you seek to support a particular diagnosis? ❹: What clinical examination would you perform and why? ❺: What investigations would be most helpful and why? ❻: What treatment options are appropriate?

...

Ophthalmology - A 60-year-old patient presents with a 1-week history of double vision. ❶: What is the likely differential diagnosis? ❷: What in the given history supports the diagnosis? ❸: What additional features in the history would you seek to support a particular diagnosis? ❹: What clinical examination would you perform and why? ❺: What investigations would be most helpful and why? ❻: What treatment options are appropriate?

...

Ophthalmology - A 60-year-old patient recently had uncomplicated cataract extraction with an intraocular lens implant. The patient complains of difficulty reading. The distance vision is good. The patient has not yet obtained new spectacles. ❶: Can you reassure the patient? Explain why they are experiencing problems with unaided near vision and what can be done to correct the problem.

...

Ophthalmology - A 65-year-old woman presents with a 1-day history of pain, redness and reduced vision in one eye. She has worn spectacles for reading since the age of 7. ❶: What is the likely differential diagnosis? ❷: What in the given history supports the diagnosis? ❸: What additional features in the history would you seek to support a particular diagnosis? ❹: What clinical examination would you perform and why? ❺: What investigations would be most helpful and why? ❻: What treatment options are appropriate?

...

Ophthalmology - A mother complains that her 4-month-old baby has had watery eyes since birth. ❶: What is the likely differential diagnosis? ❷: What in the given history supports the diagnosis? ❸: What additional features in the history would you seek to support a particular diagnosis? ❹: What clinical examination would you perform and why? ❺: What investigations would be most helpful and why? ❻: What treatment options are appropriate?

...

Ophthalmology - A parent complains that her 2-year-old infant has developed a turn in one eye. ❶: What is the likely differential diagnosis? ❷: What in the given history supports the diagnosis? ❸: What additional features in the history would you seek to support a particular diagnosis? ❹: What clinical examination would you perform and why? ❺: What investigations would be most helpful and why? ❻: What treatment options are appropriate?

...

Ophthalmology - I have recently been diagnosed with glaucoma. Can I still drive?'

...

Ophthalmology - This 60-year old-patient has been diagnosed with glaucoma. He was prescribed timolol 0.5 per cent eye drops. ❶: Is it possible to develop dyspnoea in association with timolol use? Discuss the possible side effects of this eye drop.

...

Paediatrics 2 - A 12-month-old was upset, went blue and had a "fit" Uerking all limbs, eyes rolling)

...

Paediatrics 2 - Brian: is a 6-year-old boy who fell off his bike. He has knocked out one of his upper incisors and the other one has been cracked in half

...

Paediatrics 2 - Callum is a 8 year old boy with a 4 week history of vomiting and headache. He has had unsteady gait for 3 weeks and eye deviation for the last 2 days.

...

Paediatrics 2 - Christopher is a five-year-old boy with an unretractable foreskin.

...

Paediatrics 2 - Dominic is a five-year-old boy who has fallen from a tree onto his chest and abdomen

...

Paediatrics 2 - Helen is an 18-month-old girl who is brought to see you with a "limp". Headache, seizures and other neurological problems

...

Paediatrics 2 - Jayden is a two-year-old boy with a swelling in the scrotum.

...

Paediatrics 2 - Kaitlin has had 3 urinary tract infections, but her mother says that this has not been "looked into" yet

...

Paediatrics 2 - Mark is a 3-year-old boy whose parents are worried he has "bow" legs

...

Paediatrics 2 - My 6-month-old baby has a cough, is burning up, and is struggling to breathe. A 6-month-old baby has had a cold for the past few days, but in the past 12 hours has developed a high fever, cough, poor feeding and loose stools. He has vomited twice and is restlessly sleeping more than usual or is crying inconsolably. ❶: What is the likely differential diagnosis? ❷: What in the given history supports the diagnosis? ❸: What additional features in the history would you seek to support a particular diagnosis? ❹: What clinical examination would you perform and why? ❺: What investigations would be most helpful and why? ❻: What treatment options are appropriate?

...

Paediatrics 2 - Problems of the genito-urinary or gastrointestinal systems

...

Paediatrics 2 - Sally is a 2-day-old girl whose left foot is held in an abnormal position (plantar flexed and inverted)

...

Paediatrics 2 - Tayla is a 4-year-old girl with knock-knees. Her parents want to know if this will be permanent

...

Paediatrics 2 - Tegan is a two-year-old girl with PR bleeding.

...

Paediatrics 2 - Zac is a 7-year-old boy with headaches

...

Paediatrics 3 - A neonate presents with jerking movements.

...

Paediatrics 3 - Andrew is having his 12 month vaccinations. He has multiple bruises on his trunk and some unusual skin lesions.

...

Paediatrics 3 - Beau is a 10-month old girl who is not thriving. She has infected scabies, fleas and lice

...

Paediatrics 3 - Brandon is a 6-months-old boy with failure to thrive, poor feeding. The clinic nurse has noted him to be breathing quickly on a number of occasions. You hear a murmur on examination

...

Paediatrics 3 - Colin is the shortest boy in his class. His mother is wondering if he could have "hormone treatment"

...

Paediatrics 3 - Jonas is a 12-year-old boy with abdominal pain and weight loss for 9 months.

...

Paediatrics 3 - Joshua is a 5-year-old boy with cerebral palsy who will be entering the school system

...

Paediatrics 3 - Kylie is a 14-year-old girl with recurrent abdominal pain

...

Paediatrics 3 - Maggie is a six-month-old infant with screaming episodes and vomiting.

...

Paediatrics 3 - Megan is a 5-year-old girl and has indicated to her mother that her stepfather had been touching her "down there"

...

Paediatrics 3 - Olivia is a two-day-old neonate who has not passed meconium. She has some dysmorphic features.

...

Paediatrics 3 - Parents of an 11-month-old girl who is starting to walk come to you to ask advice about how they should make the house safe? (What are the potential hazards? What measures can they take? What other resources might they find useful? Who else can they talk to for advice? What should they do in an emergency (common scenarios)? What should they have in their first aid cabinet?)

...

Paediatrics 3 - Ralph is a one-day-old neonate who has green vomit.

...

Paediatrics 3 - Rebecca is overweight. She is having trouble keeping up with the other children in the class at sports. Her mother wonders if it is a "gland" problem

...

Paediatrics 3 - Ritchie is a 1 0-year-old boy with epigastric pain

...

Paediatrics 3 - Sydney is a 1 0-year-old boy who has difficulty in learning

...

Paediatrics 3 - Tahnee is a 6-year-old girl who has recently lost weight. She has been drinking more than usual

...

Paediatrics 3 - Teagan is a 2-year-old girl with a large head (>90 th percentile for age)

...

General Practice 1 - A 32 year old woman comes to see you, she is a smoker and wants to quit.

0

General Practice 1 - A 50 year old man has been sent along by his wife. He lost his license last month for the 3rd time for 'DUI' and she has insisted that he talk to you about his alcohol use.

0

General Practice 1 - After your last consultation you made a note that you need to talk with this obese patient about losing weight next time you see them. You did not mention this to the patient.

0

General Practice 1 - You are seeing this patient who you know well with poorly controlled diabetes.

0

General Practice 1 - Your last patient has been feeling unwell for about 3 days with a cough and fever. S/he thinks it's flu, but feels really unwell, and is sometimes a bit breathless. On examination you hear crackles in the left lower and mid zones.

0

General Practice 1 - Your next patient, a middle aged woman, has had a cough for the last 3 months.

0

General Practice 1 - Your patient is 19 and tells you that s/he has recently started a triathlon-training course but finds s/he is still getting short of breath when training, and feels his/her fitness should have improved by now.

0

General Practice 1 - Your patient tells you s/he has the 'flu'. S/he has had a head cold and sore throat, and now a persistent productive cough for 5 days. S/he says s/he wants something to get rid of it fast as s/he is travelling overseas for work next week.

0

General Practice 2 - A patient comes to see you about a cold that won't go away. The nasal discharge is usually quite watery, but is often thicker in the morning. S/he sometimes wakes with a 'scratchy' throat, and sometimes feels some phlegm at the back of the throat. S/he is adamant that it isn't hay fever, as s/he also gets that around cats and dogs, and some flowers - and it only lasts for a couple of hours, with sneezing and itchy eyes.

0

General Practice 2 - A patient comes to see you for a 'Progress certificate' for their Worker's Compensation claim. S/he says the back pain is about the same, but s/he's getting increasingly frustrated about his/her slow recovery. S/he wonders if acupuncture will help.

0

General Practice 2 - Patient comes in for something for her 'migraines' - says she has been getting them for years, and nothing really seem to help, but she's heard of some nasal sprays.

0

General Practice 2 - Your next patient presents with facial pain.

0

General Practice 3 - A 40 year old male barrister has insisted that your receptionist give him an urgent appointment with you even though you are heavily booked and running late. He has a migraine and is adamant that he 'just needs a shot of pethidine'. He has never seen you before, his usual doctor is on leave and his medical records are not available. Your receptionist tells you that he is becoming angry as he has been waiting to see you for half an hour.

0

General Practice 3 - A 60 year old patient with a past history of Dukes B adenocarcinoma of the bowel resected 3 years ago presents for results. You saw him last week and he complained of right upper quadrant pain and weight loss so you arranged an ultrasound. This has shown 'multiple lesions in the liver highly suggestive of metastatic disease'.

0

General Practice 3 - A parent, a patient you know well, comes in to discuss their son's sexuality - he 'came out' last week.

0

General Practice 3 - A woman comes in to see the doctor. She is extremely embarrassed to tell the doctor that she has an unusual discharge.

0

General Practice 3 - A woman requests the morning after pill.

0

General Practice 3 - An adolescent visits the doctor because she thinks she has the flu.

0

General Practice 3 - Your next patient has suffered from episodes of abdominal pain for several months. She has seen another doctor about this and presents to you for a 'second opinion'.

0

General Practice 3 - Your next patient is a mother of four, with six-month-old twins. She has brought the babies in for their immunisations, and when you ask how she is, she tells you she is really tired. She says she thought she would have been feeling better by now.

0

General Practice 3 - Your next patient is new to the practice. He tells you he has seen a couple of doctors about his problem, but nothing they have done has helped. He says he has been tired for about eight months.

0

General Practice 3 - Your patient tells you of burning and stinging when they pass urine and are worried they may have 'caught something' whilst on holiday overseas.

0

General Practice 3 - Your patient tells you they have no energy, that they have lost their 'get up and go'.

0

General Practice 3 - Your patient, who has recently had a moderately severe viral RTI, tells you they have heart palpitations and chest tightness.

0

Neonatal - Please check for hip pathology in this newborn

0

Neonatal - You are an RMO on a labour and birthing suite. You have just witnessed the birth of a baby boy at term. He is breathing infrequently, his skin is remaining blue, tone is reduced, his heart rate is 109. Please demonstrate how you would manage the infant.

0

Neonatal - You are an RMO on a labour and birthing suite. You have just witnessed the birth of a baby boy at term. He is not breathing, his skin is pale, tone is floppy, his heart rate is 65. Please demonstrate how you would manage the infant.

0

Neonatal - You are an RMO on a labour and birthing suite. You have just witnessed the birth of a baby boy at term. He is not breathing, his skin is pale, tone is floppy, his heart rate is undetectable. Please demonstrate how you would manage the infant.

0

Neonatal - You are an RMO on a labour and birthing suite. You have just witnessed the birth of a baby boy at term. He is vigorous and crying, his skin is becoming pink, tone is good, his heart rate is 123. Please demonstrate how you would manage the infant.

0

Neonatal - You are an RMO on a labour and birthing suite. You have just witnessed the birth of a baby girl at term. She is not breathing, her skin is pale, tone is floppy, her heart rate is 68. Please demonstrate how you would manage the infant.

0

Neonatal - You are an RMO on a labour and birthing suite. You have just witnessed the birth of a baby girl at term. She is not breathing, her skin is pale, tone is floppy, her heart rate is undetectable. Please demonstrate how you would manage the infant.

0

Neonatal - You are an RMO on a labour and birthing suite. You have just witnessed the birth of a baby girl at term. She is not breathing, her skin is remaining blue, tone is reduced, her heart rate is 105. Please demonstrate how you would manage the infant.

0

Neonatal - You are an RMO on a labour and birthing suite. You have just witnessed the birth of a baby girl at term. She is vigorous and crying, her skin is becoming pink, tone is good, her heart rate is 126. Please demonstrate how you would manage the infant.

0

Ophthalmology - Eyelid eversion

0

Paediatrics 1 - Behavioural and mental health problems

0

Paediatrics 1 - Carlo is a 7-year-old boy. He has tender bruises over his lower legs and joint pains. He has oedema of the scrotum and abdominal pain

0

Paediatrics 1 - Carmen is a 16 month old girl with crying and a red ear

0

Paediatrics 1 - Chin is a 7-year-old boy with recurrent tonsillitis

0

Paediatrics 1 - Christy is a 12-month-old girl. Her mother is worried that she looks very pale. You note that her conjunctiva are also pale

0

Paediatrics 1 - Dianne a 2-year-old girl with a pink swollen eyelid

0

Paediatrics 1 - Jafri a 3-year-old boy with a lazy eye

0

Paediatrics 1 - Jaime is an 14 year old boy with acute breathlessness and wheeze

0

Paediatrics 1 - Sarah is a 8-year-old girl with chronic nasal congestion, itching, mouth breathing and snoring

0

Paediatrics 1 - Shelley: is a 7-year-old girl with a bad toothache. She is febrile.

0

Paediatrics 1 - The 3 year old child who presents with behavioural problems

0

Paediatrics 1 - Tyler is an 11 month-old boy with severe atopic dermatitis

0

Paediatrics 1 - Yean is a 4-month-old boy with a scaly oily rash on his scalp

0

Paediatrics 2 - A one week old term baby girl with "apnoeas" and no evidence of infection

0

Paediatrics 2 - Bree: a 3-month-old girl who has been increasingly difficult to settle. The community health nurse has told her mother that she is "colicky'', and that this will settle with time. Both parents are exhausted and stressed

0

Paediatrics 2 - David at his neonatal check, was found by the GP to have a poor/absent red reflex in one eye"

0

Paediatrics 2 - George is a newborn who is short of breath and has increased oral secretions.

0

Paediatrics 2 - Goran: a 2-month-old boy who is not feeding well

0

Paediatrics 2 - Jack is a 36-hour-old boy who appears dusky. He has no respiratory distress. His SaO2 is 83%

0

Paediatrics 2 - Jackson is 12 months old and his testes are not down.

0

Paediatrics 2 - Jordan is a 3-day-old baby boy noted with "clicky hips" at the "discharge check"

0

Paediatrics 2 - Lee is a 4-year-old boy with high fevers for 5 days, a blanching truncal rash, conjunctivitis and cracked lips. He has cervical lymphadenopathy.

0

Paediatrics 2 - Madeleine is 10 months old, and the clinic nurse is concerned that she is slow to gain weight

0

Paediatrics 2 - Michael is a 14 month old boy who presents limp and pale to the emergency department

0

Paediatrics 2 - Peter is a 2 year old boy with fever and grunting

0

Paediatrics 2 - Rani a 3-month-old girl with a blocked tear duct

0

Paediatrics 2 - Richard is a six-week-old infant whose vomiting after feeding has suddenly increased

0

Paediatrics 2 - Ricki is a 5-year-old girl with intermittent high fevers and rashes. So far you have not been able to find an infectious cause

0

Paediatrics 2 - Tahnee has had a tongue-tie noted at her six-week check.

0

General Medicine 3 - Examine this patient for an abdominal hernia

❶ Ask the patient to lie flat with arms by the sides. ❷ Look at the obvious lump for scars, discoloration and pulsation. ❸ Ask the patient to sit up to see if the lump increases with abdominal pressure. ❹ Ask the patient if the lump has ever been painful, then palpate the lump. ❺ Ask the patient to reduce the lump. If the patient cannot, try to reduce it yourself. ❻ Feel for a cough impulse.

Oncology - Outline the Management of Malignant melanoma

❶ Complete exision with adequate margins ❷ Regional lymph node resection if advanced T depth or clinical findings ❸ Consider adjuvant therapy (high dose IV interferon-α) following resected node⊕ disease (stage III) ❹ Monitor for loco-regional recurrence, consider excision, isolated hyperthermic limb perfusion + melphalan ❺ Metastatic: Consider enrolling in targeted/chemotherapy trial if high PS. ❻ Incurable. Palliative care, e.g. steroids for symptomatic brain mets, radiotherapy for bone

General Practice 3 - Examine this patient with tiredness and low blood pressure

❶ Look for signs of dehydration, e.g. prolonged diarrhoea. ❷ Look for pigmentation in skin creases, scars and buccal mucosa (Addison's disease). ❸ Look for signs of blood loss - pallor and postural hypotension. ❹ Look for signs of iron deficiency - angular stomatitis, koilonychia. ❺ Look for vitiligo (autoimmune disorders). ❻ Feel pulse for bradycardia (heart block, Stokes-Adams attack) or tachycardia (supraventricular tachycardia, ventricular tachycardia) and irregularity (atrial fibrillation). ❼ Listen to heart - pansystolic murmur (post-myocardial infarction ventricular septal defect or mitral regurgitation), pericardial friction rub (pericarditis).

General Practice 2 - Patient presents to your GP practice complaining of a plantar wart. During the exam you notice an irregular pigmented lesion. Discuss.

Previous sun exposure[1]Previous skin problems or operations[1]Bleeding/itching/change in size[2]Duration of facial lesion[1]Advise of Dx of plantar wart[1]Offer cryotherapy or 'paint' for wart[1]Advise patient of Dx of melanoma[2]Advise need for review and explain potential seriousness of lesion.[2]Highlight urgency for this review[1]Empathy score: 2

Neonatal - Jeffrey was born vacuum assisted 3 days ago. Yellow skin all over body. Take brief Hx and outline Mx. Mother: Born at term, well throughout, antenatal checks. Large lump on his head. Mother details unremarkable. Breast feeding. Born 3410 g, now 3050 g. Meconium day 1, nothing since, normal bladder. Jaundice occurred at 36 hours. Examiner: Dx? Clinically significant? Cause? Ix? Mx? Why manage - sequelae?

Timing of onset-not 1st day[1]Mother's blood group[1]FHx of blood disorders?[1]?Birth trauma/ingested blood?[1]Mode of feeding/problems/engorgement/letdown[1]Weight loss, bowel/bladder output[1]Ix: heelprick SBR, with unconjugated/conjugated ratio[1]BSL (hypo risk)[1]Mx: Explain reasoning[1]Improve feeding e.g. express and feed, S26, top ups[1]Phototherapy[1]Sequelae: kernicterus[1]Empathy score: 2

Paediatrics 2 - Zoey is 20/12 old has a UTI confirmed by MCS of clean catch specimen. Completing appropriate course of ABs and has returned with mother for review. Please explain Mx and why. Where did it come from? Really necessary? What are you looking for? How are they done?

Zoey will need to have urine checked again 3 days after stopping Abs[1]Will need further Ix: - Renal US[1]Not painful, description of how done[0.5]Looks for anatomical abnormalities, growth/size, can show large areas of renal scarring/damage[1.5]Micturating cysourethrogram[1]A catheter (plastic tube) inserted into bladder... explain procedure[0.5]This is uncomfortable[0.5]Only way to assess if urine refluxes back up[1]Explain vesicoureteric reflux[1]Possible need for prophylactic ABs depending on presence and degree of VUR[1]Used to protect kidney from scarring/damage[1]Need to check urine in febrile illnesses, especially FUO[1]Bacteria are normal bowel flora[1]Empathy score: 2

General Medicine 1 - A 21-year-old woman presents with a history of an episode of loss of consciousness 3 months ago. She is otherwise well and plays hockey for the university's first team. Her friends witnessed it and have come to the clinic with her. The event occurred while she had been out socializing in a hot and busy pub. After standing for 25 min waiting to be served at the crowded bar she felt light-headed and became very pale and sweaty, her vision darkened and she crumpled down onto the floor. She was unrousable for a few seconds but then came round quickly with no confusion. While unrousable her limbs were observed to jerk two to three times. A medical student who was in the pub told her that she had had an epileptic 'fit'. Examination is normal and no postural blood pressure drop is noted. ❶: What is the likely differential diagnosis? ❷: What in the given history supports the diagnosis? ❸: What additional features in the history would you seek to support a particular diagnosis? ❹: What clinical examination would you perform and why? ❺: What investigations would be most helpful and why? ❻: What treatment options are appropriate?

...

General Medicine 1 - A 22-year-old woman presents with a 2-week history of headache of gradual onset. It is generalized, worse in the morning and exacerbated by coughing and straining. There is associated nausea. She started the combined oral contraceptive pill 6 weeks ago. ❶: What is the likely differential diagnosis? ❷: What in the given history supports the diagnosis? ❸: What additional features in the history would you seek to support a particular diagnosis? ❹: What clinical examination would you perform and why? ❺: What investigations would be most helpful and why? ❻: What treatment options are appropriate?

...

General Medicine 1 - A 24-year-old woman who is known to have epilepsy started fitting after a family argument and an ambulance was called. The ambulance team tell you that when they arrived in hospital she had been fitting for 30 min. She is still fitting when she arrives in hospital and appears non-responsive. Her body is thrashing violently on the bed. Her eyes are screwed tightly shut. Her oxygen saturation is 99 per cent air. Her pupils react to light and her plantar responses are downgoing. The rest of the examination is normal. ❶: What is the likely differential diagnosis? ❷: What in the given history supports the diagnosis? ❸: What additional features in the history would you seek to support a particular diagnosis? ❹: What clinical examination would you perform and why? ❺: What investigations would be most helpful and why? ❻: What treatment options are appropriate?

...

General Medicine 1 - A 25-year-old man recently attended a medical in the process of getting into the police force. He has been told that he has a heart murmur. ❶: What specific questions would you ask the patient? ❷: What is the most likely diagnosis? ❸: What examination would you perform? ❹: What would be the initial management? ❺: What investigations would you request to confirm a diagnosis? ❻: What other issues should be addressed?

...

General Medicine 1 - A 25-year-old man with 'psychiatric problems' presents with a progressive 2-year history of 'movement problems' (both at rest and on intention) and slurring of speech. His cousin died in his teens of a 'liver problem'. ❶: What is the likely differential diagnosis? ❷: What in the given history supports the diagnosis? ❸: What additional features in the history would you seek to support a particular diagnosis? ❹: What clinical examination would you perform and why? ❺: What investigations would be most helpful and why? ❻: What treatment options are appropriate?

...

General Medicine 1 - A 25-year-old woman presents to A&E with a severe gradual-onset headache. It is right sided, throbbing and periorbital, and there is associated nausea. She has had similar episodic headaches over the last 5 years and they generally last 10 h before settling. ❶: What is the likely differential diagnosis? ❷: What in the given history supports the diagnosis? ❸: What additional features in the history would you seek to support a particular diagnosis? ❹: What clinical examination would you perform and why? ❺: What investigations would be most helpful and why? ❻: What treatment options are appropriate?

...

General Medicine 1 - A 29-year-old man presents with a 10-day history of progressive, ascending leg weakness and numbness. He suffered a 'stomach bug' while on holiday in Spain about 3 weeks ago. He was previously well. ❶: What is the likely differential diagnosis? ❷: What in the given history supports the diagnosis? ❸: What additional features in the history would you seek to support a particular diagnosis? ❹: What clinical examination would you perform and why? ❺: What investigations would be most helpful and why? ❻: What treatment options are appropriate?

...

General Medicine 1 - A 30-year-old man with productive cough, feverishness and weight loss. A 30-year-old Somali man who has lived in the UK for 2 years presents with 1 month of productive cough, feverishness and weight loss. On examination he is thin but otherwise no abnormality is detected. A chest radiograph shows patchy consolidation of the right upper zone. There is no significant past medical history. ❶: What is the likely differential diagnosis? ❷: What in the given history supports the diagnosis? ❸: What additional features in the history would you seek to support a particular diagnosis? ❹: What clinical examination would you perform and why? ❺: What investigations would be most helpful and why? ❻: What treatment options are appropriate?

...

General Medicine 1 - A 30-year-old woman presents with a 3-month history of feeling generally lethargic, intermittent drooping of her left eye and intermittent double vision. She feels that her muscles are generally weaker. These symptoms are worse towards the end of the day. She has also noticed that her voice is weaker, also worse at the end of the day. ❶: What is the likely differential diagnosis? ❷: What in the given history supports the diagnosis? ❸: What additional features in the history would you seek to support a particular diagnosis? ❹: What clinical examination would you perform and why? ❺: What investigations would be most helpful and why? ❻: What treatment options are appropriate?

...

General Medicine 1 - A 30-year-old woman with prolonged fever of unknown origin. A 30-year-old woman is referred to you with a 6-month history of fever in the evenings, often accompanied by a faint erythematous rash with aches and pain, some of which clearly arise from joints. She has not lost weight. She has already been extensively investigated with a normal chest radiograph and abdominal CT. Rheumatoid antibodies and antinuclear antibodies (ANAs) are both negative. On examination you find a faint macular rash, a temperature of 39°C, a 3-cm smooth liver enlargement and 1-cm glands in both axillae. Cardiovascular examination was unremarkable. Blood cultures yielded no growth. ❶: What is the likely differential diagnosis? ❷: What in the given history supports the diagnosis? ❸: What additional features in the history would you seek to support a particular diagnosis? ❹: What clinical examination would you perform and why? ❺: What investigations would be most helpful and why? ❻: What treatment options are appropriate?

...

General Medicine 1 - A 34-year-old woman presented with a 3-hour history of palpitations. She is mildly dyspnoeic and has some chest pain. She has been feeling anxious and suffering from insomnia for the last few weeks. On direct questioning she admits to a 10-day history of diarrhoea. On examination she has a tachycardia and her pulse is irregular. ❶: What specific questions would you ask the patient? ❷: What is the most likely diagnosis? ❸: What examination would you perform? ❹: What would be the initial management? ❺: What investigations would you request to confirm a diagnosis? ❻: What other issues should be addressed?

...

General Medicine 1 - A 35-year-old white woman presents with a 3-day history of progressive numbness below the umbilicus, with mild weakness of the lower limbs and urinary frequency. Five years previously she suffered a subacute episode of uncomfortable visual loss in the right eye with full recovery over a week. ❶: What is the likely differential diagnosis? ❷: What in the given history supports the diagnosis? ❸: What additional features in the history would you seek to support a particular diagnosis? ❹: What clinical examination would you perform and why? ❺: What investigations would be most helpful and why? ❻: What treatment options are appropriate?

...

General Medicine 1 - A 40-year-old man (previously well) presents with a 6-day history of gradual-onset worsening headache, lethargy, myalgia and fever, and (by the sixth day) altered behaviour. He is brought to hospital after his wife called an ambulance because he had a generalized seizure. ❶: What is the likely differential diagnosis? ❷: What in the given history supports the diagnosis? ❸: What additional features in the history would you seek to support a particular diagnosis? ❹: What clinical examination would you perform and why? ❺: What investigations would be most helpful and why? ❻: What treatment options are appropriate?

...

General Medicine 1 - A 40-year-old woman attends the accident and emergency department (A&E) with her worst-ever headache. It started suddenly in the occipital area 5 h ago and was associated with nausea and vomiting and some photophobia. Her neck became stiff after around 2 h. ❶: What is the likely differential diagnosis? ❷: What in the given history supports the diagnosis? ❸: What additional features in the history would you seek to support a particular diagnosis? ❹: What clinical examination would you perform and why? ❺: What investigations would be most helpful and why? ❻: What treatment options are appropriate?

...

General Medicine 1 - A 53-year-old man is brought into the accident and emergency department (A&E) by ambulance complaining of sudden onset of severe central chest pain. He is cold and clammy, and appears breathless. ❶: What specific questions would you ask the patient? ❷: What is the most likely diagnosis? ❸: What examination would you perform? ❹: What would be the initial management? ❺: What investigations would you request to confirm a diagnosis? ❻: What other issues should be addressed?

...

General Medicine 1 - A 55-year-old man with diabetes presents with a 4-day history of progressive painful right periorbital pain, drooping of the right eyelid and double vision. When he lifted his eyelid he noted that his right eye seemed to be deviated 'down and out' and he is unable to move it. ❶: What is the likely differential diagnosis? ❷: What in the given history supports the diagnosis? ❸: What additional features in the history would you seek to support a particular diagnosis? ❹: What clinical examination would you perform and why? ❺: What investigations would be most helpful and why? ❻: What treatment options are appropriate?

...

General Medicine 1 - A 58-year-old previously well man suffers an episode of loss of consciousness just after leaving the golf club bar with his friends. The patient smells of urine, seems a bit dazed, has a mild headache and feels achy, but otherwise is normal. He can't recall anything about the incident. He is a smoker. ❶: What is the likely differential diagnosis? ❷: What in the given history supports the diagnosis? ❸: What additional features in the history would you seek to support a particular diagnosis? ❹: What clinical examination would you perform and why? ❺: What investigations would be most helpful and why? ❻: What treatment options are appropriate?

...

General Medicine 1 - A 60-year-old woman attends with gradual-onset, very slowly progressive tremor in both hands over 10 years. The tremor is not present at rest. She finds it difficult to drink or to use cutlery. She comments that her grandfather, mother and brother had or have similar symptoms. She finds that a small tot of whisky helps ease the tremor. ❶: What is the likely differential diagnosis? ❷: What in the given history supports the diagnosis? ❸: What additional features in the history would you seek to support a particular diagnosis? ❹: What clinical examination would you perform and why? ❺: What investigations would be most helpful and why? ❻: What treatment options are appropriate?

...

General Medicine 1 - A 65-year-old man was brought into A&E complaining of severe tearing pain between the shoulder blades. Initially the pain was extremely severe but has now mildly subsided to a dull ache in the chest. He is normally fit and well apart from hypertension, for which he has been seeing the GP. ❶: What specific questions would you ask the patient? ❷: What is the most likely diagnosis? ❸: What examination would you perform? ❹: What would be the initial management? ❺: What investigations would you request to confirm a diagnosis? ❻: What other issues should be addressed?

...

General Medicine 1 - A 65-year-old man with known IHD, hypertension and diabetes mellitus has recently been complaining of shortness of breath. An echocardiogram has confirmed left ventricular (LV) systolic dysfunction, which is moderately severe, and he has been started on ramipril 10 mg once daily, bisoprolol 2.5 mg once daily and spironolactone 25 mg once daily. He is very concerned about the number of medications that he is now on because these are additional to his other therapies. Explain the importance of his diagnosis and treatments to him.

...

General Medicine 1 - A 65-year-old man with type 2 diabetes (now requiring insulin for the last 5 years) presents with a progressive, ascending, symmetrical numbness and burning. The symmetrical numbness and burning discomfort initially involved his feet (and more recently his ankles) over the last 8 years. ❶: What is the likely differential diagnosis? ❷: What in the given history supports the diagnosis? ❸: What additional features in the history would you seek to support a particular diagnosis? ❹: What clinical examination would you perform and why? ❺: What investigations would be most helpful and why? ❻: What treatment options are appropriate?

...

General Medicine 1 - A 68-year-old, previously active, woman is now unable to walk or stand having sustained a stroke (right middle cerebral artery territory infarction) 3 weeks previously. She has a dense left hemiparesis, homonymous hemianopia and sensory disturbance. She is also dysphasic and has difficulty swallowing. Recovery has been complicated by aspiration pneumonia. She has a urinary catheter and needs regular toileting. She is in AF, BP 142/78 mmHg. She is reluctant to engage with her programme of rehabilitation. ❶: What treatment options are available? ❷: In addition to regular medical and nursing attention, which members of the multidisciplinary team ought to be involved with her care? ❸: What medical complications might supervene during the next 4-6 weeks?

...

General Medicine 1 - A 70-year-old woman presents with a gradual-onset diffuse headache, which is more severe in the left temporal region. She comments that wearing her spectacles has become uncomfortable and combing her hair exacerbates the pain. ❶: What is the likely differential diagnosis? ❷: What in the given history supports the diagnosis? ❸: What additional features in the history would you seek to support a particular diagnosis? ❹: What clinical examination would you perform and why? ❺: What investigations would be most helpful and why? ❻: What treatment options are appropriate?

...

General Medicine 1 - A 71-year-old man has been brought to the accident and emergency department (A&E) after a fall whilst shopping. He lost consciousness momentarily and has little recollection of the event. He admits to previous episodes of dizziness, sometimes on exertion and often associated with shortness of breath. His past history includes a hiatus hernia, diverticular disease, and osteoarthritis of his neck and knees. He is taking regular omeprazole, a dietary fibre supplement and a compound analgesic containing paracetamol and codeine (codamol). Examination discloses a regular pulse and blood pressure (BP) of 114/76 mmHg. There is an ejection systolic cardiac murmur radiating to the neck. The lung fields are clear. Neurological examination is normal. ❶: What is the differential diagnosis? ❷: What features in the history support the diagnosis? ❸: What additional features in the history would you seek to support the potential diagnoses? ❹: What other features would you look for on clinical examination? ❺: What investigations would you perform? ❻: What treatment options are available?

...

General Medicine 1 - A 74-year-old man who has difficulty walking is referred to the elderly medicine day hospital. He is confined to his chair unless assisted to stand. He cannot get into bed at night. His past history includes neck surgery for cervical spondylosis. He has hypertension treated with nifedipine. He is obese, his legs are oedematous and he is incontinent of urine. He has ulceration and cellulitis affecting his lower legs. CNS examination reveals normal cognition and cranial nerves, but weak arms and legs (power 4/5), brisk reflexes bilaterally and upgoing plantar responses. His feet are warm but foot pulses are impossible to assess as a result of oedema. ❶: What is the differential diagnosis? ❷: What features in the history support the diagnosis? ❸: What additional features in the history would you seek to support the potential diagnoses? ❹: What other features would you look for on clinical examination? ❺: What investigations would you perform? ❻: What treatment options are available?

...

General Medicine 1 - A 74-year-old man with a history of alcohol abuse was admitted to hospital 24 hours ago after two witnessed epileptic seizures. He recovered fully and was initially lucid, but is now very restless and agitated. He seems to be hallucinating. Examination is difficult. He has a low-grade fever, and is tremulous and sweating. There are no obvious focal neurological signs. He is not jaundiced or anaemic. BP 178/84 mmHg; HR 106/min, AF. ❶: List the most likely causes of his or her confusional state ❷: What initial investigations should you perform? ❸: What treatment would you consider appropriate?

...

General Medicine 1 - A 80-year-old woman collapsed at home and has been brought into A&E unconscious. She was given basic life support at home by her daughter who happens to work as a ward sister on coronary care. The paramedics are performing CPR as the patient is wheeled into A&E accompanied by her tearful daughter. ❶: What specific questions would you ask the patient? ❷: What is the most likely diagnosis? ❸: What examination would you perform? ❹: What would be the initial management? ❺: What investigations would you request to confirm a diagnosis? ❻: What other issues should be addressed?

...

General Medicine 1 - A house-bound 94-year-old woman with epilepsy (controlled with phenytoin) presents with falls, weakness and generalized aches and pains. Biochemical tests reveal: • Calcium (corrected) 1.85 mmol/L (normal range 2.05-2.60 mmol/L) • Phosphate 0.68 mmol/L (normal range 0.8-1.45 mmol/L) • Albumin 32 g/L (normal range 35-48 g/L) • Alkaline phosphatase (ALP) 458 U/L (normal range 30-200 U/L). ❶: What is the likely diagnosis? ❷: What factors might have precipitated this condition? ❸: What signs would you look for on examination? ❹: How would you treat the condition?

...

General Medicine 1 - A previously well 39-year-old man attends A&E with a left-sided headache, neck pain and and slight word-finding difficulty after a mild whiplash injury 2 hours earlier. He says his right arm doesn't 'feel right'. His wife has noticed a slight drooping of the eyelid on the left. ❶: What is the likely differential diagnosis? ❷: What in the given history supports the diagnosis? ❸: What additional features in the history would you seek to support a particular diagnosis? ❹: What clinical examination would you perform and why? ❺: What investigations would be most helpful and why? ❻: What treatment options are appropriate?

...

General Medicine 1 - A woman of 30 who has fainted and who has fever and a rash. A 30-year-old woman was well until 48 h before presentation. Her illness began with nausea and vomiting and later she felt feverish. On the morning of admission she had got up, then felt faint and briefly lost consciousness. On admission her temperature is 39°C, pulse 110/min, blood pressure 100/60 mmHg supine. There is a diffuse erythematous rash on her face and trunk which blanches on pressure. She is menstruating. ❶: What is the likely differential diagnosis? ❷: What in the given history supports the diagnosis? ❸: What additional features in the history would you seek to support a particular diagnosis? ❹: What clinical examination would you perform and why? ❺: What investigations would be most helpful and why? ❻: What treatment options are appropriate?

...

General Medicine 1 - An 82-year-old woman presents with recurrent falls. She is housebound. Her most recent fall occurred while getting up to go to the toilet at night. She has arthritis, heart failure and poor vision as a result of macular degeneration, and has had a previous hip replacement. She receives treatment with furosemide, ramipril, co-codamol (paracetamol/codeine compound analgesic), dothiepin and temazepam. She is very frail. The main findings on examination are: abbreviated mental test 6/10, vision - large print only, kyphotic spine, brisk reflexes bilaterally; peripheral oedema, clear lung fields and normal jugular venous pressure (JVP); heart rate (HR) 112/min atrial fibrillation (AF); BP sitting 114/62 mmHg, mitral regurgitation. ❶: What is the differential diagnosis? ❷: What features in the history support the diagnosis? ❸: What additional features in the history would you seek to support the potential diagnoses? ❹: What other features would you look for on clinical examination? ❺: What investigations would you perform? ❻: What treatment options are available?

...

General Medicine 1 - An 84-year-old woman is found wandering in the street at night by the neighbours and is brought to hospital. She is very disorientated and confused. She is alert, but restless and agitated. She is convinced that she is late for work. A telephone call to her niece confirms that the patient has been somewhat confused and forgetful for the last 9 months or so. Further enquiry reveals that she needs help with her household and financial affairs. She has never wandered before. Examination discloses AF (126/min), cardiomegaly, mitral incompetence and peripheral oedema. Her feet are in a state of neglect and there is an infected bunion. Abbreviated mental test is 4/10. There are no focal neurological signs. The remainder of the examination is unremarkable. Q3 is not applicable in this case. ❶: List the most likely causes of his or her confusional state ❷: What initial investigations should you perform? ❸: What treatment would you consider appropriate? ❹: The niece is worried about her aunt returning home alone at the conclusion of the hospital admission. How would you address this concern?

...

General Medicine 1 - The wife of an elderly man is distressed because her husband's attendance at the local elderly care day hospital for physiotherapy has been discontinued. He unfortunately suffered a severe stroke 3 months ago and is now confined to a wheelchair. He has difficulty speaking as a result of dysphasia. What advice should be given?

...

General Medicine 1 - This 73-year-old man has recently been diagnosed with angina. He is able to walk 300 metres before experiencing chest tightness, but considerably less when there is an incline. His recent exercise stress test has indicated ischaemic heart disease (IHD) and he has been prescribed sublingual GTN (glyceryl trinitrate) tablets and a β-adrenoreceptor blocker. Please advise him on how to use the medication and explain any side effects of which he should be aware.

...

General Medicine 1 - You have been asked to see an 83-year-old man who has been brought to A&E. He lives in a residential home, and for the last 2 days he has become confused. He is not eating or drinking and his walking has deteriorated. He has Parkinson's disease, a history of angina and type 2 diabetes mellitus. His medication includes cobeneldopa, selegiline, gliclazide, a nitrate preparation and aspirin. On examination the significant findings are as follows: • Abbreviated mental test: 4/10; he is agitated and restless • His tongue is dry • BP 108/62 mmHg • HR 104/min regular • His chest is clear • Marked signs of parkinsonism (rigidity, tremor and bradykinesia) • Urinalysis: protein ++, blood +, glucose ++ and ketones negative. ❶: List the most likely causes of his or her confusional state ❷: What initial investigations should you perform? ❸: What treatment would you consider appropriate?

...

General Medicine 1 - You have been called to see a 72-year-old man who lives alone at home. The district nurse has become increasingly concerned about his health. The patient has been house-bound for some time, but is now unable to rise from his chair. He is a large man and his left hip is very painful. There is no history of a fall. He is unkempt and his clothes smell of urine. He is a heavy smoker, and is breathless and wheezy with a chronic cough. His legs are swollen and blistered. Detailed examination is difficult, but does reveal signs of airflow obstruction, cyanosis, elevated JVP and peripheral oedema. Movement at the left hip is restricted and painful. He is very reluctant to leave his home. ❶: What investigations would you perform? ❷: What treatment options are available? ❸: What are the likely causes of his immobility? ❹: What can be done to improve his situation at home?

...

General Medicine 2 - A 30-year-old man attends A&E complaining of chest pain and breathlessness. The pain developed suddenly, 16 hours before attendance, and he has gradually become increasingly short of breath since. The pain is described as sharp, worse on inspiration or coughing, and located on the right thoracic chest wall. He denies new cough or sputum production. ❶: What is the likely differential diagnosis? ❷: What issues in the given history support the diagnosis? ❸: What additional features of the history would you seek to support a particular diagnosis? ❹: What clinical examination would you perform and why? ❺: What investigations would be most helpful and why? ❻: What treatment options are appropriate?

...

General Medicine 2 - A 34-year-old man was found to have microscopic haematuria on an insurance medical examination. He was asymptomatic. His BP was 124/80 mmHg and his serum creatinine 102 μmol/L. ❶: What is the likely diagnosis? ❷: How would you investigate this case? ❸: How would you manage the case? ❹: What is the prognosis?

...

General Medicine 2 - A 45-year-old mother of two presents at A&E on a Sunday night complaining of coughing large volumes of fresh red blood for the last 2 days. She estimates that she has coughed up five mugs of fresh blood. She describes a cough with purulent sputum for the last month, but cannot remember a time when she did not have a cough. She frequently receives antibiotics from her GP when the sputum turns green or increases in volume. She has been increasingly breathless for the last 2 weeks, although she has noticed increasing breathlessness for 3 years ❶: What is the likely differential diagnosis? ❷: What issues in the given history support the diagnosis? ❸: What additional features of the history would you seek to support a particular diagnosis? ❹: What clinical examination would you perform and why? ❺: What investigations would be most helpful and why? ❻: What treatment options are appropriate?

...

General Medicine 2 - A 49-year-old man feels unwell with temperatures, myalgias, and pain in his wrists, knees and ankles. He is loosing weight and cannot work. His left eye has become increasingly sore and red. For some time he has had a cough productive of yellow/white sputum but today coughed up red blood. Investigations reveal a haemoglobin (Hb) of 10.8 g/dL, white cell count (WCC) of 14.8 × 109/L and ESR of 98 mm/h. Urinalysis demonstrated blood +++ and protein ++. Apart from a 3- year history of sinusitis requiring antibiotics and nasal spray, there is no significant past medical history. ❶: What is the likely differential diagnosis? ❷: How would you investigate this case? ❸: How would you confirm the likely diagnosis? ❹: How would you initially manage the case? ❺: What are the principles of long-term management? ❻: What is the prognosis?

...

General Medicine 2 - A 60-year-old man attends the medical outpatient clinic complaining of 4 weeks of cough productive of white sputum and 2 weeks of haemoptysis. The haemoptysis is described as 'streaks of old blood through the spit', and is trivial in volume. It has remained constant throughout the 2-week period. There is no dyspnoea or chest pain, but he is a life-long smoker. He has noticed his clothes being looser over the previous 6 months and describes a loss of appetite, but denies dysphagia or altered bowel habit. He is easily tired and notices his voice becoming hoarse in the evenings. He is a bookmaker and is married with two daughters, one of whom attends with him. She adds that her father appears slightly confused at times, particularly in the evenings. ❶: What is the likely differential diagnosis? ❷: What issues in the given history support the diagnosis? ❸: What additional features of the history would you seek to support a particular diagnosis? ❹: What clinical examination would you perform and why? ❺: What investigations would be most helpful and why? ❻: What treatment options are appropriate?

...

General Medicine 2 - A 62-year-old man attends the respiratory clinic complaining of worsening cough, spit and haemoptysis. This has been of 2 months' duration and is associated with weight loss, anorexia, fevers, night sweats and purulent sputum. He denies chest discomfort or dyspnoea. He lives alone, smokes 20 cigarettes per day and admits to drinking in excess of 40 units of alcohol per week. As a child he was treated for TB at the same time as his father. He was a welder in a shipyard before being made redundant. ❶: What is the likely differential diagnosis? ❷: What issues in the given history support the diagnosis? ❸: What additional features of the history would you seek to support a particular diagnosis? ❹: What clinical examination would you perform and why? ❺: What investigations would be most helpful and why? ❻: What treatment options are appropriate?

...

General Medicine 2 - A 62-year-old woman presented to her GP with symptoms of tiredness, lethargy, aching joints, poor appetite and nausea of 4-6 weeks' duration. Physical examination was unremarkable except pallor and a BP of 160/100 mmHg. Routine blood tests revealed: haemoglobin (Hb) 10 g/dL, blood urea 26 mmol/L and serum creatinine 386 μmol/L (glomerular filtration rate or GFR 14 mL/min). The patient was referred to the local hospital. Further examination revealed purpuric spots in both legs and +++ blood and +++ protein on urinalysis. ❶: What is the likely diagnosis? ❷: How would you investigate this case? ❸: How would you manage the case? ❹: What is the prognosis?

...

General Medicine 2 - A 66-year-old man presents complaining of worsening exercise tolerance as a result of breathlessness. He can walk 200 metres on a flat road whereas 1 year ago he could manage at least 1 mile. He also describes feeling short of breath at rest. He is severely restricted on hills or stairways. He denies cough, but has lost 3 kg over the last 6 months. He is a life-long non-smoker and worked in the ship-building industry where he was exposed to asbestos fibres. His GP describes inspiratory crackles at both lung bases. ❶: What is the likely differential diagnosis? ❷: What issues in the given history support the diagnosis? ❸: What additional features of the history would you seek to support a particular diagnosis? ❹: What clinical examination would you perform and why? ❺: What investigations would be most helpful and why? ❻: What treatment options are appropriate?

...

General Medicine 2 - A 66-year-old retired publican attends the respiratory outpatient clinic complaining of severe exercise limitation as a result of breathlessness. He has trouble moving about the house and rarely goes outside. He feels worse in the mornings and describes wheeze. His symptoms have developed over 3 years and his GP has tried inhalers but they have not helped. He has had a cough with sputum for more than 10 years. He has been a smoker for more than 40 years and has smoked 5-10 cigarettes per day over this time. ❶: What is the likely differential diagnosis? ❷: What issues in the given history support the diagnosis? ❸: What additional features of the history would you seek to support a particular diagnosis? ❹: What clinical examination would you perform and why? ❺: What investigations would be most helpful and why? ❻: What treatment options are appropriate?

...

General Medicine 2 - A 72-year-old man has been referred by the GP to A&E with a history of being non-specifically unwell for 2 days. He has a past history of ischaemic heart disease, atrial fibrillation, congestive cardiac failure and stable CRF. His regular medications include isosorbide mononitrate, digoxin, furosemide, spironolactone and an ACE inhibitor. On examination the patient is mildly confused, dehydrated, peripherally cold, bradycardic and hypotensive (BP 96/60 mmHg). His initial blood tests show Na+ 130 mmol/L, K+ 7.2 mmol/L, urea 42 mmol/L and creatinine of 412 μmol/L (GFR 14 mL/min). ❶: What is the likely diagnosis? ❷: How would you investigate this case? ❸: What would be the initial management? ❹: What would be the long-term management?

...

General Medicine 2 - A 76-year-old man presented to his GP with a history of difficulty in passing urine in the form of hesitancy, poor stream and occasional pain while passing urine of 3 months' duration. He also suffered from nocturia. Physical examination was unremarkable. Urinalysis showed presence of + for each of blood and protein. Blood test showed a raised serum creatinine of 560 μmol/L (GFR 11 mL/min). The patient was urgently referred to the nearby district general hospital. Further examination in the hospital revealed a palpable bladder and a smoothly enlarged prostate on per rectal examination. ❶: What is the likely diagnosis? ❷: How would you investigate this case? ❸: How would you manage the case? ❹: What is the prognosis?

...

General Medicine 2 - An elderly hypertensive woman is attending the surgery for regular assessment of her BP. On this occasion her daughter attends as well and mentions that the family is very concerned about the patient's apparent incontinence of urine. The daughter goes on to say that her mother's house and clothing have now developed a constant smell of urine. On direct questioning the patient makes light of this problem. She seems sensible. How would you proceed?

...

General Medicine 2 - Please answer the patient's questions: ❶: I have recently been diagnosed as having chronic kidney disease. What problems can I expect to have? ❷: As a patient with chronic kidney disease, do I have a higher risk of heart disease? ❸: As a patient with chronic kidney disease, do I need to avoid any drugs? ❹: As a patient with kidney failure, do I need to stick to a diet? ❺: If I need to go on dialysis treatment what are the treatment options?

...

General Medicine 3 - 'Do all people with diabetes end up on kidney dialysis treatment?' ❶: Please counsel the patient.

...

General Medicine 3 - A 30-year-old ex-drug abuser presents with feeling vaguely unwell. His GP has obtained liver function tests, but are mildly abnormal with an alanine aminotransferase (ALT) of 50 IU/L (normal range < 40 IU/L). ❶: What is the likely differential diagnosis? ❷: What in the given history supports the diagnosis? ❸: What additional features in the history would you seek to support a particular diagnosis? ❹: What clinical examination would you perform and why? ❺: What investigations would be most helpful and why? ❻: What treatment options are appropriate? ❼: Doctor, I have given up my drug habit, but have hepatitis C. Do I need to be treated?

...

General Medicine 3 - A 35-year-old woman has completed 6 months of anticoagulation therapy after having a lifethreatening pulmonary embolus. She now wonders if she should consider oral contraception rather than risk further pregnancy. This 35-year-old woman was finally diagnosed as having a life-threatening PE. Having received 6 months of anticoagulation therapy she returns to her doctor for advice. Her main concerns surround whether she should now consider oral contraception rather than risk further pregnancy. ❶: What risks exist and how should the GP advise her?

...

General Medicine 3 - A 35-year-old woman presents with a feeling of discomfort after eating meals. She describes a fullness after eating a modest amount, and can no longer finish the meal that she used to eat. The indigestion is also troublesome. ❶: What is the likely differential diagnosis? ❷: What in the given history supports the diagnosis? ❸: What additional features in the history would you seek to support a particular diagnosis? ❹: What clinical examination would you perform and why? ❺: What investigations would be most helpful and why? ❻: What treatment options are appropriate?

...

General Medicine 3 - A 48-year-old man with alcohol problems presents with upper abdominal pain with some radiation to the back. ❶: What specific questions would you ask the patient? ❷: What is the most likely diagnosis? ❸: What examination would you perform? ❹: What would be the initial management? ❺: What investigations would you request to confirm a diagnosis? ❻: What other issues should be addressed? ❼: Why should I stop drinking?

...

General Medicine 3 - A 57-year-old man with type 2 diabetes is currently treated with gliclazide 160 mg twice daily and metformin 500 mg three times daily, but his glycaemic control is suboptimal. He reports blood glucose levels between 12 and 20 mmol/L) and has an elevated HbA1c level (11.0 per cent). He claims good dietary adherence but is overweight (110 kg, BMI 33 kg/m2). He denies any symptoms and has previously refused to consider insulin. He has normal VA but fundal photography shows hard exudates encroaching on the left macula. ❶: What are the treatment options? ❷: What factors influence the choice of therapy? ❸: What would be the preferred choice of treatment? ❹: How would the impact of treatment be assessed?

...

General Medicine 3 - A 57-year-old woman presents to the surgery with lethargy. She has a long history of recurrent episodes of vaginal irritation, which had been formally diagnosed as candidal thrush on one occasion. Treatment with clotrimazole leads to a short amelioration of symptoms. A fasting plasma glucose level has been reported by the laboratory to be 8.2 mmol/L. ❶: What specific questions would you ask the patient? ❷: What investigations would you request to confirm a diagnosis? ❸: What examination would you perform? ❹: What would be the initial management? ❺: What are the long-term sequelae? ❻: What issues need to be addressed apart from blood glucose control?

...

General Medicine 3 - A 60-year-old woman presents with a 2-day history of abdominal distension, pain, profuse bilious vomiting and constipation. ❶: What is the likely differential diagnosis? ❷: What in the given history supports the diagnosis? ❸: What additional features in the history would you seek to support a particular diagnosis? ❹: What clinical examination would you perform and why? ❺: What investigations would be most helpful and why? ❻: What treatment options are appropriate?

...

General Medicine 3 - A 65-year-old woman is reviewed in clinic following recent colonic surgery. Shortly after surgery she developed a hot swollen leg from a deep vein thrombosis (DVT), confirmed on duplex scanning. She has been started on anticoagulation therapy. She is upset that this developed and wants to you to explain why it occurred and what was done to prevent it. How would you approach answering this woman?

...

General Medicine 3 - A 67-year-old woman presented with diarrhoea. She describes her stools as pale. You discover that she has lost weight, and has nocturia and thirst. ❶: What specific questions would you ask the patient? ❷: What is the most likely diagnosis? ❸: What examination would you perform? ❹: What would be the initial management? ❺: What investigations would you request to confirm a diagnosis? ❻: What other issues should be addressed?

...

General Medicine 3 - A 68-year-old man with type 2 diabetes has a new person check in primary care. He has no other significant medical history apart from diabetes and he does not smoke. His BMI is 32 kg/m2 and he has no evidence of diabetic retinopathy or foot complications. BP 170/96 mmHg and urinalysis shows glycosuria ++ but no protein. A random total cholesterol (TC) is 5.6 mmol/L and triglycerides 2.3 mmol/L. He takes metformin 1 g twice daily and has an HbA1c level of 7.5 per cent (normal range < 5 per cent). ❶: Are further assessments required before the italicized finding provokes a drug intervention? ❷: If drug therapy were indicated, which would be the most appropriate choice of agent? ❸: What sort of follow-up is indicated? ❹: What other issues need to be addressed as regards cardiovascular risk?

...

General Medicine 3 - A 70-year-old man is brought into A&E complaining of abdominal pain. Over the last few months he had noted a change in bowel habit and a loss of weight. On examination he is pale, tachycardic and hypotensive, and his abdomen is rigid on palpation. ❶: What is the likely differential diagnosis? ❷: What in the given history supports the diagnosis? ❸: What additional features in the history would you seek to support a particular diagnosis? ❹: What clinical examination would you perform and why? ❺: What investigations would be most helpful and why? ❻: What treatment options are appropriate?

...

General Medicine 3 - A 73-year-old man is found to have glycosuria on urine testing as part of a new patient assessment. A random finger-prick test by the practice nurse using a modern, calibrated, glucose meter gives a level of 11.4 mmol/L. Arrangements are madefor a repeat blood test after an overnight fast; this plasma glucose level is analysed by the local hospital clinical chemistry department and is reported at 6.7 mmol/L. ❶: What specific questions would you ask the patient? ❷: What investigations would you request to confirm a diagnosis? ❸: What examination would you perform? ❹: What would be the initial management? ❺: What are the long-term sequelae? ❻: What issues need to be addressed apart from blood glucose control?

...

General Medicine 3 - A 73-year-old man presents with a 5-day history of passing black stools. On the fifth day he began to feel breathless and tired, particularly on climbing stairs. ❶: What specific questions would you ask the patient? ❷: What is the most likely diagnosis? ❸: What examination would you perform? ❹: What would be the initial management? ❺: What investigations would you request to confirm a diagnosis? ❻: What other issues should be addressed?

...

General Medicine 3 - A 75-year-old man with diabetes of 15 years' standing complains of difficulty in seeing the TV. There has been no pain or redness affecting his eyes and he has not had headaches. His diabetic control is poor but stable. ❶: What questions would you ask the patient and why? ❷: What investigations would you request? ❸: What examination would you perform? ❹: What are the differential diagnoses related to diabetes?

...

General Medicine 3 - A 76-year-old woman with type 2 diabetes is under regular review. She has a history of angina, which is well controlled, and does not smoke. Her BMI is 28 kg/m2 and she has background diabetic retinopathy and absent foot pulses. BP 166/70 mmHg and urinalysis shows persistent + albuminuria in the absence of infection. BP recordings over the previous 6 months have shown similar levels. She takes gliclazide 80 mg twice daily, and isosorbide mononitrate 60 mg, simvastatin 10 mg and aspirin 75 mg once daily. A random TC is 4.8 mmol/L, triglycerides 2.3 mmol/L. and HbA1c 7.8 per cent (normal range < 5 per cent). ❶: Are further assessments required before the italicized finding provokes a drug intervention? ❷: If drug therapy were indicated, which would be the most appropriate choice of agent? ❸: What sort of follow-up is indicated? ❹: What other issues need to be addressed as regards cardiovascular risk?

...

General Medicine 3 - A 77-year-old man has been found collapsed at home by a care assistant. He was previously independent but known to have hypertension, angina and osteoarthritis. His regular medication was bendrofluazide 5 mg once daily, atenolol 50 mg once daily and aspirin. He had been noted to be lethargic over the previous few weeks and had been incontinent of urine on a couple of occasions. On arrival in the accident and emergency department (A&E) he was drowsy but orientated. Vital signs were normal but he was clinically dehydrated. Urinalysis showed ++++ glucose, ++ protein and + ketones. Plasma glucose was 84 mmol/L. ❶: What is the likely diagnosis? ❷: What investigations would be performed? ❸: How would the diagnosis be confirmed? ❹: What would be the initial management? ❺: What are the potential complications? ❻: What issues need to be addressed when the patient has fully recovered?

...

General Medicine 3 - A 78-year-old man with Parkinson's disease is admitted from a nursing home with faecal incontinence. Previously he had been noted to have stubborn bowels. Both he and the nurses were surprised when he began staining his clothing. ❶: What is the likely differential diagnosis? ❷: What in the given history supports the diagnosis? ❸: What additional features in the history would you seek to support a particular diagnosis? ❹: What clinical examination would you perform and why? ❺: What investigations would be most helpful and why? ❻: What treatment options are appropriate? ❼: Doctor, why can't my daughter simply take senna when she has constipation?

...

General Medicine 3 - A 78-year-old woman, disabled by stroke, is cared for by her daughter. Care at home is proving difficult as a result of persistent urinary incontinence such that a nursing home placement is now being considered. The patient has a left hemiparesis and dysphasia, and needs help to stand, transfer and walk. She receives treatment with aspirin (for stroke), amlodipine (for hypertension) and furosemide (for swollen ankles). The patient is alert and responsive, but has difficulty communicating as a result of her dysphasia. Sometimes she is able to indicate that the lavatory is needed, but is usually wet before toileting can be achieved. In addition to the signs of stroke, examination shows an excoriated perineum. There is no palpable bladder. Rectal examination is normal. There is no uterovaginal prolapse. ❶: What are the likely causes of the problem? ❷: How should the problem be investigated? ❸: What can be done to help?

...

General Medicine 3 - A 90-year-old woman who lives alone has slipped and fallen in her bathroom at home. She has escaped serious injury. Subsequent visits by the district nurse to assess the situation reveal that the elderly woman's clothing is always soaked in urine and the house has a strong smell of urine. The patient is reluctant to discuss the situation but does admit to a swelling 'down below'. She is sensible and independent in her activities of daily living. She is reluctant to be examined by the nurse, but in due course assessment reveals a swelling at the vaginal introitus, which on further examination is shown to be the cervix. When the patient stands up she leaks urine. Her bowel function is normal. There are no other abnormal findings. In particular there is no palpable bladder, neurological examination is normal and urinalysis is negative. ❶: What are the likely causes of the problem? ❷: What treatments should be considered? ❸: Suggest three reasons why the incontinence has gone unreported by the patient for so long

...

General Medicine 3 - A fit 84-year-old women with type 2 diabetes had been started on gliclazide after strict dietary adherence had failed to control her symptoms. She now presents with persistently high glucose levels (home blood glucose monitoring or HBGM 12-15 mmol/L) and has an elevated HbA1c level (9.0 per cent) despite taking gliclazide, 160 mg in the morning, 80 mg in the evening. She is symptomatic with lethargy, frequency and dysuria. Random fingerstick glucose is 15 mmol/L and urine dipstick shows ++++ glucose, ++ albumin and + haematuria. BP is normal, weight is 67 kg (BMI 24 kg/m2) and she has no evidence of retinopathy. There is early peripheral neuropathy and foot pulses are absent. ❶: What are the treatment options? ❷: What factors influence the choice of therapy? ❸: What would be the preferred choice of treatment? ❹: How would the impact of treatment be assessed?

...

General Medicine 3 - A patient with coeliac disease who infrequently attends clinic. He presents with weight loss and anaemia at the insistence of his wife. ❶: What is the likely differential diagnosis? ❷: What in the given history supports the diagnosis? ❸: What additional features in the history would you seek to support a particular diagnosis? ❹: What clinical examination would you perform and why? ❺: What investigations would be most helpful and why? ❻: What treatment options are appropriate?

...

General Medicine 3 - A retired publican presents with progressive jaundice and abdominal swelling. He drinks more than 40 units of alcohol per week. Previously he has been admitted with fever and vomiting, which he was told was caused by alcohol. After several days in hospital, the patient with jaundice and ascites becomes confused, disorientated and aggressive. A couple of days after showing evidence of encephalopathy, this patient with chronic liver disease has haematemesis. ❶: What is the likely differential diagnosis? ❷: What in the given history supports the diagnosis? ❸: What additional features in the history would you seek to support a particular diagnosis? ❹: What clinical examination would you perform and why? ❺: What investigations would be most helpful and why? ❻: What treatment options are appropriate? ❼: Should my husband have a transplant? Why is my husband confused?

...

General Medicine 3 - A teenage girl presents with abdominal discomfort and a story of passing pellet-like stools. She experiences abdominal bloating before defecation and after defecation feels that she has not adequately emptied her bowels. ❶: What is the likely differential diagnosis? ❷: What in the given history supports the diagnosis? ❸: What additional features in the history would you seek to support a particular diagnosis? ❹: What clinical examination would you perform and why? ❺: What investigations would be most helpful and why? ❻: What treatment options are appropriate?

...

General Medicine 3 - After an elective period in India, a medical student returned to Australia with diarrhoea. The diarrhoea initially appeared to settle and then has gradually worsened over the 2 months before coming to see you. ❶: What is the likely differential diagnosis? ❷: What in the given history supports the diagnosis? ❸: What additional features in the history would you seek to support a particular diagnosis? ❹: What clinical examination would you perform and why? ❺: What investigations would be most helpful and why? ❻: What treatment options are appropriate? ❼: Doctor, I have severe bloating. Is it the result of food allergy? ❽: I have been told that I have IBS. Should I take a fibre supplement?

...

General Medicine 3 - An 18-year-old male smoker presents with colicky, central, abdominal pain and vomiting 30-60 min after meals. He had been told that he may have IBD, but his mother is concerned because he has lost weight. ❶: What specific questions would you ask the patient? ❷: What is the most likely diagnosis? ❸: What examination would you perform? ❹: What would be the initial management? ❺: What investigations would you request to confirm a diagnosis? ❻: What other issues should be addressed? ❼: Will I get cancer?

...

General Medicine 3 - An 18-year-old student has returned from a gap year spent in India with a short history of diarrhoea, abdominal discomfort and jaundice. ❶: What is the likely differential diagnosis? ❷: What in the given history supports the diagnosis? ❸: What additional features in the history would you seek to support a particular diagnosis? ❹: What clinical examination would you perform and why? ❺: What investigations would be most helpful and why? ❻: What treatment options are appropriate? ❼: Is there a risk that I will pass it on?

...

General Medicine 3 - An 82-year-old man with advanced Parkinson's disease and dementia is cared for in a nursing home. His nursing needs are usually fairly predictable. Of late, he has become more agitated than usual. He seems uncomfortable and has started to shout and rattle the bedsides, and occasionally he hits out at the nurses. The on-call GP has prescribed a sedative to settle the patient at night. He is continuously wet with urine and, unusually for him, is now incontinent of liquid faeces too. He has no fever and other vital signs are normal. ❶: What are the likely causes of the problem? ❷: How should the problem be investigated? ❸: What can be done to help?

...

General Medicine 3 - Patient with diabetes: Why is my blood pressure so difficult to control? ❶: Please counsel the patient.

...

General Medicine 3 - What advice do I give my partner if they witness me having a 'hypo' reaction? ❶: Please counsel the patient.

...

General Medicine 3 - What should I do with my insulin if I develop a vomiting illness that stops me eating? ❶: Please counsel the patient.

...

Neonatal - You are attending the delivery of a term infant in the labour ward. At delivery the baby has two loops of cord wrapped tightly around the neck. The baby is blue in colour and not breathing at birth. The liquor was clear at delivery. ❶ Please demonstrate how you would manage the resuscitation of the baby on the doll provided. Explain to the examiner as you go what steps you are taking. ❸ Explain to the examiner how you would determine the Apgar score for an infant 5 minutes after birth, and why the Apgar score is performed.

...

Obstetrics and Gynaecology 1 - Annabelle Gordon is a 35-year-old primigravida at 34 weeks gestation, who presents via ambulance with an antepartum haemorrhage at 0100 hours. The bleeding is fresh and at least 2 cups in volume, with ongoing bleeding continuing. She has been a regular attendee at antenatal care, with an uneventful pregnancy to date. The 20-week scan showed an anterior, not low placenta. You are attending her. Please take any further relevant history and say what examination you would conduct, as well as management, based on the history above and your findings. The examiner will play the role of the patient. ❶ Please take a brief and directed history. ❷ Please tell the examiner what physical examination findings you would like to know. ❸ Please indicate your initial management steps. ❹ What is your provisional diagnosis?

...

Obstetrics and Gynaecology 1 - Mrs Gore is a 27 -year-old woman presenting to your clinic with a monochorionic diamniotic twin pregnancy. ❶ Counsel her as to the increased risks compared with a singleton pregnancy. ❷ I have been reading in the paper over the weekend about twin- twin transfusion syndrome. How will we tell if I am developing this condition? ❸ Mrs Gore does not develop twin-twin transfusion syndrome during her pregnancy. What conditions and preparations would need to be in place for you to be happy to deliver Mrs Gore's twins vaginally, rather than by caesarean section?

...

Obstetrics and Gynaecology 1 - Mrs Ward has presented to the labour ward at 39 weeks gestation with a history of fluid loss from the vagina. Please assess and advise on management. ❶ The examiner will play the role of the patient. ❷ Mrs Ward goes home to await the onset of normal labour and agrees to our management plan. She returns the next day with a yellow-green colour to her liquor. Please advise on your management. ❸ Another woman is on the ward with ruptured membranes at 30 weeks gestation. What features would suggest chorioamnionitis?

...

Obstetrics and Gynaecology 1 - Triage the following patients: ❶ 28 yo F. Irregular periods. 130 kg. Tired. ❷ 42 yo F. G4P0. 12/40. Spotting. ❸ 52 yo F. Radical hysterectomy + nodes 10 days ago. Severe acute low abdominal pain. Hasn't voided urine for 12/24 ❹ 26 yo F. Irregular periods. RIF pain. 36.5°C, 90 bpm. 90/60 mm Hg. LMP = 6/52 ago ❺ 18 yo F. Diffuse pelvic pain. 37.6°. 90 bpm. 90/60 mm Hg.

...

Obstetrics and Gynaecology 1 - Triage the following patients: ❶ Andrea P. G2 P1. 37 weeks. Spontaneous labour. Contracting regularly, membranes intact and was 2 cm dilated on last examination three hours ago. History of ELUSCS for breech presentation for first pregnancy. Andrea would really like to have a vaginal birth her partner is concerned something will go wrong. ❷ Mary S. G1 PO. 40 weeks. Admitted an hour ago. History of contracting intermittently past 48 hrs. Patient (and her partner) have not slept and are exhausted. Now contracting every 4 minutes. Membranes intact. Fetal heart 130bpm. ❸ Jane. G2 P1. 37 weeks. Admitted in established labour ten minutes ago. The midwife has paged you to say the patient is draining meconium stained liquor and the fetus palpates as an oblique lie. ❹ Brigid S. G4P1. 28 wee~s2: Admitted 30 mins ago. Irregular contractions. Midwife informs you she has dysuria and a low grade temperature.

...

Obstetrics and Gynaecology 1 - Triage the following patients: ❶ Jane G: G1PO. Term pregnancy. Spontaneous labour. Intact membranes. Cephalic presentation. Contracting well. Fetal heart satisfactory. 4cm dilated on examination 2 hrs ago. ❷ Paula G. G2 P1. Awaiting induction (booked) for post dates. Paula is at 40 weeks plus 10 days gestation otherwise her pregnancy is uncomplicated. ❸ Sue T. G2 Pt. Term Pregnancy. Spontaneous rupture of membranes 24 hours ago. Not contracting. Observations stable. Sue is angrily demanding to speak to the doctor about her care. ❹ Karen S. G1PO. Term pregnancy. Cephalic presentation. Contracting 4-5 minutely. Membranes intact. Fetal heart tracing showing occasional decelerations of concern to the midwife. Vaginal examination 5cm (last vaginal examination 4hrs ago was also 5cm dilatation). ❺ Kate P. G1PO. 37 weeks. Mary has been an inpatient for one week for monitoring of her preeclampsia. A decision has been made to deliver her today as her BP is extremely labile. The resident from the ward has called to book her and informs you her Bishop's score is 2.

...

Obstetrics and Gynaecology 1 - You are a rural GP obstetrician called by the midwives. "Mrs Wyatt delivered her baby 15 minutes ago, and is now bleeding very heavily. Could you please come to the labour ward as soon as possible." ❶ Describe your assessment and management of the patient. ❷ What are the risk factors for a post-partum haemorrhage? (In general- not specific to this patient.) ❸ Would this episode change your management of Mrs Wyatt's labour for her next baby? If so, how?

...

Obstetrics and Gynaecology 1 - You are called to the labour ward to see Mr and Mrs Harlow, who presented in their first pregnancy at 38 weeks gestation with a 3-day history of reduced fetal movements. They have had a fetal death in utero confirmed on ultrasound, and are understandably distraught. ❶ Take a focused history and advise on appropriate investigations. ❷ Advise and counsel the couple on subsequent management.

...

Obstetrics and Gynaecology 1 - You are the casualty medical officer seeing Mrs Rachel Turner, a 24-year-old primigravida, at roughly 9 weeks gestation, in the Emergency Department. She has presented with PV bleeding. ❶ Take a history, examine and initiate investigation of the patient. ❷ What are the differential diagnoses of the cause of the PV bleeding? ❸ • Ultrasound report: 6/40 sized intra-uterine gestational sac. No fetal pole or fetal heart visible. Cervical os open • BHCG = 8000 IU/mL • Haemoglobin 130 g/dl. What is your management? ❹ Tell me what went wrong, doctor. Counsel the patient.

...

Obstetrics and Gynaecology 1 - You are the obstetrics registrar, called to see Mrs Joan Grey in the labour ward. The midwife on duty tells you over the phone: "Mrs Grey has been pushing for two hours, and is getting tired. Please come and assess her." ❶ What are the conditions that must be satisfied for you to safely perform a forceps delivery? ❷ What are the potential complications of a forceps delivery?

...

Obstetrics and Gynaecology 1 - You are the obstetrics resident, called to see Mrs Walker, a 25-year-old primigravida, in labour. ❶ Please take an appropriate history, examine the patient, and advise on your management. ❷ The midwife calls you 2 hours later to say that she has heard a deceleration during the last contraction, and that she has put on a CTG monitor. She asks you to come and review the fetal heart trace. What are the features of the trace that you would look for? What features indicate a reassuring CTG? ❸ Mrs Walker is reassessed by vaginal examination 4 hours after your initial examination, and is found to have a cervical dilatation of 5 em. Outline your assessment and management.

...

Obstetrics and Gynaecology 2 - A-49-year-old woman, Mrs Broadbent, presents to you with an 8-week history of increasing abdominal girth, urinary frequency and a palpable abdominal mass. ❶ Please take a history from the patient. ❷ Ask for relevant examination findings. ❸ What investigations would you like to order? ❹ The results of your investigations show: FBE, U+E+Cr, LFTs: normal. • CA125: 1205 U/mL ( <45 U/mL) • CA15.3: 11 U/mL ( <30 U/mL) • CA19.9: 45 U/mL ( <39 U/mL) • CEA 6.4 1-1g/L (non-smokers <3.5; smokers < 6.5 1-1g/L) • LDH 225 U/L (100-230) CXR: small pleural effusion in R lung field. • Pelvic ultrasound: Bilateral solid and cystic ovarian masses with low resistance blood flow. Small uterus, endometrial thickness 3 mm, ascites, normal kidneys • CT scan: Large solid, cystic mass arising from pelvis, ascites, omental thickening, no lymphadenopathy "Doctor, what do my results show, and what do we do now?"

...

Obstetrics and Gynaecology 2 - Mrs D, a 62-year-old woman, is referred to you with the sensation of a lump in her vagina, as well as urinary incontinence. ❶ Please take an appropriate history. ❷ Please ask for relevant examination findings. ❸ Please explain to Mrs D what investigations you would like to perform, and what management options are available to her. Outline any relevant health issues you have identified.

...

Obstetrics and Gynaecology 2 - Mrs Meadows, a 46-year-old woman, presents with heavy, irregular periods. ❶ Please take a history relevant to the presenting complaint. ❷ Please ask for relevant examination findings. ❸ What investigations would you like to order? ❹ The results of your investigations are as follows: • Pap smear: normal • Haemoglobin: 9.7 g/dl • Iron studies: consistent with Fe-deficiency anaemia • Normal thyroid, renal and liver function • Pelvic ultrasound: - Endometrium 20 mm thick - Uterus 10 em long, 4.8 em antero-posterior diameter, 6 em wide. Normal uterus, ovaries and tubes - Corpus luteum in left ovary - No free fluid in pelvis • Pipelle (if asked for): endometrial hyperplasia without atypia - (If pipelle not asked for) "What would you advise next?" • Hysteroscopy/D+C findings: endometrial hyperplasia without atypia. How would you manage this patient's problem, and what are the side effects of treatment?

...

Obstetrics and Gynaecology 2 - Ms H, a 24-year-old woman, presents to you complaining of pelvic pain. ❶ Please manage her problem. ❷ Ms H's abdominal and pelvic examination found a normal sized uterus which was fixed in a retroverted position. There was the impression of a mass on the right side of the pelvis. Ms H has a laparoscopy at which endometriosis is seen and the areas of endometriosis are removed. Please explain endometriosis to Ms H and the management of her condition.

...

Obstetrics and Gynaecology 2 - You are attached to the Gynaecology Oncology Unit and have just assisted at a laparotomy on a 54-year-old woman with advanced ovarian cancer. She has had a total abdominal hysterectomy, bilateral salpingo-oophorectomy and omentectomy. Blood loss was estimated at 300 mL and 5 L of ascitic fluid was drained at the time of the surgery. Fluid replacement was 2.5 L crystalloid and urine output was 250 mL. ❶ Please describe your post-operative orders for the first 24 hours. The patient is stable; there are no drains in situ. The patient has an indwelling urinary catheter and has patient-controlled analgesia (PCA) with morphine. ❷ It is day 3 post-op. The patient has been stable and was commenced on clear fluids yesterday. You are asked to see her as she has vomited a large volume of bile-stained fluid and is complaining of abdominal pain. Please assess the patient. ❸ What investigations would you like to order? ❹ The results of the investigations are as follows: • CXR: mild atelectasis only • AXR: air-fluid levels and distended loops of small bowel • FBE: Hb 10.6 g/dl, WCC 18.0 • Urea/electrolytes: potassium 3.0, otherwise normal What is your diagnosis and management?

...

Obstetrics and Gynaecology 2 - You are seeing Ms T, who presents with irregular menstrual periods 6 to 12 weeks apart, and excessive hair growth. Please assess her and manage her problem. ❶ Please take a history of the presenting complaint from Ms T. ❷ Please ask for relevant examination findings. ❸ Are there any tests that you would order? What are the treatment options for this woman?

...

Obstetrics and Gynaecology 3 - Miss Bertram is a 23-year-old woman who has come to see you following her routine Pap smear. The Pap smear has shown low-grade cervical dysplasia- CIN1/HPV. ❶ Please explain this to her, as well as your management plan. ❷ Is there a role for the HPV vaccine in this woman? ❸ Describe the HPV vaccines currently available. ❹ Miss Bertram returns in 12 months for her repeat Pap smear, which still shows CINl/HPV. What is the next course of action? ❺ What does a colposcopy involve? ❻ Miss Bertram has a biopsy that shows CIN3/HPV. The transformation zone is in full view, and the changes do not extend up the endocervical canal. What are the treatment options?

...

Obstetrics and Gynaecology 3 - Mr and Mrs F are unable to become pregnant after trying for 3 years. ❶ Please take a history about the presenting problem. ❷ Please ask for relevant examination findings. ❸ What are the key investigations to be performed?

...

Obstetrics and Gynaecology 3 - Mrs Brown is a 24-year-old nullipara with a 20-year history of type 1 diabetes (insulin-dependent diabetes mellitus). She wishes to have a baby. ❶ Please take a history. ❷ Please counsel Mrs Brown with respect to pregnancy planning and care based on the history you have obtained.

...

Obstetrics and Gynaecology 3 - Mrs Hassim is a 28-year-old recent migrant from Afghanistan presenting to the public antenatal clinic at 8 weeks gestation for her first antenatal visit. She is fully veiled. ❶ Please advise her on the antenatal tests required, and explain to her in lay terms what the tests are for. ❷ You are seeing her two weeks later. The results of the tests include: • Blood group: AB +ve, no antibodies • Full blood examination: Hb 120 (110-140), MCV 69 (80-100), WCC 8.0 (4-11), platelets 280 (150-450) • RPR: negative • Hepatitis B serology: negative • Rubella serology: immune • HIV serology: negative • MSU: negative • Vitamin D: 20 (75-250) Please explain the significance of any abnormal tests, and what, if anything, needs to be done about them.

...

Obstetrics and Gynaecology 3 - Mrs W, a 62-year-old woman, comes to see you for some advice regarding her family history of cancer. ❶ Please take an appropriate history from Mrs W. ❷ What advice would you give Mrs W regarding her family history? ❸ What is the likely hereditary cancer syndrome in this family? What are the other main gynaecological family cancer syndromes, and what are the genes that are mutated? ❹ What screening tests, and what prophylactic surgery, are available for: a HNPCC? b BRCA 1/2?

...

Obstetrics and Gynaecology 3 - Ms B presents to have a routine Pap smear performed. She last had a normal smear 3 years ago and her smears have always been normal. ❶ Please explain why Pap smears are performed and how often they should be done. ❷ Please perform a Pap smear on the mannequin. Describe the steps of performing the Pap smear and explain what you are doing to the examiner.

...

Obstetrics and Gynaecology 3 - Ms C, a 24-year-old woman, presents requesting a script for the combined oral contraceptive pill. ❶ Please counsel her. ❷ Please inform Ms C on how to start the COCP and the side effects. ❸ Please advise Ms C on the benefits and risks of using the COCP.

...

Obstetrics and Gynaecology 3 - Ms V is presenting for the 'morning-after pill'. She had unprotected sex last night with a long-standing male partner. ❶ Please take a history from Ms V. ❷ Please inform Ms V on the different types of emergency contraception currently available in Australia, their effectiveness and side effects. ❸ What should be advised for Ms V for the future?

...

Obstetrics and Gynaecology 3 - You are an RMO working in an outpatient clinic. Your next patient is Mary and her notes indicate she has come to discuss having a tubal ligation. Mary has no relevant surgical or medical history. Please discuss tubal ligation with Mary to enable her to make an informed decision.

...

Obstetrics and Gynaecology 3 - You are in an antenatal clinic. Mrs Joanne Thompson, a 30-year-old primigravida at 34 weeks gestation is enquiring about pain relief options in labour. "Hello, doctor. I am a bit worried about how I will cope in labour. What can I do for pain relief?" ❶ Can you tell me any advantages or disadvantages between the various options?

...

Ophthalmology - A 30-year-old woman complains of episodes of visual loss in both eyes associated with eye movements. She has a 1-month history of intermittent headache. ❶: What is the likely differential diagnosis? ❷: What in the given history supports the diagnosis? ❸: What additional features in the history would you seek to support a particular diagnosis? ❹: What clinical examination would you perform and why? ❺: What investigations would be most helpful and why? ❻: What treatment options are appropriate?

...

Ophthalmology - A 35-year-old patient presents with a 1-year history of difficulty seeing at night. The patient also complains of constriction of the visual field. ❶: What is the likely differential diagnosis? ❷: What in the given history supports the diagnosis? ❸: What additional features in the history would you seek to support a particular diagnosis? ❹: What clinical examination would you perform and why? ❺: What investigations would be most helpful and why? ❻: What treatment options are appropriate?

...

Ophthalmology - A 40-year-old patient gives a 1-week history of sudden-onset flashing lights and floaters in one eye. The patient has worn distance spectacles since the age of 12. ❶: What is the likely differential diagnosis? ❷: What in the given history supports the diagnosis? ❸: What additional features in the history would you seek to support a particular diagnosis? ❹: What clinical examination would you perform and why? ❺: What investigations would be most helpful and why? ❻: What treatment options are appropriate?

...

Paediatrics 2 - My 2-day-old baby is blue and can't breathe. A 2-day-old baby looks dusky and is having difficulty breathing. He has not been feeding well, and has been sweaty. A murmur was heard at birth. ❶: What is the likely differential diagnosis? ❷: What in the given history supports the diagnosis? ❸: What additional features in the history would you seek to support a particular diagnosis? ❹: What clinical examination would you perform and why? ❺: What investigations would be most helpful and why? ❻: What treatment options are appropriate?

...

Paediatrics 2 - My 5-month-old isn't breathing. A 5-month-old baby is found in its cot, not breathing, 1 hour after being put down to sleep. ❶: What is the likely differential diagnosis? ❷: What in the given history supports the diagnosis? ❸: What additional features in the history would you seek to support a particular diagnosis? ❹: What clinical examination would you perform and why? ❺: What investigations would be most helpful and why? ❻: What treatment options are appropriate?

...

General Practice 2 - A 36 year old woman, phones requesting a home visit. She says she is dreadfully dizzy and nauseous, and every time she tries to get up the room seems to spin. This started yesterday and is getting worse.

0

General Practice 2 - Your next patient is a 20-year-old student, who works part time as a waiter. S/he comes in with a 4-day history of a sore throat - "the worst I've ever had". S/he says s/he has a fever and a headache, and feels quite tired.

0

General Practice 2 - Your patient comes in and tells you s/he feels dreadful - headache, feeling really clogged up, and a toothache. S/he says she had a head cold about 10 days ago, which s/he thought had finally gone, but then this started 2 days ago

0

General Practice 2 - Your patient comes in with knee pain. It's been present really for about six months. It's getting worse - thought it was just part of getting old, but now worried it is gout or that really bad arthritis that cripples you

0

General Practice 2 - Your patient is a 4-year-old, whose parent brings him/her in because of earache. The child was awake and distressed for much of the night. The parent says the child has been going to swimming lessons and wonders if this could be causing the pain.

0

General Practice 2 - Your patient tells you s/he has done his/her back in again. S/he says s/he was lifting some groceries out of the car boot. When s/he turned s/he felt the pain suddenly in her/his back, and found it difficult to straighten up. You know this patient has a history of chronic lower back pain.

0

Obstetrics and Gynaecology 2 - Take a history from a woman with genuine stress incontinence

0

Obstetrics and Gynaecology 4 - A patient tells you she is hoping to become pregnant soon. What history would you want to know and what advice would you give her?

0

Obstetrics and Gynaecology 4 - Describe what is routinely done at a "booking" antenatal clinic in terms of history, examination and investigations.

0

Ophthalmology - Fundoscopy

0

Ophthalmology - Pupillary reflexes

0

Ophthalmology - Slides of eye disorders

0

Ophthalmology - Slit lamp examination

0

Ophthalmology - Visual acuity

0

Paediatrics 1 - Aaron is a 18 month old boy from Karratha. He has a 4 week history of fever (resolved), irritability, decreased appetite, poor sleep and sore feet. Amoxycillin has not led to any improvement. He has back and side pain and the abdominal examination is very difficult due to pain.

0

Paediatrics 1 - Alexandra is a 5 yr old girl with a 2 week history of pallor, lethargy, with headache, bone pain and fever unresolved with antibiotics.

0

Paediatrics 1 - Amy is a 11-month-old girl with urticaria and angioedema after eating a piece of cake

0

Paediatrics 1 - Anna is a 6 months old girl with an URTI and increasing respiratory distress

0

Paediatrics 1 - Jake is a 7 -year-old boy who presents following a generalised seizure

0

Paediatrics 1 - Jemma is a well 5-year-old. She has multiple bruises, mainly over her legs and forearms, and you can see numerous petechial lesions

0

Paediatrics 1 - Jodi is a 2-week-old infant with pustular rash on the face and trunk

0

Paediatrics 1 - Julie: a 4-month-old girl with bad nappy rash. The creams from the pharmacy do not seem to be helping.

0

Paediatrics 1 - Keegan a 6-year -old boy with a unilateral red eye

0

Paediatrics 1 - Lucy is a three-year-old girl with fever, weight loss and a large R.sided abdominal mass.

0

Paediatrics 1 - Melody is a 14 year old girl who has been absent from school for a term because of recurrent aches and pains

0

Paediatrics 1 - Mike a 5-year-old boy with speech delay, and poor hearing

0

Paediatrics 1 - Nerrisa is a newborn infant with a large sponge red "birth mark" near his L eye

0

Paediatrics 1 - Sarah a 3-year-old girl, found by the clinic nurse to have a visual acuity of 6/24 right eye and 6/6 left eye

0

Paediatrics 2 - Paediatric cardiac arrest

ABC [1]Check response, call for help [0.5]Airway opening manoeuvres, chin lift, jaw thrust, do not sweep[1]Look, listen, feel[0.5]2 rescue breaths, note gentle rise + fall[1]Pulse check <10 seconds (not essential) brachial/femoral[0]Chest compressions: hand position-lower half of sternum, if 2 thumbs hand size adequate[1]1/3 of chest wall[1]Rate close to 100/min[1]Ventilation technique: note and achieve good rise and fall[2]High flow O2 used or requested[1]Ratio: 15:2 (accept 30:2) approx 2 cycles/min[1]Continue for 2 min-may check after 1 minute[1]IV normal saline bolus given 10-20 mL/kg[1]Adrenaline IV given, correct dose of 10 mcg/kg/look up dose[1]

General Practice 1 - Country GP. Geoff is a 39 yo bank manager. He has been seen on a number of occasions in the practice over the last five years. His BP has been measured and recorded. The best reading was 140/98 and today it is 160/105. You note his BP is taken using the right arm and the correct sized cuff is always used with the patient sitting. When his blood pressure was taken supine last visit it was 155/100 at 5 and 10 minutes. Outline your Mx plan for this patient from the General Practice viewpoint.

Able to explain to Geoff that he has moderate hypertension and it is probably time to treat him with an antihypertensive medication. [3]Management: lifestyle modification eg low salt diet, weight loss, exercise, controlling stress[3]Ix: discussed Eg: fasting BSL, fasting lipids,UEC, MSU, ECG, CXR etc [3]Able to discuss phamacotherapy eg thiazide, ACE, beta blocker, calcium channel antagonist etc[3]Empathy score: 2

General Practice 3 - You are a GP in a country town. Anne, a 20 year old woman, has come to see you wishing to commence the oral combined contraceptive pill. A nurse has taken her history. Anne has no relevant medical or family history to contraindicate commencing this contraception. Anne's BP is 120/70 and her weight is 64 kgs. Using this sample of the contraceptive pill explain to Anne: 1. the action of the OCP. 2. the common side-effects of this contraception. 3. counsel Anne how to use the oral contraceptive pill most effectively. What happens if I forget a pill? Will alcohol or anything else stop it working?

Action: inhibit ovulation, cervical mucous changes, decreased receptivity endometrium [3] Common side effects (must name at least 6 for full marks) breakthrough bleeding, fluid retention, weight gain, breast enlargement/tenderness, nausea, acne, depression, loss of libido.[2]Instructions: Take as directed. Can take if missed within 12 hrs. 7 day rule: need contraception for the first 7 days or if a pill missed over 24 hrs. If you miss a pill and there is less than 7 active pills in the pack miss the pill-free period and go onto a new pack. Reduced efficacy if gastro upset / on antibiotics / other medications. COC for contraception only / need for safe sex practices. Advise to link taking COC with a daily habit. Makes an effort to ensure pt understands ie. invites questions/asks her to repeat information [7]Empathy score: 2

Paediatrics 3 - Remote Aboriginal community. 18 yo F with 13 month old Jacinta. Discharged from distant regional hospital. Gastroenteritis + failure to thrive. Brief Hx, respond to mother's concerns, explain FTT.

Ask mother questions that elicit how she feels [2]Ask about feeding - milk, other liquids, solids and frequency of solids [2] Ask about living circumstances - who she lives with, is she with her mothers family, how many people in the house etc [1]Acknowledge the mother's anger and accept her concerns about the hospital, without alienating the mother and making her feel guilty! [2]Explain growth pattern and FTT, refer to charts, overcrowding, how breast not sufficient over 6/12, solid is needed. Avoid judgement. [3]Avoid soft drink, reduces appetite, harder to put on weight, tooth decay. [2]Empathy score: 2

General Practice 1 - Colle's fracture - gross anatomical appearance, X-ray features.

Clinical: Dinner fork deformity, soft tissue swelling, blood, ? Open[2]X-ray: #distal radius with 50% dorsal displacement, dorsal angulation, shortening, associated #distal ulna.[2]Dx: Colles'[1]Further assessment: Distal neurovascular function e.g. median nerve, check if open[1]Mx: analgesia, splint, reduction with regional/general anaesthesia, back slab[2]Position: wrist slightly flexed, ulnar deviation[2]Complications: tendon rupture, median nerve injury, vascular compromise, stiffness/OA, malunion, carpal tunnel syndrome[2]Predisposing: osteoporosis Dx with bone densitometry[2]

General Practice 1 - Your next patient comes to see you about her/his asthma. It has been worse over the past few months, and s/he is needing to use a puffer a couple of times a day.

Communicate clearly, avoid jargon[0.5]Check patient's understanding of information and instructions[0.5]Choose preventer+reliever, ICS may be combined with LABA[1]Watch technique, recognise if poor.[1]Advise use of spacer[1]Demonstrate good MDI technique: Shake[0.5]Hold correctly[0.5]Exhale[0.5]Inhale and hold[0.5]Arange appropriate follow up (1-3/52)[1]Safety net including warning signs of severe asthma and action to take[1]Advice on care of delivery device[1]Appropriate dose[1]Possible side effects, wrinsing mouth to avoid thrush[1]Written action plan[1]Empathy score: 2

Obstetrics and Gynaecology 1 - You are a resident on the labour ward. Please answer the phone when it rings. Ask some questions about the situation and give the midwife some advice. 32yoF delivery 35 minutes ago, heavy bleeding. G3P2, 2nd was emergency CS... placenta not yet delivered, 2nd degree tear.

DDx: Retained placenta[2]Atonic uterus[1]Vaginal tear[2]Ruptured uterus[1]Hx: risk factors for PPH - Past CS, prolonged labour, 2nd degree tear[2]Patient assessment: vital signs, blood loss[2]Mx: ABCs, IV access, fluids, X-match, empty bladder, uterine massage, oxytocin infusion, gentle traction to deliver placenta. Pad, pressure and repair for vaginal tear[4]

General Medicine 2 - 55 yo M. Noticing some SOB walking up stairs. Used to smoke 30/day from age 15, quit 5 yo post pneumonia. FHx not surprising. Worked on wharfs. No other medical intervention.

Describe in systematic fashion. Heart size, expansion (posterior ribs), masses[2]Ix: ECG, ABG, PFTs, Stress tests, FBC, UEC[2]ECG interpret: Sinus, normal axis, RBBB, systematic[2]PFTs: obstructive[1]ABG interpretation: compensated hypoxia, not a candidate for home O2[2]DDx: IHD, COPD, mild CCF, exercise related arrhythmia[2]Mx (COPD): Inhalers, pulmonary rehab, vaccination, review and monitor[2]Mx (IHD): Aspirin, cardiology input[1]

Paediatrics 0 - You are helping at a local GP clinic. Mrs Rhonda James brings her 12 week old baby Thomas to see you. Take a short history, explain how you would examine the baby to the examiner then explain your Mx to Mrs James. Mother is 25, G1P1, labour 16/24, SVD, continuously vomits e.g. milk, irritable, not projectile, ?pyloric stenosis.

Enquire about general health[1]Vomiting: f, quant, colour, timing, projectile...[1.5]Complications: cough/weight gain[1]Ask about emotions[1]DDx: reflux, colic, pyloric stenosis[1.5]Reassure that reflux most likely[0.5]Exam: Hydration[0.5]Request test feed to feel pylorus[0.5]Check child health record[0.5]Reassure about normal growth pattern[0.5]Mx: more upright[0.5]No activity after feeding[0.5]Thickening agent[0.5]Mention use of ranitidine/PPI at some point[0.5]Request urinalysis to exclude UTI[0.5]Need to return to check response to treatment/weight[1]Empathy score: 2

General Practice 1 - Mabel is 50 years old with chronic headaches. Take history. Had headaches for years, treated them with aspirin/codeine. Recently headaches became 1 sided. Smoker. BP 170/100. DDx.

Establish rapport[1]Organized approach to history taking[2]Note change in Hx... red flag[1]DDx: Tension headache[1]Migraine headache[1]Sinus headache/rebound headache analgesia[1]Cervical spondylosis[1]Contemplate possibility of haemorrhage/SOL[1]Follow up need for CT[1]BP being elevated[1]Other CVS risk factors[1]Empathy score: 2

Oncology - GP:Wife of 67 year old recently d/c from RPH with lung cancer. Radiotherapy but poor prognosis. A few medications. Never met before. Continue consult, formulate simple management plan.

Establish rapport[2]Allow patient to express concerns[3]Assess understanding and feelings to home terminal care[2]Make specific arrangement to see the man at home or here[1]Elicit current symptoms/problems[2]Contact silver chain[1]Review meds[1]Empathy score: 2

Paediatrics 2 - Isaac 7/12 old. Worried about vomiting overnight, felt hot. Medical student. Nurse asking for opinion because no doctor. 2-3 hours to nearest ED. Brief Hx, interpret exam, outline Mx.

Establish temporal Hx/brief duration[1]Recognise fever on Hx[1]Hx of localising signs-ENT, chest/abdomen/urine[1]Try to quantify fluid/weight loss[1]Background Hx, PMHx, Contacts, Vaccines[1]Identify abnormal/worrying signs - specifically perfusion/BP[1]Ix/Mx: recognise septic baby[2]Septic screen: BC, CXR, urine +/- LP, IV/IO access[1]O2, 0.9% saline bolus 20 ml/kg[1]Empirical AB: ceftriaxone if available[1]Plan transfer to larger centre[1]Empathy score: 2

Paediatrics 2 - You are an RMO who has been asked to speak to a 12 year old and his family after recently being hospitalised for an anaphylactic reaction. He is doing well and is now being discharged. Please discuss with an appropriate discharge plan and any warning signs of anaphylaxis. Please answer any questions the parent(s) may have.

Explains anaphylaxis (no jargon) • Multisystem allergic reaction involving lungs and/or heart and other organs (skin or GI tract) [1]• Life threatening medical emergency [1]States what can cause anaphylaxis • Foods [0.5]• Insects [0.5]• Medications [0.5]Explains Sx of anaphylaxis (severe) at least 4 • Difficulty/noisy breathing (hoarseness or stridor) • Swelling of tongue or throat • Difficultly talking and/or hoarse voice • Wheeze or persistent cough • LOC &/or collapse • Pale and floppy (young kids) [4]Symptoms that can preceded anaphylaxis - at least 2 • Swelling of face, lips and eyes • Hives (urticaria) or welts • Stomach pain, vomiting [1]Management: • Referral to specialist (immunologist/allergist) [0.5]• Identifies the trigger/s of anaphylaxis (blood tests for allergen specific IgE - RAST tests or skin prick tests) [0.5]• Education on avoidance of triggers (esp. food & involve allergy dietian) [1]• Provides anaphylaxis action plan [1]• Involvement of school and/or daycare [0.5]• Instruction on use of EpiPen and emphasises its importance & to always keep on person [2]• Medical alert bracelet

Paediatrics 3 - 6 yo F. Vomiting 3-4 days. Take Hx, Dx, explain, Ix, Mx

General state of health[1]Vomiting: f, quant, colour, timing, projectile...[1]Eating and drinking[1]Diarrhoea?[1]Infectious contacts/recent illness?[1]Elicit nocturnal polyuria[1]Hx of weight loss[1]Ketone breath[1]Working Dx of DM1[1]Explain Dx using understandable and correct terms[1]Explain ketosis, realise urgency[1]Specialist referral[1]Empathy score: 2

Paediatrics 2 - You are an RMO working in PMH ED when parent brings in their child stating that they're sick and are making funny breathing noises. Please take a brief relevant Hx. After watching the video state what signs you see and formulate a DDx. Then outline your initial Mx plan. 6/12 girl started being unwell 3 nights ago and typically gets worse at night, refused to breast feed. Began slowly, don't think she ingested/has choked on anything. No drooling. Persistent cough, feverish, refuses to feed, can't settle. Haven't changed her nappy in about 12 hours, no diarrhoea/bowel motions. Runny nose later on she started making a funny noise. Never made it before. Immunised, no contact with sick people/overseas, taken care of at home. 3.5kg birth weight, SVD@term, no significant PMHx, NKA. Developmentally appropriate. No one smokes at home

History (onset, duration, background history, cough [1]Vaccination, contact with ill persons, day care attendance, overseas travel [1]Birth weight, term baby, complications, past medical history [0.5]Feeding (eating and drinking), defecating, urinating, wet nappies/heavy? (weight loss/hydration) [0.5]Crying, unsettled, irritable, lethargic [0.5]Video: • Inspiratory stridor [0.5]• Tracheal tug [0.5]• Subcostal recession & sterna recession [0.5]• Increased work of breathing [0.5]Recognises the child is in respiratory distress / upper airway obstruction [1]Ddx: Croup (viral laryngotracheitis), bacterial tracheitis (rare), foreign body. Epiglottitis (rare and vaccinated), angioedema (allergic rxn). [1]Management of acute upper airway obstruction • Admit to hospital [0.5]• Oxygen with adrenaline nebuliser [1]• Single dose steroids, either IM dexamethasone or oral prednisolone [1]• Contact ICU for potential transfer, monitoring and intubation [1]• Minimise disturbing the child - DO NOT: • inspect oropharynx • Send patient for radiology • Insert IV • Take blood gases [1]Empathy score: 2

Paediatrics 2 - You are a 6th year medical student 'working' in a rural hospital emergency department after hours when a distraught parent brings in their 8 month old child. Began vomiting last night, has not stopped since, is not feeding, very irritable and hard to settle, crying constantly. T = 39oC, HR = 190 (110-160), RR = 40 (20-30), BP = 55 systolic (70-90), CRT = <3 sec.

History: • Establishes history and duration and other Sx [1]• Recognises fever on history [1]• History of localising signs, i.e. cough (chest), diarrhoea (abdo) etc [1]• Tries to quantify fluid loss on history [1]• Background Hx, PMHx, contacts, vaccines, sick contacts [1]• Interprets worrying vital signs (temp and BP particularly) [1]• Recognises septic baby [1]Investigation / Management [1]• Septic screen (BC, CXR, urinalysis & MC/S, +/- LP, CRP, FBC, glucose, U&E), [1]• (urine: suprapubic aspirate, catheter, clean catch bag) [1]• establish IO/IV access [1]• Oxygen; 0.9% saline bolus (20 mL/kg) + maintain fluid balance [1]• Empirical antibiotics (ceftriaxone) [1]• Transfer to larger hospital for backup and support [1]

General Practice 1 - Leon 19 yo M mother insists poorly controlled asthma, Lots of ventolin, used to be on flixotide but lost it.

Hx: Age of onset[1]FHx[1]Triggers: exercise, cold air, hayfever, dust, smoke[3]Previous hospital admissions[1]Compliance with current meds[1]Attitude towards condition[1]Able to get Leon to demonstrate how he uses his accuhaler[2]Student can demonstrate correct use of Accuhaler[2]Empathy score: 2

General Practice 1 - 56 yo F. Fatigue over 2-3/12. Aunt had thyroid problem. Weight gain. Ix, Meds, Explain.

Hx: HxPC, PMHx, Med, Allergy, FHx, SHx, smoking/etoh, systems review[4]Screen for DM, Anaemia, Depression, Hypothyroid[2]Goitre, skin, voice, cold[1]Ix: BSL, TSH, FBC/ferritin/Fe, UEC[2]Explain/reassure[1]Commence oral thyroxine. Suggested dose 50-100 mcg daily[1]Recheck TFTs in 6-8 weeks and check thyroid antibodies[1]Empathy score: 2

Oncology - 60F. Has mammogram. Further Ix? Take Hx, outline Mx.

Hx: Lump, pain, discharge? Own breast examination? Regular mammogram/pap smears? Any Hx breast lumps/cancer? other cancers? Age at menarche/menopause? HRT/OCP? Ever smoked? How many children, did she breast feed? Family history?[8]Further steps: This may or may not be Ca so...[1]Ultrasound[1]FNA/Core biopsy (today!)[1]Refer to specialists[1]Empathy score: 2

Obstetrics and Gynaecology 1 - Outline the Management of Gestational trophoblastic disease

Hydatiform mole: Complete (CHM): 1/2 sperm in 'empty' ovum. Diploid (XX). Partial (PHM): 2 sperm 1 ovum: Triploid (69 XXX/XXY/XYY). Both can be "invasive". 3% of CHM become choriocarcinoma. 50% of choriocarcinomas follow HM, 40% normal pregnancy, 5% miscarriage/ectopic, 5% non-gestational. Placental site trophoblastic tumor - ultra-rare, high grade. ❶ Hx/Exam: Abnormal vaginal bleeding (CHM: watery brown ± Grapelike clusters of tissue), hyperemesis gravidum (CHM), hyperthyroidism (CHM), hypertension/early pre-eclampsia. Excessive uterine size (CHM) or small for gestation (PHM). Haemoptysis (chorioCa) ❷ Blood tests: β hCG level, group and antibody screen, FBC, coagulation studies, UEC, TFTs, LFTs ❸ Imaging: Abdominal USS: "Snowstorm" (CHM), fetal parts (only PHM). CXR. Choriocarcinoma:Staging CT, LP for β hCG if lung mets. ❹ Molar pregnancy: Surgical intervention with suction dilation and curettage ± US guidance. PSTT: Hysterectomy ❺ Send products of conception for histology. ☎ Notify oncology team if GTD confirmed. ❻ Give RhD immunoglobulin to RhD ⊖ women post-surgery. ❼ Weekly serum β hCG levels until normal. If it normalises within 8/52 no further Ix needed (CHM). Avoid OCP/HRT/pregnacy during surveillance period. ❽ Indications for chemotherapy: • Remote metastases. • Histologic evidence of choriocarcinoma. • ↑β hCG after 6/12. >20,000 IU/L after 4/52, plateau in 3 consecutive samples or rising in 2. ❾ Chemotherapy for GTD: Low risk: MTX + folinic acid. Medium risk: drugs in sequence, MTX, etoposide. High risk: intensive weekly schedule of EMA:EtoposideMTXdActinomycin. Alternating with "CO":Cyclophosphamide and vincristine ❿ With any subsequent ⊕ pregnancy need early USS to confirm, send POC/placenta to histopathology and check β hCG 6/52 after resolution. Choriocarcinoma needs lifetime serum β hCG levels.

Obstetrics and Gynaecology 1 - 20 year old, 11 weeks pregnant. Bright red blood. Cramping pain. DDx? Risks? Mx?

Miscarriage/spont abortion, threatened or inevitable[2]Implantation bleed[1]Ectopic[2]Molar pregnancy[0.5]Cervical bleeding[0.5]Safety net if blood loss becomes heavy[1]Rh issues[1]Abdominal exam[1]PV/spec exam[1]Referral for US[1]Gynae referral, may need operative[1]Empathy score: 2

Neonatal - Perform a "day 1 check" on this newborn baby boy.

Observe infant and comment on colour (jaundice/pallor/cyanosis/plethora)[1]Discuss weight, length, head circumference, importance of plotting on percentile chart[1]Check anterior fontanelle[1]Observe face - note symmetrical/position of ears/shape of eyes[1]Red reflex to exclude cataracts[1]Look inside mouth for cleft of hard/soft palate[1]Auscultate heart, comment on heart sounds/murmurs[1]Observe chest/check for presence/absence of respiratory distress[1]Palpate abdomen for organomegaly (liver/spleen/kidneys) and presence of hernias. Umbilicus for infection.[1]Genitalia (nomal male/female)[1]Femoral pulses[1]Examine for developmental dysplasia of hips[1]Check tone of infant including head lag[1]Turn over - check spine, check anus is patent[1]

Neonatal - Perform a "day 1 check" on this newborn baby girl.

Observe infant and comment on colour (jaundice/pallor/cyanosis/plethora)[1]Discuss weight, length, head circumference, importance of plotting on percentile chart[1]Check anterior fontanelle[1]Observe face - note symmetrical/position of ears/shape of eyes[1]Red reflex to exclude cataracts[1]Look inside mouth for cleft of hard/soft palate[1]Auscultate heart, comment on heart sounds/murmurs[1]Observe chest/check for presence/absence of respiratory distress[1]Palpate abdomen for organomegaly (liver/spleen/kidneys) and presence of hernias. Umbilicus for infection.[1]Genitalia (nomal male/female)[1]Femoral pulses[1]Examine for developmental dysplasia of hips[1]Check tone of infant including head lag[1]Turn over - check spine, check anus is patent[1]

Obstetrics and Gynaecology 1 - 32 weeks at antenatal visit. G1P0. Outline process of physical exam (abdominal palpation). When should she come in to L&D as EDD approaches?

Palpation: Fundal height for growth, lie for longitudinal, presentation, liquor volume, (also foetal back for position, descent, movement)[6]When to come in: ROM, regular contractions ~every 5 min, anything worrying, constant abdominal pain, any PV blood, change in movement, constant headache/visual changes.[6]Empathy score: 2

Paediatrics 1 - Your next patient is a three year old Aboriginal child, Tyson. He has a problem with his ear and has been brought in by his grandmother. Tyson's Grandma has taken over his care because his mother has moved into town and is 'on the grog' and they share a house with an aunt an uncle and six other children (two are brothers the rest are cousins). His ear has been discharging for some time now , probably a couple of months but the family have been travelling because of funerals so this is the first time the child has come to the doctor. Tyson was born at term and has missed some immunisations. Big brother also had a similar problem about a year ago, which eventually cleared up. Grandma has noticed she has to shout for Tyson to hear sometimes, and he doesn't seem to talk much. He has had four hospital admissions in the past , three times for gastroenteritis and once for pneumonia. The health worker on triage noted that he was playing with toys and did not seem to respond when she spoke to him. NB: The student is not required to take a history and will be shown a photo of a discharging ear. Possible and most likely Dx? What other examination would be relevant? How might you examine the discharging ear more effectively? Ix? Mx plan for ear discharge?

Possible Dx: Chronic suppurative otitis media (most probable) [1]Acute OM with perforation [0.5]Foreign Body [0.5]Otitis externa [0.5]Cholesteatoma [0.5]Further examination: ENT exam (both ears, throat, teeth, nose, cervical LN) [2]Systemic (chest, heart and lungs) pulse, RR, skin, temp[2]Hearing [1]Observation of child's general appearance and demeanour [0.5]Examination of drum: ear wash out with 1:20 solution of dilute Betadine + mop dry with tissue spears[1] Ix: Audiology [1]FBP/finger prick haemacue [1]Mx: dry mopping [0.5]bd ear wash outs as above [1]Instillation of antibiotic ear drops 2 bd, with weekly review of ear until dry for 2/52 [1]

"Paediatrics 3 - Simon is a 4 year old boy who has presented with an 8 hour history of being unwell with 3 non-bilious vomits and 2 loose stools. You feel that he looks reasonably well, that he is not dehydrated and that you can "wait and see" how he goes. What advice would you give to his parent regarding management for the next 24 hours. Parent: I gave him a drink but he vomited it all back. What is it (gastrolyte / ORF)? My mother says I shouldn't give him any food or milk. I'm worried that he hasn't eaten since last night. What should I worry about? Could this be anything else? What should I look out for? Examiner: What do you think this illness could be? What is the main concern? What would be features of dehydration? What other causes could there be for this illness?"

Probably gastroenteritis-the main problem is dehydration [1] Many other causes [0.5] Recommend H2O/ORF, rationale [1.5] Frequent small drinks - better absorbtion/less likely to vomit [0.5] Avoid hypertonic fluids (sweet/juices) [1] Alarming: poor intake but copious vomiting/diarrhoea [0.5] Pallor/lethargic/drowsy/sunken [1.5] Poor urine output [0.5] High fever/headache/urinary Sx [1.5] bile/abdo pain/other [1.5] He has the energy reserves to carry him through a period of fasting [0.5] Early introduction of diet promotes recovery [0.5] Antiemetics cause dystonic reactions in children. [0.5] Careful handwashing required to avoid cross-infection. [0.5]Empathy score: 2

General Practice 3 - Rural GP. 18F. Uni student. Asking about contraception going to visit BF in Kalgoorlie. Explain + manage

Put patient at ease, develop rapport.[1]Age at menarche, LMP, frequency, variation, duration, quantity, dys-, PMS[2]Mention full range available: OCP, minipill, implanon, depot, IUCD, diaphragms, condoms, withdrawal/rhythm/billings.[4]Frequency of sex, consequences of pregnancy[1]Contraindications to combined OCP - smoker, focal migraine, hypertension, liver disease, cancer, personal/FHx clotting.[2]Need for follow up/pap smear. Weight, BP, STI screening?[2]Empathy score: 2

Obstetrics and Gynaecology 1 - Specialist GP. Called back urgently by midwife to see Emma. 30 minutes ago was in 2nd stage of labour with head in view. 20 minutes ago delivered healthy baby boy. Emma feeling dizzy. Conscious but incoherent. 120 bpm, 80/50, 28 resps.

Recognise that this is an emergency (hypovolaemic shock)[1]Call for help[1]ABC: O2[1]Elevate legs[1]Large bore IV[1]Order appropriate IV fluids[1]Essential information: Was placenta delivered/complete?[0.5]Get estimate of blood loss[0.5]Ask if syntocin/ergotamine given[0.5]Specific Mx of PPH: massage uterus[1]Request IDC[0.5]Blood for crossmatch and FBC[1]Misoprostol/syntocin infusion[1]Further measures:Bimanual compression, prostaglandin F2 alpha, balloon tamponade, laparotomy... BLSut/IIA ligation[1]Empathy score: 2

Paediatrics 2 - You are an RMO in a park on your day off when you notice a collapsed teenager. She has a bag of opened food beside her and crumbs around her mouth. On examination: Swollen lips, eyes, face and tongue. Marked inspiratory stridor. Medical alert bracelet on her wrist: "ALLERGIC TO PEANUTS". She has 2 EpiPen Jr's in her hand. Please appropriately manage this patient. The examiner will provide you with details about the patient's condition as your treatment progresses and ask you questions.

Recognises signs of anaphylaxis [1]Evaluates ABC ( danger, response, dials 000, opens airway, looks for signs of life, stays with patient) [3]Administers epipen correctly: Pulls off grey safety cap [1]Places black end against anterior-lateral aspect of thigh [1]Pushes down hard until click is heard AND holds in place for 10 seconds [1]Removes pen and massages site for 10 seconds [1]Correctly positions patient: Patient is unconscious - places in recovery position [1]Q: if this patient was conscious what position would you put them in? A: lay flat and elevate legs [1]Correctly gives adrenaline after 5 minutes with no response [2]Q: would you give an EpiPen if you were NOT sure this was anaphylaxis? A: Yes [1]If you did not have access to an EpiPen and were in hospital what dose of adrenaline would you give and by what route? A: 1:1000 and IM or they would check or wait for senior help to arrive [1][ ]

General Practice 1 - You are seeing Bianca a 20 yr old university student. She complains of fatigue and lassitude for the last six months 1. Take a short history from Bianca. You may ask the examiner for the examination findings when you have completed this. 2. The examiner will then ask you to discuss the likely cause of her symptoms and any Investigations you might like to order.

Relationships/occupation/social[1]Diet + exercise[1]Smoking/EtOH[1]Allergies/medication[1]Menstrual[1]Depression? Mood/sleep/appetite[1]Hypothyroid: weight gain, cold intolerance[1]Red flag symptoms: weight loss, night sweats[1]Likely Dx: Fe deficient anaemia.[1]Psychosocial/depression (common)[1]Ix: FBC, Fe studies, TFTs, UEC, BSL, LFTs[2]Empathy score: 2

Obstetrics and Gynaecology 2 - Cheryl is a 51 yo F who has come to see you because her periods have become irregular. Take a Hx, explain the Dx and discuss Mx options.

Symptoms of menopause - hot flushes, mood, libidio, poor sleep, vaginal dryness, dyspareunia[2]Contraindications to HRT, e.g. BrCa, Liver disease, DVT/PE[1]DDx: Menopause[1]Depression[1]Thyroid[1]Discuss blood tests: Oestrogen, FSH, LH, TFTs[1]Encourage health screen: CVS risk factors, mammogram, bone density, pap smear[1]HRT: Risks, benefits, SE, mode of delivery, Oestrogen only vs combined[3]Address bullshit "natural" issue:[1]Empathy score: 2

Oncology - Your next patient is Jim: 30 years. You ordered US for irregular right testicular lump. Advise Jim of result and answer questions. Report: A heterogeneous mass 3 cm in diameter... Internal echo texture shows cystic and solid components... epididymis and surrounding soft tissue appear normal. Lesion has vascularity... consistent with possible malignancy in absence of trauma. Questions: What is on US? If cancer what tests do I need? Would I need surgery/other?

Tell them in direct/honest manner, advise further Ix needed in tertiary centre: CT, bloods, surgery[3]Advise possible treatments depending on tumour type - e.g. surgery, chemo, DXRT[1]End with closure and hope for patient.[2]Advise further tests will be needed: FBP, alpha fetoprotein, HCG, LDH, CT, CXR[2]Refer to consultant surgeon[1]Advise of adjunct treatment: chemo, DXRT[2]Prognosis good for seminoma (highly radiosensitive) less for nonseminoma[1]Empathy score: 2

Oncology - Robert has incurable cancer. Please answer his questions about anti-nausea medication. Can you tell me what they all are? When should I start taking them? What side effects? Do I have to take them if I'm feeling ok?

Used to prevent nausea in people having chemo[1]Exactly as directed[1]Ondansetron 1 capsule bd before eating[0.5]Dexamethasone: 1 tablet bd[0.5]With food[0.5]take 2nd earlier e.g. 4pm[0.5]Metoclopramide 1-2 tablets as needed[0.5]if you feel you need extra help'[0.5]Start taking ondansetron + dexamethasone morn after chemo[1]You will be given meds at hospital so don't need to take any on day 1[1]Ondansetron SE: constipation, headache, dry mouth[2]Dexamethasone (short term): heartburn, increased appetite, blood sugar, insomnia [2]Metoclopramide: restlessness, dystonia [2]Take regularly to stay well, don't need maxalon if feeling ok[1]

Oncology - Outline the Management of Breast cancer

❶ "Triple assessment" (confirms in 99%): clinical exam (breast, any lumps, lymph nodes), imaging (bilateral mammogram ± US) and pathology: core biopsy preferred over FNA ❷ Ultrasound axilla, sentinal node biopsy ± abnormal nodes ❸ Wide local incision or mastectomy for 1° tumour ❹ Radiotherapy to prevent local recurrence ❺ Consider adjuvant chemotherapy for most patients ❻ Endocrine therapy if ER⊕ (aromatase inh if post-menopausal, tamoxifen + LHRH agonist if pre-menopausal)/HER2⊕ (trastuzumab) ❼ If bone mets: bisphosphonates and radiotherapy ❽ Regular follow up

Obstetrics and Gynaecology 2 - Outline the Management of Vaginal cancer

❶ 1° vaginal cancer is rare, most are metastases of other gynaecological cancers. 80% of 1° are squamous cell, 10% adeno-. ❷ Vaginal and speculum examination can be used to detect. MRI is the investigation of choice to evaluate local spread. ❸ ☎ Refer to specialist gynaecological oncology unit. ❹ Radical radiotherapy with XBRT pelvis and utero-vaginal brachytherapy is the treatment of choice. It may be possible to transposition the ovaries outside of the field. ❺ In early stages (I/II) Hysterectomy with partial/total vaginectomy ± groin ± pelvic lymph node dissection may be effective. ❻ Surgery for locally advanced/recurrent disease is pelvic exenteration.

General Practice 3 - Examine this patient with an acutely swollen scrotum

❶ 28% of torsions present with abdominal pain only, so examine the genitals of any boy presenting with this. ❷ Look for scrotal bruising, redness and swelling. Examine the abdomen to exclude intra-abdominal causes of scrotal pain. ❸ Try to palpate the cord structures, the body of the testis and the epididymis separately and note where the pain localizes. If the cord is short and thickened, and the testis retracted and swollen, then torsion is most likely. This requires immediate surgical referral. ❹ If the testis is swollen, the cord is not shortened and the boy had facial swelling 10 days earlier, mumps orchitis is likely. ❺ If the epididymis is palpable separately from the body of the testis and is tender and swollen, then epididymitis is possible, although it is rare in adolescents. In young men assume epididymitis is due to an STI, treat and refer to a specialist clinic. In the over-fifties and when urinalysis is abnormal, investigate and treat as urinary tract infection. ❻ If there is an isolated tender swelling at the top of the testis, assume that it is probably torsion of a hydatid of Morgagni. ❼ If in doubt, organize an urgent ultrasound scan. If ultrasound is unavailable, then refer for immediate scrotal exploration to avoid infarction of the testis. ❽ Always explain the need for a possible orchidectomy if the testis is necrotic and for fixation of the contralateral testis prior to surgery, and obtain informed consent for this. ❾ Carry out urinalysis.

Obstetrics and Gynaecology 2 - Outline the Management of Vulval cancer

❶ 90% of 1° cancers are squamous cell (5% melanoma, treat as). VIN 1/2/3 correlate with CIN but frequently recur. Monitor suspicious lesions. Imiquimod may be of use. ❷ Hx: Lesion/symptoms, any treatments, cormorbidity, FHx, performance status. Exam: General health, site/lesion, inguinal lymph nodes, anaesthetic workup. Exam may need to be later UA if painful. ❸ Ix: FBC, UEC, LFTs, CXR. Staging CT: Chest/abdomen/pelvis ± MRI. Biopsy, excising the lesion if possible and send to histopathology. ❹ ☎ Refer to specialist gynaecological oncology unit. ❺ Mainstay of treatment is Surgery: Wide (1 cm) local incision of 1° lesion. ❻ If invasion > 1 mm (stage ≥ Ib) groin lymphadenectomy is vital. Can be ipsilateral but must be bilateral if central 1° or ⊕LN. Use separate incisions to 1°. Sentinal node idea is experimental. ❼ Radiotherapy may be used pre-surgery to shrink tumour and ↓ morbidity if major structures involved. Can use post-surgery to prevent ↻ if excised groin LN are ⊕. Also given if pelvic LN potentially ⊕. ⚠ Not a substitute for groin dissection. ❽ Can add chemotherapy to enhance radiotherapy: 5-FU and mitomycin C.

General Practice 2 - Examine this patient with sudden-onset headache

❶ Assess and record level of consciousness (intracranial lesions). ❷ Measure pulse, blood pressure and temperature. ❸ Look for signs of meningeal irritation: photophobia, neck stiffness and Kernig's sign (bacterial or viral meningitis, subarachnoid haemorrhage). ❹ Examine the optic fundi for papilloedema (a late sign of raised intracranial pressure; absence of papilloedema does not exclude raised intracranial pressure). ❺ Examine for focal neurological signs (cerebral haemorrhage, other structural lesions). ❻ Examine for a purpuric rash (meningococcal meningitis). ❼ In those >55 years feel for palpable temporal arteries and scalp tenderness (temporal arteritis).

General Medicine 2 - Examine this patient with oedema

❶ Assess extent and degree of oedema present, e.g. facial, flanks, abdominal wall, sacral, genitalia, thighs, ankles and pedal. ❷ Look for nail changes associated with hypoproteinaemia (leukonychia, Muehrcke's nails, Beau's lines). ❸ Examine optic fundi for diabetic or hypertensive retinopathy. ❹ Measure blood pressure (may be elevated, but may be low with postural drop if plasma volume reduced due to severe hypoproteinaemia). ❺ Assess JVP (may be high-heart failure; but may be low if plasma volume reduced due to severe hypoproteinaemia). ❻ Auscultate for heart murmurs, especially a pansystolic mitral murmur to indicate dilatation of the left ventricle. ❼ Listen for extra heart sounds to indicate fluid overload and/or heart failure. ❽ In the lungs, look for signs of pleural effusions (nephrotic syndrome) or crackles indicating pulmonary oedema. ❾ Examine the abdomen for enlarged, smooth kidneys (renal amyloidosis causing nephrotic syndrome). ❿ Test for ascites (nephrotic syndrome). ⓫ Assess peripheral nerve function (peripheral neuropathy seen in diabetes mellitus and amyloidosis). ⓬ Examine urine for haematuria, proteinuria, casts and crystals.

General Medicine 2 - Examine this patient with acute breathlessness

❶ Assess severity of dyspnoea and record respiratory rate per minute. ❷ Look for evidence of weight loss or anaemia. ❸ Examine pulse for tachycardia, arrhythmias (cardiac disease) or bradycardia (cardiac disease or severe hypoxia). ❹ Measure blood pressure - hypotension found in septicaemia and tension pneumothorax. ❺ Examine tongue and lips for central cyanosis (severe hypoxia). ❻ Examine JVP - raised in cardiac failure, cor pulmonale, massive pulmonary thromboembolism and tension pneumothorax. ❼ Palpate trachea and apex beat for mediastinal shift - displaced away from the lesion in tension pneumothorax or massive pleural effusion; toward the lesion in lobar or lung collapse due to lung cancer. ❽ Palpate for chest expansion - unilateral reduction in pleural effusion, consolidation (pneumonia), lung or lobar collapse or pneumothorax. ❾ Percuss over the chest - hyper-resonant note may indicate pneumothorax, dull note consolidation (pneumonia), or lung or lobar collapse and a 'stony dull' note pleural effusion. ❿ Auscultate the chest - unilateral early inspiratory crackles (bronchial infection or pneumonia), bilateral basal medium crackles (pulmonary oedema), bilateral fine late inspiratory crackles (pulmonary fibrosis), bronchial breathing (pneumonia) and reduced breath sounds (lung or lobar collapse or pleural effusion). ⓫ Examine lower limb for pitting oedema (bilateral in cor pulmonale or heart failure; unilateral in deep vein thrombosis).

General Medicine 1 - Examine this patient with right leg pain paraesthesiae and numbness in the foot

❶ Assess the distribution of pain and sensory loss. Do they fit a dermatomal pattern consistent with compression of one nerve root? ❷ Examine the power of the lower limb muscles, particularly the flexors (L5 nerve root) and extensors (S1 nerve root) of the ankle and toes. ❸ Examine the reflexes of the leg. Is the ankle reflex present? Which nerve root is responsible for the ankle reflex? ❹ Assess nerve compression using the sciatic and femoral nerve stretch tests. ❺ Examine the circulation of the leg for arterial and venous insufficiency (peripheral vascular disease, deep venous thrombosis). ❻ Examine the hip joint for evidence of degenerative osteoarthritis.

Ophthalmology - Examine this patient with acute redness and pain in one eye

❶ Assess the distribution of the redness: Diffuse redness suggests conjunctivitis, episcleritis, scleritis Redness of the lower inner eyelid suggests conjunctivitis Circumciliary injection suggests keratitis, iritis or angle-closure glaucoma Redness resolving with phenylephrine drops suggests episcleritis. ❷ Look for evidence of ocular discharge (conjunctivitis). ❸ Examine the clarity of the iris - a hazy iris suggests corneal oedema (acute angle-closure glaucoma) or aqueous chamber inflammatory cells (acute iritis). ❹ Look for the small, irregularly shaped pupil of acute iritis, or an oval, mid-dilated, poorly reactive pupil (acute-angle closure glaucoma). ❺ Ask the patient to move the eye - pain indicates scleritis. ❻ Examine the red reflex with direct ophthalmoscopy: corneal ulceration appears black - confirm with fluorescein dye. ❼ Examine the urine for microscopic haematuria or proteinuria (systemic vasculitis). ❽ Assess visual acuity.

Oncology - Outline the Management of 1° Brain tumour (specifically glioma)

❶ Brain imaging: MRI + CT head ± nuclear ❷ Confirm histological diagnosis (biopsy/resection) ❸ Low grade may be treated with watchful waiting ± anticonvulsants or immediate surgery. De-bulking may help high grade but not curative ❹ Radical radiotherapy mainstay for high-grade, may also be used for residual low grade/palliation ❺ Temozolomide chemotherapy improves survival in high grade ❻ With full Rx glioblastoma has median survival of 14.6 months - supportive/end of life care

General Practice 2 - Examine this patient who complains of earache

❶ Carry out a general examination, looking for evidence of systemic upset (acute otitis media). ❷ Inspect and palpate the pinna and surrounding area (acute mastoiditis, acute otitis externa, trauma (haematoma), previous surgery). ❸ Inspect both external auditory meati for signs of discharge, crusting and excoriation (acute otitis media, acute otitis externa). ❹ Feel the tragus for tenderness (acute otitis externa). ❺ Using the otoscope, examine the external auditory meati (lift the pinna upwards and backwards to straighten the canal). ❻ Inspect the tympanic membrane for surface appearance and integrity. If the view is obscured with wax/debris, consider manual removal or microsuction. ❼ Ask the patient to open and close the jaw while you palpate the temporomandibular joints, listening for a click (temporomandibular joint dysfunction). ❽ Examine the throat (referred pain from the oropharynx). ❾ Consider other investigations such as audiology (hearing test).

General Medicine 1 - Examine this patient with slowly progressive weakness and numbness of the legs

❶ Carry out general examination to exclude heart failure, anaemia and hypothyroidism. ❷ Examine the lymph nodes, liver and spleen (metastatic tumours, lymphoma and other reticuloses, immunodeficiency). ❸ Examine the neurological system to determine the pattern and location of the lesion: Truncal sensory level with hyper-reflexia, weakness and spasticity below level - cord lesion. Loss of limb reflexes, wasting of limb muscles, and glove and stocking sensory deficit - peripheral neuropathy. ❹ Examine the spine for deformity and local tenderness.

General Practice 3 - Perform microscopy on this urine specimen

❶ Centrifuge 10 ml of fresh urine for 5 minutes at 3000 rpm. ❷ Remove the supernatant, leaving 0.5 ml of urine and the deposit. ❸ Mix the sediment gently and place one drop on a clean slide using a pipette. ❹ Overlay a cover slip and examine under the microscope with low illumination and low-power magnification. ❺ Examine under high-power magnification to clarify abnormalities seen. ❻ Dispose of sample, slide and pipette safely and wash hands. ❼ Document the findings in the patient's record.

General Practice 1 - Examine this patient with high blood pressure

❶ Check the pulse rate - irregularly irregular suggests atrial fibrillation. ❷ Measure the blood pressure in both arms. ❸ Check for radiofemoral delay (coarctation of the aorta). ❹ Examine the optic fundi for hypertensive retinopathy. ❺ Look for features of Cushing's syndrome or virilization. ❻ Palpate the abdomen for renal enlargement (adult polycystic kidney disease) and for the abnormal pulsation of an abdominal aortic aneurysm. ❼ Listen for bruits over the renal arteries (renal artery stenosis). ❽ Examine the heart for the heave of left ventricular hypertrophy and for a fourth heart sound. ❾ Look for evidence of heart failure (raised JVP, basal lung crackles, ankle oedema). ❿ Perform microscopic examination of the urine, looking for red cell casts.

Oncology - Outline the Management of Prostate cancer

❶ Digital rectal exam + PSA ❷ If high risk: Ultrasound and biopsy ❸ MRI for locoregional spread if T3/4/biochem/symptoms: isotope bone scan ❹ Radical prostatectomy definitive for localised, radical radiotherapy (beam/brachy) is another option, active surveillance possible if >70 ❺ For metastatic disease hormone treatment (↑ survival:, 3 options) Surgical castration, medical castration (LHRH antagonists [must give anti-androgen for 2 weeks due to flare]), androgen antagonist ❻ Palliative radiotherapy e.g. for bone mets

Oncology - Outline the Management of Multiple myeloma

❶ Dx: FBC, ESR, UEC, Ca❷⁺, albumin, electropheresis and immunofixation of urine + serum, β2-microglobulin, bone marrow aspirate + biopsy, skeletal survey ❷ Incurable, first line treatment is thalidomide + chemotherapy aiming for remission ❸ lenalidomide and bortezomib may be used 2nd/3rd line ❹ Bisphosphonates for life/1 year following remission ❺ Analgesia/palliative radiotherapy/EPO as needed

General Medicine 1 - Examine this patient with progressive clumsiness of the arm and leg, slurred speech and morning headache

❶ Examine optic fundi for papilloedema (raised intracranial pressure). ❷ Assess the motor and sensory systems of the limbs; exclude impaired joint position sense. ❸ Examine eye movement - nystagmus. ❹ Assess the lower cranial nerves - bulbar palsy. ❺ Test co-ordination of the limbs using finger-nose and heel-shin tests, and for rebound phenomenon. Assess ability to undertake rapid alternating movements - cerebellar ataxia. ❻ Check the stance and gait, and ability to sit without support (truncal ataxia).

Obstetrics and Gynaecology 2 - Examine this patient with urinary incontinence

❶ Examine the abdomen for any large mass arising from the pelvis. ❷ Examine the vulva. ❸ Inspect the urethral orifice. Ask the woman to cough and look for urine leaking. ❹ Look for prolapse of the vaginal walls or descent of the cervix outside the vaginal orifice. ❺ Carry out a bimanual examination to assess enlargement of the pelvic organs. ❻ Dipstick the urine for glucose, protein and blood to exclude diabetes and urinary tract infection. ❼ Arrange urodynamic assessment tests to differentiate between stress and urge incontinence.

General Medicine 3 - Examine this patient who may have chronic liver disease

❶ Examine the fingers for white nails (leukonychia) with loss of the lunula (half-moons at the nail base) and ankle oedema - all due to hypoalbuminaemia. ❷ Look for spider naevi (upper half of the body), palmar erythema, gynaecomastia and testicular atrophy - all due to oestrogen excess. ❸ Look for bruising due to: Thrombocytopenia (hypersplenism) Reduced hepatic synthesis of coagulation factors (II, VII, IX and X) Multiple falls due to alcohol intoxication. ❹ Look for muscle wasting from malnutrition and/or liver synthetic failure. ❺ Examine for signs of liver failure: Look for a coarse flap of the outstretched hands (asterixis) due to metabolic brainstem dysfunction (other causes include heart, renal and respiratory failure) Smell the patient's breath to detect fetor hepaticus (a sweetish, musty smell) due to accumulation of the volatile amine, methyl mercaptan Check for signs of hepatic encephalopathy (grade 1-4) due to the metabolic changes affecting cerebral function.

Oncology - Outline the Management of Chronic lymphocytic leukaemia

❶ FBC+differential+film, bone marrow aspirate+biopsy, immunophenotyping, FISH, abdominal ultrasound ❷ Chlorambucil/fludarabine ± cyclophosphamide ± steroids ❸ Alternate regimes if failure: Fludarabine/CVP/CHOP, anti-CD52 (alemtuzumab) if suitable ❹ Irradiation for symptomatic lymph nodes, splenectomy if massive/hypersplenic

Oncology - Outline the Management of Acute myeloid leukaemia

❶ FBC+film, bone marrow biopsy ❷ Assess age, performance status and if Acute Promyelocytic Leukaemia ❸ Chemotherapy is mainstay: Anthracycline + cytarabine ± etoposide/thioguanine OR all-trans retinoic acid if APL ❹ Consolidation therapy, anthracycline for APL ❺ Consider stem-cell Tx in young patients with suitable donor

Oncology - Outline the Management of Chronic myeloid leukaemia

❶ FBC+film, bone marrow biopsy, cytogenetics/molecular genetics ❷ Leukapharesis if symptomatic hyperviscosity ❸ Imatinib mainstay of treatment, 2nd generation (nilotinib/dasatinib) if this fails ❹ Allogenic stem cell Tx can be curative ❺ Combination chemotherapy may be required in blast crisis

Oncology - Outline the Management of Acute lymphoblastic leukaemia

❶ FBC+film, bone marrow biopsy, lumbar puncture ❷ Choose treatment based on risk group and subtype ❸ Induction of remission: Vincristine, steroids, L-asparaginase ± antracycline (adults, poor-risk children) ❹ Intensification/consolidation: new chemotherapy, clear CNS with MTX/irradiation ❺ Maintenance: 2 years of cyclic MTX, mercaptapurine, vincristine, steroids ± intrathecal prophylaxis ❻ Consider stem-cell Tx in high-risk/unresponsive

General Medicine 1 - Examine this patient with sudden loss of consciousness

❶ Feel the pulse for bradycardia (faint or syncope; heart block or Stokes-Adams attack), (supraventricular tachycardia or ventricular tachycardia; sinus tachycardia after seizure) and irregularity (sudden-onset atrial fibrillation). ❷ Measure the BP for hypotension (septicaemia, acute myocardial infarction). ❸ If possible, measure the BP erect and supine (postural fall of >20 mmHg systolic BP). ❹ Check temperature - raised in meningitis and subarachnoid haemorrhage. ❺ Listen for structural heart disease (valvular heart disease, ventricular septal defect post-myocardial infarction, cardiac tamponade). ❻ Listen to the carotids for bruits (embolic cerebrovascular accident). ❼ Examine for focal neurological signs (post-epileptic seizure, cerebral haemorrhage). ❽ Inspect the tongue for lacerations and check for urinary incontinence (post-epileptic seizure). ❾ Assess level of consciousness (epileptic seizure, subarachnoid haemorrhage, intracranial lesion).

General Medicine 1 - Examine this patient with sudden-onset central chest pain

❶ Feel the pulse for bradycardia (heart block), tachycardia (supraventricular tachycardia, ventricular tachycardia) and irregularity (atrial fibrillation, multiple ventricular extrasystoles). ❷ Palpate carotid and femoral pulses (may be weak or absent in aortic dissection). ❸ Measure the BP. ❹ Look for the JVP - raised in heart failure. ❺ Examine the trachea and cardiac apex beat for mediastinal shift (tension pneumothorax). ❻ Palpate the epigastrium for tenderness (gastro-oesophageal reflux, peptic ulcer, oesophagitis). ❼ Listen to the heart for extra heart sounds or gallop rhythm (heart failure), pansystolic murmur radiating to the left axilla (mitral regurgitation due to papillary muscle rupture post-myocardial infarction), pansystolic murmur at the left sternal edge (ventricular septal defect post-myocardial infarction) and pericardial friction rub (pericarditis).

General Medicine 1 - Examine this patient with acute pain and discoloration the right foot

❶ Feel the radial pulse for irregularity, e.g. atrial fibrillation. ❷ Look for evidence of tobacco staining on fingers. ❸ Measure the blood pressure in both arms - unequal in aortic dissection. ❹ Listen to the heart - early diastolic murmur may indicate aortic regurgitation due to aortic dissection. ❺ Feel the abdomen for pulsation of abdominal aortic aneurysm. ❻ Auscultate the abdomen for bruits of aorto-iliac disease. ❼ Look at the feet for discoloration: marbled in complete acute ischaemia due to embolus; fine reticular blanching/mottling in early stages; coarse, more fixed mottling in late stages. ❽ Palpate the femoral and popliteal pulses and feel for popliteal aneurysms. ❾ Squeeze the calves - tenderness suggests muscle infarction. ❿ Compare the warmth of both feet with the back of your hand. ⓫ Palpate for the dorsalis pedis and posterior tibial pulses in both feet. ⓬ Ask the patient to wiggle the toes, and test fine sensation - both motor and sensory function are absent in limb-threatening ischaemia. ⓭ If acute ischaemia is suspected, refer urgently to a vascular surgeon.

Oncology - Outline the Management of Colorectal cancer

❶ Full colonoscopy (mark site if for surgery) ❷ Staging US/CT of liver ± pelvis ❸ Excise bowel segment + arterial supply/lymphatics ❹ Need adjuvant chemotherapy if Duke C (50% relapse), consider if Duke B (30%): 5-FU ± oxaliplatin ❺ Consider partial liver resection if metastases ❻ Advanced: Chemotherapy + cetuximab (anti-EGFR), consider palliative stenting etc. ❼ Follow up surveillance

General Medicine 2 - Examine this patient with pleuritic chest pain

❶ General examination, assessing the degree and site of pain. ❷ Examine for fever, confusion and raised respiratory rate - all possible in pneumonia. ❸ Look for vasculitic skin spots (pulmonary vasculitis). ❹ Examine JVP - raised in massive pulmonary embolism. ❺ Palpate chest wall for tenderness - found in trauma, fractured rib and sometimes pulmonary embolism. ❻ Percuss the chest - dullness is consistent with consolidation (pneumonia) or pleural effusion (pulmonary embolism). ❼ Auscultate the chest - bronchial breathing is found over consolidation (pneumonia); reduced air entry over pleural effusion (pulmonary embolism); pleural friction rub heard over consolidation (pneumonia), pulmonary embolism with effusion and sometimes rib fracture. ❽ Examine lower legs for signs of deep vein thrombosis (pulmonary embolism).

General Medicine 2 - Examine this patient with haemoptysis

❶ General examination, looking for evidence of weight loss, anaemia, iron deficiency and bruising. ❷ Examine hands for finger clubbing (lung cancer or bronchiectasis). ❸ Feel neck for lymphadenopathy, especially scalene nodes (lung cancer or lymphoma). ❹ Inspect chest, e.g. for previous lung resection (lung cancer or bronchiectasis). ❺ Palpate trachea and apex beat for mediastinal shift (lung or lobar collapse due to lung cancer). ❻ Percuss for dullness - 'stony dullness' in pleural effusion (lung cancer, mesothelioma, tuberculous effusion). ❼ Auscultate for crackles persisting on inspiration and expiration (bronchiectasis) or reduced air entry (lung or lobar collapse due to lung cancer; or pleural effusion due to lung cancer or mesothelioma).

General Practice 3 - Obtain a midstream urine sample from this patient

❶ Give the patient a sterile urine container (a tray for a female patient). ❷ Ask the patient to start passing urine before collecting 10-20 ml in the container. ❸ Wash hands, put on gloves, and transfer urine from the tray to the laboratory container (female patient). ❹ Label the container with time, date and patient's name. ❺ Send to the laboratory without delay. If delay is unavoidable, place the sample in a fridge (not freezer) until transport is available. ❻ In children or non-compliant adults, consider suprapubic aspiration and samples obtained via a urethral catheter.

General Practice 2 - Examine this patient with a pigmented skin lesion on his back.

❶ Has the mole appeared recently or has it changed? (Malignant melanoma can occur in an existing melanocytic naevus or as a new lesion.) ❷ Has the mole has become bigger, has it been itchy or has it bled (malignant melanoma). ❸ Is there a family history of malignant melanoma or other skin cancer? (A family history is found in 10% of patients with malignant melanoma.) ❹ Does the patient have a fair skin type, i.e. does he burn easily and tan poorly or not at all? (Skin cancers are more common with a pale skin.) ❺ Look at the edge of the mole, the distribution of pigment within it and whether it is inflamed or ulcerated. (Malignant melanoma commonly shows variation in pigmentation or outline and has an irregular or diffuse edge.) Remember the mnemonic 'ABCD' - Asymmetry, Border irregular, Colour irregular and Diameter >6 mm. ❻ Examine the entire skin for other suspicious moles or skin cancers. (Abnormal moles are more common in patients who have a malignant melanoma.) ❼ Request urgent surgical excision biopsy for histological examination.

Oncology - Outline the Management of Oesophageal cancer

❶ History and examination - symptoms, comorbidity and evidence of metastases: e.g. cervical LN, abdomen. ❷ Ix: FBC, UEC, LFTs, Coags, CRP ❸ Upper GI endoscopy + biopsy. Ba swallow if failed intubation or ? post-cricoid Ca. ❹ Staging: Endo-oesophageal US, CT: chest/abdomen, ± laparoscopy ± PET ± bronchoscopy. ❺ Potentially curative: • Squamous cell: Radical XBRT followed by radical surgical resection. • Adenocarcinoma: neoadjuvant chemotherapy (if large) + radical surgical resection. ❻ Palliative: Endoluminal metal stenting, XBRT, laser therapy ❼ Metastatic: Systemic chemotherapy if symptomatic: Cisplatin + 5-FU ± Epirubicin

Oncology - Outline the Management of Gastric cancer

❶ History and examination ❷ Ix: FBC, UEC, LFTs, Coags, CRP ❸ Upper GI endoscopy + biopsy. ❹ Staging: Endoscopic US, CT: chest/abdomen, + laparoscopy ❺ Surgical resection best hope for long term survival, ideally T1N0/N1, consider LN dissection, post operative radiotherapy + 5-FU based chemo ❻ Advanced: combination chemotherapy e.g. Epirubicin + Oxaliplatin + Capecitabine [etoposide, irinotecan, doxetacel also have efficacy...] ❼ Palliative: Endoluminal metal stenting, surgical bypass.

Oncology - Outline the Management of Pancreatic cancer

❶ History and examination ❷ Ix: FBC, UEC, LFTs, Coags, CRP, ERCP, endoscopic US, CT, laparoscopy ❸ FNA for pathological Dx ❹ If suitable for surgical excision: Pancreaticoduodenectomy (Whipple's procedure) ± pylorus preservation ± vascular/nodal dissection. ❺ Locally unresectable: 5-FU based chemotherapy combined with radiotherapy. ❻ Advanced: Gemcitabine standard of care for palliative chemotherapy (if PST adequate) ± Erlotinib. Other palliative issues: Nutrition, pain, stenting for biliary obstruction.

Oncology - Outline the Management of Testicular cancer

❶ History and examination: Abdominal, testicular, supraclavicular LN palpation, respiratory and neurological examination. ❷ Ix: USS both testes, CXR, Tumour markers: αFP, β hCG, LDH. CT head/chest/abdomen ❸ Radical orchidectomy via inguinal incision as soon as possible. ❹ Carcinoma in situ: Low dose XRT (20 Gy in 10 fractions) ❺ Stage I seminoma: Either: • Adjuvant radiotherapy to para-aortic LN (T11-L5) • Annual surveillance CT abdomen/pelvis OR • One cycle adjuvant carboplatin chemotherapy ❻ Stage IIA/IIB seminoma: Orchidectomy + 30 Gy/15 vs 36 Gy/18 para-aortic + ipsilateral iliac ❼ Stage I NSGCT: Orchidectomy, then • Surveillance (CT, CXR + tumour markers) • 2 cycles of adjuvant chemotherapy: Bleomycin, Etoposide, Cisplatin • Nerve sparing retroperitoneal LN dissection ± chemotherapy ❽ Stage IIc-IV seminoma, IM-IV NSGCT: • Good prognosis: Chemotherapy: 3xBEP or 4xEP (respiratory fibrosis limiting factor for Bleomycin) ± surgical resection. • Intermediate/poor prognosis: 4xBEP, consider clinical trials. ❾ Regular follow up.

Oncology - Outline the Management of Osteosarcoma

❶ Hx and Physical Exam: Evidence of possible 1° with bone mets. ❷ FBC, UEC, CRP, LDH, LFTs with αFP, Plain X-rays of site, MRI bone + soft tissue, CT-chest, ☢ bone scan. Angiography may assist in surgery planning. ❸ ☎ Need assessment by surgeon specialising in bone cancer who should take bone biopsy (trephine/core) for pathology. Biopsy tract will later need to be removed. ❹ Surgery: en-block resection with clear margins may be effective, otherwise amputation. Metasectomy may be effective in more advanced disease. ❺ Chemotherapy: may be used neo/adjuvantly to surgery. Most active agents are: doxorubicin, cisplatin, ↑dose MTX, ifosfamide

Oncology - Outline the Management of Renal cell carcinoma

❶ Hx and Physical Exam: Family Hx. ❷ Ix: FBC, LFTs, UEC, LDH, Ca❷⁺, Contrast-enhanced CT chest + abdomen ± head. ❸ Surgical resection (radical nephrectomy) only curative treatment. No evidence that adjuvant therapy is beneficial. ❹ RCCa relatively chemo/radioresistant. 5-FU most consistently active ± gemcitabine. ❺ Biologicals: IL-2, induces response in 10-25%. IFNα is also frequently used. ❻ Targeted: Bevacizumab (anti-VEGF) + IFNα. Small-molecule inhibitors: Sunitinib (1st line good-risk in UK) and Sorafenib (oral TK-inhibitors), Temsirolimus (mTOR inhibitor, used in poor-risk).

Oncology - Outline the Management of Transitional cell carcinoma

❶ Hx and physical exam ❷ FBC, LFTs, UEC ❸ Cystoscopy and Transurethral Resection of Bladder Tumour TURBT: Pathological Dx and staging. ❹ If ↓grade, superficial, no further Ix needed. ↑grade has risk of synchronous disease, IVU needed. ❺ If G3 pT1 and muscle invasive disease staging with CXR and CT/MRI abdomen+pelvis. ❻ Superficial: Single infusion of intravesical mitomicin ↓risk of recurrence. ↻ disease can be treated with intravesical mitomycin/epirubicin or bacillus Calmette-Guérin (BCG) ❼ Muscle invasive: Radical cystectomy OR radiotherapy with bladder conversion ± neo/adjuvant chemotherapy. ❽ Radical surgery: Cystoprostatectomy/anterior bladder exenteration with LN dissection. Radical radiotherapy may be given for elderly patients with comorbidities.

Ophthalmology - Outline the Management of Retinoblastoma

❶ Hx and physical exam ❷ Imaging: Ocular USS. CT best avoided. MRI may be used if indicated. Mutation testing can be done on peripheral blood. ❸ Specialist team required for treatment. ❹ Laser: Green (532 nm, photocoagulation) or Infrared (810 nm, thermotherapy) suitable for small tumours. ❺ Chemotherapy (e.g. Carboplatin, Etoposide and Vincristine) may be used to shrink tumour to size that can be treated with a laser. ❻ Cryotherapy may be used for larger peripheral tumours. ❼ XBRT is dangerous with germline mutation due to 2° cancer risk. Plaque brachytherapy may be useful for some tumours. ❽ Ennucleation is the oldest and most effective treatment. Eye and long section of optic nerve are removed and replaced with implant for volume and later a prosthetic shell is added. ❾ Need long term psychological support and counselling about eye care, genetics and cancer risk factors.

Oncology - Outline the Management of Gallbladder/bile duct cancer

❶ Hx and physical examination ❷ FBC, LFTs, UEC, Abdominal USS, CT scan, ERCP ❸ Radical surgical resection required. Adjuvant therapy not of any proven benefit. ❹ Gemcitabine can be used as monotherapy for palliative chemotherapy.

Oncology - Outline the Management of Malignant mesothelioma

❶ Hx and physical examination, specifically occupational/exposure Hx, thorax, ascites, mediastinal obstruction. ❷ Ix: FBC, Coags, LFTs, UEC, CXR, CT ± MRI ❸ Try for pathological specimen: aspiration cytology, US/CT guided biopsy, thoracoscopic biopsy. ❹ Consider mediastinoscopy + laparoscopy for LN staging and peritoneal/diaphragmatic involvement. ❺ Extrapleural pneumonectomy may offer long-term survival in a minority of patients (<5%). Pleurectomy + decortication/pleurodesis may help control effusions. ❻ Chemotherapy agents may be effective: Cis/carboplatin ifosfamide, mitomicin, pemetrexed. ❼ Specialist palliative care is needed and compensation may be an issue.

Oncology - Outline the Management of Thyroid cancer

❶ Hx and physical examination. ❷ FBC, LFTs, UEC, TFTs, Thyroid autoantibodies, Calcitonin, High resolution thyroid USS + FNA. Consider staging imaging: CT/MRI head + neck, CT chest + liver ± bone scan for lytic lesions. ❸ Screen for phaeochromocytoma (MEN2) and ↑Ca❷⁺ (MEN2a) pre-operatively if medullary subtype. 50% of anaplastic is metastatic at presentation. ❹ Aim for definitive surgery, at least total lobectomy (if ↓risk) typically total thyroidectomy (must be total for medullary). ❺ More extensive (pT4): Either • Extensive resection of all involved structures: thyroid, larynx, oesophagus etc. OR • Conservative surgery followed by XBRT ❻ Medullary: Central and ipsilateral LN dissection. Radical neck dissection or excision of ↻ disease may be needed. ❼ 131Iodine ablation ± XBRT may be required for metastatic disease. ❽ Check Ca❷⁺ and nerve function for surgical complications. Follow up with serum thyroglobulin monitoring. Replace thyroid hormones at slightly ↑thyroxine dose (100-200 μg od) to ↓TSH

Obstetrics and Gynaecology 2 - Outline the Management of Uterine cancer

❶ Hx: PMB most common presentation (⚠ 10% will have endometrial Ca/atypical hyperplasia), menstrual/obstetric history, PM/FHx of cancers (e.g. HNPCC: 40-60% lifetime risk of Endometrial Ca). Drug: HRT/OCP/antihypertensives/tamoxifen/OHGAs. Comorbidities. ❷ Exam: vulva/vagina/spec, rule out other causes of bleeding. Bimanual assess uterine size/mobility, adenexal masses. ❸ Ix: FBC, UEC, LFTs. Transvaginal USS (endometrial thickness < 4 mm = very low risk in postmenopausal women, don't need sampling), endometrial biopsy ± hysteroscopy. Staging: CXR, Pelvic MRI. ❹ ☎ Refer to specialist gynaecological oncology unit. ❺ 46% of women with atypical hyperplasia have a cancer. The others are at very high risk. For cancer/AH recommend Total Abdominal Hysterectomy + Bilateral Salpingo-Oophrectomy if cancer/patient >45 + Pelvic Washings if Cancer] ❻ Consider pelvic ± para-aortic lymph node clearance ± infracolic omentectomy if high risk subtype (e.g. clear cell, serous, carcinosarcoma) or evidence of spread. ❼ Adjuvent radiotherapy recommended if stage Ib (½> of myometrium) AND grade 3/3, or if stage Ic (½<) and any grade. Vault radiation and/or pelvic XRT may be used if not suitable for surgery. ❽ For conservative treatment of atypia give ↑dose progestogens e.g. MedroxyProgesterone Acetate 100 mg daily, biopsy 2-3 monthly and follow closely. Can also be used for palliation (200 mg bd) in advanced disease. ❾ Endometrial is the most survivable gynaecological cancer, follow up with regularly Hx e.g. residual symptoms and full physical/spec/PV/PR exams. ❿ Sarcomas are rare, making up only 3-5% of uterine cancers but 26% of the mortality. Post operative chemotherapy does not help stage I/II disease.

Obstetrics and Gynaecology 2 - Outline the Management of Ovarian cancer

❶ Hx: Symptoms, Risk factors, FHx (consider genetic screen, lifetime risk with two 1° relatives ≈ 60%), comorbidities ❷ Physical exam: Systemic signs, chest, abdominal & pelvic. ❸ Bloods: FBC, UEC, LFTs (especially albumin). Tumour markers: CA125 (most important but non specific, use CEA to exclude CRC CA19.9 for pancreatic), rarer ovarian tumour markers: AFP, hCG, LDH, inhibin, oestradiol. ❹ Imaging: USS abdomen/pelvis, CXR, CT abdomen/pelvis. ❺ Calculate Risk of Malignancy Index, if high risk ☎ refer to specialist gynaecological oncology unit ❻ Surgical staging laparotomy (debulk as much tumour as possible): hysterectomy, bilateral salpingo-oophrectomy, omentectomy, pelvic & para-aortic LN sampling, peritoneal biopsies + pelvic/ascitic sampling, remove any abdominal mets. ❼ Fertility sparing surgery may be an option in some low-risk cases (using intraoperative pathology). In these cases (uterus + contralateral ovary preserved) patient should have 3-monthly CA125 measurement and 6 monthly transvaginal USS. ❽ Unless low-risk (stage Ia-Ib and low grade) adjuvant chemo recommended: IV Carboplatin (check renal function) ± paclitaxel. Carboplatin monotherapy is very well tolerated. ❾ XRT may be of use in stage III with surgical remission (whole abdominal), or for palliation (e.g. vaginal bleeding, brain mets). ❿ Non-epithelial ovarian cancers are rare (10% of 1° cancers), many are more aggressive. Germ cell tumours have an excellent prognosis, for Ic and above use fertility sparing surgery and Bleomycin Etoposide cisPlatin chemotherapy. ⓫ Follow up with clinical examination, USS and tumour markers.

Oncology - Outline the Management of Head and neck cancer

❶ Hx: Symptoms, risk factors (tobacco, alcohol), diet, infections (HPV, HSV1/2, EBV, T.p.p), FHx, Chemical/wood dust/radiation exposure ❷ Physical exam: Inspection/flexible fibreoptic endoscopy, bimanual examination of oral cavity, cervical LN examination, systemic examination for mets ❸ Ix: FBC, UEC, LFTs, coags. Dental assessment. Local CT/MRI head and neck for extent/spread. Skeletal scintigraphy if ?bone mets ± PET scan. ❹ Consider EUA and biopsy. FNA for salivary gland to ↓ seeding risk. Consider US guided FNA of LN. ❺ T1-2, N0, M0 disease treated with single modality radiotherapy/surgery. ❻ More advanced may require combined modality. Cisplatin can be used as sensitising agent. Cetuximab may be useful. ❼ Metastatic disease: Combination chemotherapy if good PST: Cisplatin plus 5-FU ± Cetuximab or a taxane. ❽ Follow up surveillance is important. ❾ Rehabilitation: Speech/airway, dental, nutrition, psychological/body image, address risk factors.

Oncology - Outline the Management of Hepatocellular carcinoma

❶ Hx: symptoms of any cancer, (2°?) FHx, risk factors, Hx/evidence of liver disease/cirrhosis ❷ FBC, UEC, LFTs, Coags, CRP, LDH, HBV, HCV, αFP, CXR, Liver USS ❸ Dx: • Non-invasive [if cirrhotic, HBV] based on size, αFP levels and hallmark features on 4-phase CT/dynamic contrast enhanced MRI • Pathologic: US/CT guided fine needle biopsy ❹ Staging: CT thorax, abdomen. Bone scan. Assess performance status (PST) and Child-Pugh Classification. ❺ Very early stage ("0", PST=0, Child-Pugh A "in situ" single lesion < 2 cm): If normal portal pressure, bilirubin: Surgical resection with 1 cm margin. Otherwise see "Early stage". ❻ Stages A-C (Early/intermediate/advanced, Child-Pugh=A-B): • Early ("A", 1-3 lesions ≤ 3 cm, PST=0): Cadaveric or living donor liver Tx. If comorbidities: Radiofrequency or percutaneous ethanol injection. • Intermediate ("B", multinodular, PST = 0): Transcatheter arterial chemoembolisation (TACE). • Advanced ("C", Portal invasion, N1, M1 or PST = 1-2): Sorafenib (multitargeted TK inhibitor) ❼ Terminal Stage (D, PST > 2, Child-Pugh = C): Best supportive care.

Oncology - Outline the Management of Lung cancer

❶ Initial Ix: CXR [obviously], FBC [anaemia], Ca❷⁺ [paraneoplastic/bone], UEC [contrast], LFTs, lung function tests, coags [e.g. biopsy/scope] ❷ Contrast CT chest and abdomen ± head ❸ Try to get a tissue Dx least invasive: sputum cytology (~useless), FNA accessible nodes, bronch, pleural, FNA/core lung/liver, VATS... open lung ❹ NSCLC: If thought to be treatable FDG-PET for staging. SCLC never localised: limited or extensive ❺ Surgery only chance to cure early NSCLC, if small inoperable, radiotherapy offers some hope. Chemotherapy primary treatment for SCLC, it will come back. ❻ Chemotherapy ± TK inhibitors for advanced NSCLC. ❼ Palliative radiotherapy may help for both, e.g. brain mets, thorax

General Practice 2 - Examine this patient who complains of a blocked nose

❶ Inspect the external nasal pyramid for asymmetry or evidence of trauma. ❷ Using your thumb, lift the nasal tip to inspect the nostrils, e.g. septal haematoma following a blow to the nose. ❸ Ask the patient to sniff while gently occluding each nostril in turn. In children hold a mirror or metal spatula under their nostrils, looking for two patches of condensation from expired air - the 'mirror test'. ❹ Examine the anterior nasal cavity, especially the position of the septum and size of the inferior turbinates, using a large speculum on an otoscope. Look for nasal polyps, foreign bodies (especially in children) or other soft tissue swellings. ❺ Refer to visualize the postnasal space (nasopharynx): with endoscopy in adults and older children, or lateral soft tissue X-ray in young children/infants (adenoidal hypertrophy). ❻ Consider further investigations, such as allergy testing (allergic rhinitis) and a CT scan of the paranasal sinuses (chronic rhinosinusitis).

General Practice 2 - Examine this patient who complains of being dizzy and lightheaded

❶ Is the patient pale (anaemia) or sweating (cardiac disease, anxiety, labyrinthitis)? ❷ Check pulse, blood pressure lying and standing, and temperature (dysrhythmias, hypotension or viral infection). ❸ Look for a facial palsy (cholesteatoma). ❹ Examine the ears (cholesteatoma). ❺ Examine for nystagmus. ❻ Carry out the Dix-Hallpike test. ❼ Perform Romberg's test (sensory ataxia). ❽ Carry out Unterberger's test (labyrinthitis, vestibular disease). ❾ Watch the patient walk: slow, fast and turn, heel to toe (cerebellar disease).

Ophthalmology - Examine this patient with sudden loss of vision in one eye

❶ Look at the hands for signs of smoking. ❷ Examine the pulse for arrhythmia (atrial fibrillation). ❸ Measure the blood pressure (hypertension). ❹ Listen for cardiac murmurs and carotid bruits (valvular heart disease, carotid artery stenosis). ❺ Measure the visual acuity in both eyes. ❻ Check colour vision (optic nerve disease). ❼ Assess both peripheral visual fields for homonymous hemianopia (cerebrovascular accident) and the affected eye's visual field for an altitudinal or arcuate field defect (optic nerve disease). ❽ Examine for a relative afferent pupillary defect (optic nerve disease). ❾ Assess pain on eye movement (optic neuritis). ❿ Perform ophthalmoscopy. The optic nerve is: White and swollen in arteritic anterior ischaemic optic neuropathy (giant cell arteritis) Pink and swollen in non-arteritic anterior ischaemic optic neuropathy. ⓫ Look at the optic fundus: Numerous retinal haemorrhages indicate venous occlusion Pallor indicates arterial occlusion Retinal embolism is seen at arterial bifurcations (thromboembolic disease).

Obstetrics and Gynaecology 2 - Examine this patient with very heavy menstrual bleeding

❶ Look for evidence of iron deficiency (smooth tongue, angular stomatitis, koilonychia) and anaemia. ❷ Examine the patient's abdomen for any mass arising from the pelvis. ❸ Perform a speculum examination to check the cervix for any polyps or prolapsing fibroid. ❹ Carry out a bimanual pelvic examination to assess the size, position and surface characteristics of the uterus, looking for fibroids. ❺ Check haemoglobin concentration and serum ferritin. ❻ Arrange a pelvic ultrasound scan. ❼ If the patient is >45 years, arrange an endometrial biopsy. Consider a diagnostic hysteroscopy if the ultrasound suggests the presence of an endometrial abnormality.

Obstetrics and Gynaecology 2 - Examine this patient with postmenopausal bleeding

❶ Look for evidence of weight loss or anaemia. ❷ Feel the abdomen to detect any mass arising from the pelvis. ❸ Inspect the vulva for possible malignancy. ❹ Pass a vaginal speculum and inspect: the vaginal walls for atrophic change or, rarely, vaginal cancer the cervix for cancer or a cervical polyp. ❺ Take a cervical smear if this is due or the cervix looks abnormal. ❻ Perform bimanual examination and assess uterine size and any adnexal mass. ❼ Consider further investigations, e.g. endometrial biopsy, transvaginal ultrasound scan, examination under anaesthetic, diagnostic curettage, colposcopy and cervical biopsy.

General Medicine 2 - Examine this patient with newly diagnosed renal failure

❶ Look for pallor, uraemic complexion, excoriation, bruising, uraemic fetor, asterixis (features of chronic renal failure). Look for hyperventilation (Kussmaul respiration), suggesting metabolic acidosis. ❷ Examine skin for signs of cutaneous vasculitis, or purpura (Henoch-Schönlein disease). Look for the nail-bed pigmentation of chronic renal failure. ❸ Assess skin turgor and eyeball tone as possible indicators of severe salt and water depletion causing renal failure. ❹ Look for limbic calcification (chronic renal failure). ❺ Look at optic fundi for hypertensive or diabetic retinopathy, or retinal infarcts in systemic vasculitis or systemic lupus erythematosus. ❻ Measure blood pressure (elevated in chronic renal failure; may be low with postural drop if the patient is fluid-deplete). ❼ Listen for heart murmurs, especially an early diastolic murmur or other manifestations to suggest subacute bacterial endocarditis. ❽ Examine abdomen for enlarged, smooth kidneys or bladder (obstructive uropathy). ❾ Palpate for large cystic kidneys in adult polycystic kidney disease. ❿ Auscultate for epigastric and femoral bruits, and assess peripheral pulses (may suggest generalized vasculopathy with renal artery stenosis). ⓫ Look for peripheral oedema. ⓬ Assess peripheral nerve function (peripheral neuropathy of chronic renal failure or diabetic neuropathy). ⓭ Examine urine for haematuria, proteinuria, casts, crystals, white cells and bacteria.

General Medicine 3 - Examine this patient with tiredness and pallor

❶ Look for signs of chronic blood loss or iron deficiency - pallor, angular stomatitis, koilonychia. ❷ Look at face for puffiness about the eyes (hypothyroidism) and, in men, poor beard growth (hypopituitarism). ❸ Listen to voice - slow, deliberate, croaky (hypothyroidism). ❹ Measure BP - hypotension in hypovolaemia, Addison's disease and hypopituitarism. ❺ Feel the neck for goitre (hypothyroidism). ❻ Palpate abdomen for tenderness, masses and hepatosplenomegaly. ❼ Examine for absence of axillary and pubic hair (hypopituitarism). ❽ Examine external genitalia - small testes in men (hypopituitarism). ❾ Examine visual fields - bitemporal hemianopia (pituitary tumour). ❿ Percuss tendon reflexes - delayed relaxation in hypothyroidism.

General Medicine 3 - Examine this patient with jaundice

❶ Look for signs of chronic liver disease: Finger clubbing, palmar erythema, leukonychia Spider naevi: upper trunk, head, neck and arms Gynaecomastia in males. ❷ Look for signs of liver failure: Flapping tremor of outstretched hands Sweet smell on breath of fetor hepaticus Confusion, diminished mental state. ❸ Look for pallor - haemolytic anaemia. ❹ Look for signs of weight loss - malignancy with liver metastases. ❺ Smell the breath for alcohol. ❻ Look for hepatomegaly, splenomegaly, ascites and caput medusae. ❼ Look for: Needle track marks - evidence of drug misuse (hepatitis B and C) Tattoos and body piercing - hepatitis B and C ❽ Look for scratch marks (obstructive jaundice). ❾ Examine ankles and sacrum for oedema (hypoalbuminaemia). ❿ Look at stools - pale in post-hepatic jaundice. ⓫ Urinalysis - dark urine in hepatic or post-hepatic jaundice.

Obstetrics and Gynaecology 1 - Examine this patient with acute pelvic pain

❶ Look for signs of shock: pallor, low blood pressure and weak rapid pulse (acute blood loss or sepsis). ❷ Record the temperature. ❸ Feel the lower abdomen for tenderness, guarding, rebound and any masses arising from the pelvis. ❹ Inspect the vulva for signs of bleeding. ❺ Pass a vaginal speculum and look for bleeding or vaginal discharge. ❻ Take endocervical swabs for chlamydia or gonorrhoea and a high vaginal swab. ❼ Perform bimanual examination to assess the size of the uterus and any adnexal masses, and to elicit any tenderness of the pelvic organs. ❽ Perform rectal examination if you suspect appendicitis. ❾ Perform a pregnancy test in any woman of reproductive age. Measure serum beta human chorionic gonadotrophin if the pregnancy test is negative and you suspect ectopic pregnancy. ❿ Consider further investigations, including pelvic ultrasound scan and diagnostic laparoscopy.

Oncology - Examine this patient with a breast lump

❶ Look for signs of swelling, skin changes (dimpling, erythema, peau d'orange) and nipple change (asymmetric pull, Paget's disease). ❷ Examine the breast for a mass and record site, size, consistency, position, tethering and fixation to chest wall or skin. ❸ Check for bloodstained nipple discharge. ❹ Examine regional nodes in axilla and supraclavicular fossa. Comment on size, consistency and fixation.

Obstetrics and Gynaecology 3 - Examine this patient with a nipple discharge

❶ Look for visible abnormalities with the patient sitting, first with her hands on her thighs and then above her head. ❷ Palpate for any masses. ❸ Gently massage the breast and identify any nipple discharge, noting colour, position, laterality and number of ducts affected. ❹ Examine the axilla if you detect any lumps. ❺ Test the discharge for blood with Microstix.

General Medicine 2 - Examine this patient with loin pain

❶ Measure temperature - raised in urinary infection. ❷ Measure blood pressure - elevated in renovascular disease and chronic kidney disease. ❸ Look at optic fundi for hypertensive retinopathy. ❹ Examine abdomen for tenderness (renal or ureteric stones or other causes of obstruction, renal infection or inflammation). ❺ Examine abdomen for enlarged, palpable kidneys: Unilateral in renal cancer, urinary tract obstruction Bilateral in adult polycystic kidney disease. ❻ Look for evidence of non-renal causes of loin pain, e.g. spinal or other locomotor disease. ❼ Examine urine for haematuria and proteinuria - haematuria alone is non-specific; presence of both suggests renal inflammation.

General Medicine 2 - Examine this patient with frank haematuria

❶ Measure temperature - raised in urinary tract infection, connective tissue disease, endocarditis with renal involvement. ❷ Look for bruising or purpura (coagulation disorder, Henoch-Schönlein purpura, vasculitis). ❸ Examine nails for splinter haemorrhages (bacterial endocarditis with renal involvement). ❹ Measure blood pressure - elevated in renovascular and chronic kidney disease. ❺ Look at optic fundi for hypertensive retinopathy. ❻ Examine abdomen for renal tenderness (renal or ureteric stones or other causes of obstruction, renal infection or inflammation). ❼ Examine abdomen for enlarged, palpable kidneys - unilateral in renal cancer, urinary tract obstruction; bilateral in polycystic kidney disease. ❽ Palpate suprapubically for palpable bladder (benign prostatic enlargement, prostate or bladder cancer). ❾ Assess peripheral pulses and listen for renal artery or other bruits (renovascular disease). ❿ In men, perform rectal examination to assess enlargement of prostate - irregular in prostate cancer. In women, consider a pelvic examination to assess reproductive organs.

Obstetrics and Gynaecology 2 - Examine this patient with secondary amenorrhoea

❶ Measure the BMI. ❷ Consider hyperthyroidism - palpate the thyroid gland, look for exophthalmos, check the pulse and examine for tremor. ❸ Look for acne or excess body hair affecting the patient's face (upper lip and chin) or the nipple area, or extending up to the umbilicus from the pubic hair, suggesting possible polycystic ovary syndrome (PCOS). ❹ Examine the nipples and check for galactorrhoea (hyperprolactinaemia). ❺ Examine the abdomen for any masses arising from the pelvis. Consider pregnancy. ❻ Use a speculum to examine the cervix then carry out a bimanual pelvic examination to assess uterine size. ❼ Perform a pregnancy test. ❽ Check the FBC and thyroid function. Check hormone profile: FSH and LH (↑ if the patient is menopausal; ↓ if she has anorexia/weight loss, is stressed or exercises excessively), prolactin and testosterone (↑ in the presence of PCOS). ❾ Further investigations as appropriate, e.g. pelvic ultrasound scan of the ovaries if PCOS is suspected.

Paediatrics 0 - Examine this patient with inability to walk at 18 months

❶ Movement and posture: Perform the 180° test: pull the supine baby by the hands to a sitting position and look for head lag. Does the child sit unsupported? Put your hands under the baby's axillae and raise him up, noting tone. Bring the baby's feet down to touch the table and see if the child supports his weight. Turn the baby quickly face down and note whether the child supports his head. Watch for the 'parachute reflex', whereby the baby suddenly extends the arms as if for protection as you lie him down (usually only 6-9 months). Does the baby push his chest up off the floor by extending the arms? Does he roll from front to back and from back to front? Note any asymmetry of movement. ❷ Vision and manipulation: Can the baby build a tower of two blocks? ❸ Hearing and speech: Does the baby say three words, other than mama and dada? Does he understand simple commands? ❹ Social behaviour: Does the baby drink from a cup? Can the child feed himself with a spoon?

General Medicine 3 - Examine this patient with weight loss and a good appetite

❶ Observe patient's demeanour - hyperactive, 'staring eyes' (hyperthyroidism). ❷ Examine eyes for lid lag and proptosis (thyroid eye disease). ❸ Look for signs of dehydration - dry tongue, lack of skin turgor, e.g. diabetes mellitus, coeliac disease. ❹ Examine hands for finger clubbing (coeliac disease), warmth, sweating, fine tremor (hyperthyroidism). ❺ Examine nails for any onycholysis or thyroid acropachy (hyperthyroidism). ❻ Feel pulse - tachycardia in hyperthyroidism, bradycardia in hypothyroidism. ❼ Palpate neck for goitre (diffuse or nodular). ❽ Look at legs for pretibial myxoedema. ❾ Look at optic fundi for diabetic retinopathy. ❿ Perform urinalysis for glycosuria (diabetes mellitus).

General Medicine 3 - Examine this patient with excessive sweating and snoring

❶ On greeting the patient, note the large, moist and fleshy hands. ❷ Lift a pinch of skin from the dorsum of the hand and note its increased thickness. ❸ Look at the face for signs of acromegaly: thick, greasy skin, especially over the forehead, large nose and tongue, prognathism and separation of lower teeth. ❹ Inspect feet for increased soft tissues. ❺ Look for signs of carpal tunnel syndrome (wasted thenar eminence(s), sensory loss). ❻ Check visual fields (possible bitemporal hemianopia). ❼ Measure BP (one-third of acromegalics have hypertension). ❽ Test the urine for glycosuria (one-third of acromegalics have diabetes mellitus).

General Practice 2 - Examine this patient who complains of a sore throat

❶ Perform a general examination, looking for evidence of systemic upset (pyrexia, flushing, tachycardia, drooling). ❷ Palpate the neck for lymphadenopathy, especially tender jugulodigastric nodes (viral pharyngitis, bacterial tonsillitis or glandular fever). ❸ Inspect the oral cavity and oropharynx with the patient's dentures removed, using a bright light source and a wooden tongue depressor to look at the surface of and any asymmetry in the tonsils. ❹ Ask the patient to open the mouth; difficulty (trismus) indicates inflammation affecting muscles of mastication. ❺ Feel the oral cavity, especially the tongue, using your gloved finger. ❻ Look into the nostrils, using the broad auriscope attachment, for evidence of inflammation (coryza). ❼ Consider further examination of the larynx and hypopharynx (specialist examination). ❽ Consider further investigations (blood film and monospot for glandular fever), allergy testing if seasonal symptoms (allergic rhinitis), throat swab (for bacterial tonsillitis).

General Medicine 3 - Examine this patient with an abdominal swelling

❶ Position the patient supine with one pillow beneath the head, not the shoulders. ❷ Expose the area to be examined while maintaining the patient's dignity. ❸ Look at the shape of the abdomen to clarify the nature of any distension. ❹ Note the position of scars and check the nature of any surgery with the patient. ❺ Ask the patient about areas of tenderness and ask him to tell you if discomfort arises. ❻ Palpate superficially, then more deeply in the abdomen. ❼ Palpate and percuss for liver enlargement. ❽ Palpate and percuss for splenic enlargement. ❾ Percuss the flanks and hypogastric areas for evidence of dullness. If this is present, look for shifting dullness by percussing in the supine, then the left lateral position. If ascites is suspected, look for a fluid thrill. If this is present, ask the patient to place a hand along the midline to exclude cutaneous transmission of a thrill.

Obstetrics and Gynaecology 3 - Outline the Management of Cervical cancer (Squamous, adeno is similar)

❶ Prevention: Vaccinate against type 16 and 18 (also low risk 6/11) in year 7 (age 12) girls. ❷ Screening: Pap smears. 2 yearly from age 18 or 1st sexual intercourse + 2 (whichever is later?). ↻ in 6-12/52 if unsatisfactory. ❸ If possible/definite Low Grade Squamous Intraepithelial Lesion (LGSIL/"CIN 1") ↻ in 12/12 [unless 30+ years and history of recent + smear:immediate colposcopy/↻ pap smear in 6/12] ❹ If ↻ smear shows LGSIL/HGSIL or index shows possible/HGSIL → ☎ Refer to gynaecologist for colposcopy to assess lesion ± targeted biopsy for histopathology. ❺ Histologically confirmed HGSIL (CIN 2/3) needs treatment to prevent malignancy. Options: ... ... ... followed by screening... ... ... colposcopy, cytology, HPV typing after 4-6/12, and more extensive follow up. ❻ For cancer surgery minimally invasive... cone biopsy to preserve fertility or simple total hysterectomy... up to 3 mm if lymphovascular space... consider pelvic lymphadenectomy. 3+ mm (1A2) modified radical hysterectomy + pelvic lymphadenectomy OR intracavitary RT ± External beam XRT pelvis (needed for 1B, 2A). ... complicated basically. ❼ More advanced e.g. with pelvic/para-aortic lymphadenopathy... External beam XRT pelvis + concurrent IV Cisplatin if ECOG < 4 and adequate bone marrow/liver/renal function. ❽ Metastatic/recurrent: Chemo/radio ± debulking LN, palliative care, varies based on treated or not.

Paediatrics 0 - Examine this patient with enuresis at 10 years of age

❶ Record height and weight on a growth chart. ❷ Measure blood pressure (hypertension from renal scarring associated with chronic reflux). ❸ Examine the abdomen - an indentable mass (faeces) in the left iliac fossa suggests constipation exacerbating enuresis. ❹ Ballot the loins to detect enlarged kidneys. ❺ Palpate the bladder and percuss to delineate a full bladder. ❻ Examine the lower spine for a dimple or tuft of hair (spina bifida occulta). ❼ Examine the lower limbs for muscle tone and tendon reflexes. ❽ Test the urine for haematuria and proteinuria, and for diabetes. ❾ Obtain a mid-stream specimen of urine (MSSU) to exclude infection.

General Medicine 2 - Examine this patient with fever and breathlessness

❶ Record respiratory rate - >30/min is a sign of severe pneumonia. ❷ Assess severity of breathlessness (use MRC scale). ❸ Assess mental state and look for confusion - a sign of severe pneumonia. ❹ Measure blood pressure - hypotension found in septicaemia; diastolic BP <60 mmHg is a sign of severe pneumonia. ❺ Examine tongue and lips for central cyanosis (severe hypoxia). ❻ Palpate for chest expansion - unilateral reduction in pleural effusion, empyema or consolidation (pneumonia). ❼ Percuss the chest - dull note (consolidation/pneumonia), 'stony dull' note (pleural effusion/empyema). ❽ Auscultate the chest - reduced breath sounds (pleural effusion), unilateral inspiratory crackles (bronchial infection or pneumonia), bronchial breathing and whispering pectoriloquy (consolidation/pneumonia).

General Medicine 3 - Examine this patient with weight loss and heat intolerance

❶ Shake hands and note excessive warmth and sweating. ❷ Look for fine tremor of outstretched hands and acropachy of the fingers (hyperthyroidism). ❸ Examine the radial pulse (usually rapid and high-volume unless on a β-blocker). An irregularly irregular pulse is likely to indicate atrial fibrillation. ❹ Examine the thyroid and describe its size, surface and consistency. Listen for a thyroid bruit. ❺ Look for evidence of thyroid eye disease. ❻ Inspect the shins for pretibial myxoedema.

General Practice 3 - Perform urinalysis

❶ The urine specimen should be <4 hours old. ❷ Wear a disposable apron and gloves. ❸ Examine the specimen for smell, colour, clarity and other abnormalities. ❹ Use a watch that indicates seconds. ❺ Dip the reagent strip into the urine specimen, making sure all the test areas are covered. Remove the strip after 2 seconds. ❻ Tap the edge of the strip against the rim of the container to remove excess urine. ❼ Hold the strip horizontally with test areas upward. Hold as close to the colour chart as possible and compare the test areas, reading each reagent at the time specified. ❽ Dispose of test strip, apron and gloves into a clinical waste bag; wash hands. ❾ Document the results in the patient's clinical record.

Oncology - Outline the Management of Cancer of unknown primary

❶ Thorough Hx including FHx. Thorough examination including rectal, breast, pelvic/testicular ❷ Ix: FBC, UEC, TFTs, LFTs, CXR, USS/CT abdomen ± Tumour markers: CA125, CA 15-3, CEA, CA 19-9, hCG, AFP ❸ Consider sampling of any available LN/mets/effusion, with pathology/immunohistochemistry to identify tissue type. ❹ Exclude potentially curable: • If LN: Exclude lymphoma. • Cervical LN: Exclude head and neck. • Axillary LN: Exclude breast. • Bone: Exclude breast, thyroid. • Abdominal/pelvic: Exclude ovarian/prostate. • Thorax/CNS/Liver: low yield of treatable ❺ If single metastatic lesion consider surgical excision. ❻ Consider local radiotherapy for symptomatic lesions, e.g. bone, CNS. ❼ Consider systemic chemotherapy based on organ function, performance status, response to treatment and QOL issues. ❽ Palliative care

Oncology - Outline the Management of Hodgkin's lymphoma

❶ Whole lymph node biopsy for Dx ❷ Staging: CT neck, chest, abdomen, pelvis ± FDG-PET. Bone marrow trephine if advanced ❸ Assess prognostic factors, Radiotherapy standard for early disease ❹ Adriamycin, Bleomycin, Vinblastine, Dacarbazine (ABVD) usually added or used alone in advanced disease ❺ Salvage therapy may include stem cell Tx and/or alternate chemotherapy regimes

Oncology - Outline the Management of Non-Hodgkin's lymphoma

❶ Whole lymph node or extranodal site biopsy for Dx ❷ Staging: CXR, CT chest, abdomen, pelvis, FBC+film, bone marrow aspirate+biopsy, UEC, LFTs, Ca❷⁺, uric acid, LDH, β2-microglobulin ± Neuro/bone imaging/LP ❸[True] stage I follicular lymphoma curable with involved field radiotherapy, marginal-zone MALT lymphoma curable with H. pylori erradication ❹ More advanced/symtomatic FL: Rituximab (anti-CD20) based chemotherapy e.g. R-CHOP, R-CVP. ❺ Allogenic stem cell Tx second line ❻ Higher grade NHLs may involve CNS, higher risk of tumour lysis syndrome

General Practice 2 - Examine this patient with acute lumbar back pain

❶ With the patient standing, look for spinal deformity, e.g. decreased lordosis, scoliosis, scars, and for skin abnormality, e.g. hairy patch found in spina bifida. ❷ Feel the spinous processes and paraspinal tissues for any focal tenderness. ❸ Gently percuss the vertebral column with your closed fist, noting any tenderness. ❹ Ask the patient to try to touch his or her toes with the legs kept straight and record how far down the patient can reach. ❺ Ask the patient to straighten up and lean back as far as possible and record the amount of movement. ❻ Ask the patient to reach down to each side, touching the outside of the leg as far as possible, and record the amount of flexion. ❼ Perform the straight leg-raising test. ❽ Perform the femoral stretch test. ❾ Check lower limb reflexes, power and sensation (including the perineum), and anal tone.

General Medicine 3 - Examine this patient with an inguinal hernia

❶ With the patient supine, expose the groins and scrotum. ❷ Inspect both groins for swelling and scars indicating previous surgery. ❸ Ask the patient to point to where the swelling usually appears. ❹ Ask the patient to cough and look for the appearance of a swelling. ❺ Ask the patient to reduce the lump. If the patient cannot, try to reduce it yourself. ❻ Palpate the swelling to check its position relative to the pubic tubercle. ❼ If the hernia is reducible, apply pressure over the deep inguinal ring and ask the patient to cough. Does this control the hernia? ❽ Examine the other groin to exclude bilateral hernias. ❾ In a male patient, examine the scrotum and testes with the patient standing. Does the hernia pass into the scrotum? If it does and it is irreducible, does it transilluminate and are there bowel sounds audible in the hernia?

General Medicine 3 - A 48-year-old man is admitted to hospital with newly diagnosed diabetes. He is discharged on twice-daily insulin and after 6 weeks has established reasonable diabetic control with no hypoglycaemia. He has previously driven heavy goods vehicles and is keen to return to work. He asks your advice about driving.

❶: Please counsel the patient.

General Medicine 3 - A 53-year-old man is diagnosed with type 2 diabetes mellitus. He is treated with diet and subsequently oral agents. A couple of years after diagnosis, he begins to take his diet more seriously and takes up regular exercise. He loses weight and his glucose levels fall to such an extent that he is experiencing frequent hypoglycaemia. He is keen to reduce his medication and wants to know if the diabetes is disappearing.

❶: Please counsel the patient.

General Medicine 2 - A 65-year-old man, recently diagnosed as having mycobacterium TB by sputum examination, is brought to the chest clinic by his daughter. She tells you that her father is an alcoholic and returns a large number of anti-TB tablets that have not been taken.

❶: What are the risks of poor compliance with treatment in 'open TB'? ❷: What options are available to improve compliance?

General Medicine 3 - A 72-year-old man has recently been diagnosed as having type 2 diabetes. He is mildly symptomatic with lethargy, thirst and polyuria. A random blood glucose level is 13 mmol/L and dipstick urine testing revealed +++ glucose but no ketones. He is a known hypertensive, taking bendrofluazide 5 mg once daily and his weight is stable at 94 kg (BMI 32 kg/m2). On examination, there is no evidence of large or small vessel complications.

❶: What are the treatment options? ❷: What factors influence the choice of therapy? ❸: What would be the preferred choice of treatment? ❹: How would the impact of treatment be assessed?

General Medicine 3 - A 44-year-old woman presents with excessive sweating and an increase in hat size over a 3-year period. She comments that her face and hands just seem 'bigger' than before.

❶: What do you think the likely diagnosis is? ❷: What tests could you do to confirm this? ❸: What tests would help establish the cause? ❹: What is the initial management? ❺: What are the long-term complications/sequelae?

General Medicine 3 - A 56-year-old woman presents with depression, hirsutism, weight gain and muscle weakness. On examination her GP found her to be hypertensive and to have wide, purple striae on her abdomen and thighs. Her visual fields were full to confrontation and there was no bruising.

❶: What do you think the likely diagnosis is? ❷: What tests could you do to confirm this? ❸: What tests would help establish the cause? ❹: What is the initial management? ❺: What are the long-term complications/sequelae? ❻: Will the tumour spread to other parts of my body? ❼: Can I have tablets instead of an operation?

Ophthalmology - A 60-year-old patient had successful cataract surgery 18 months ago. The patient complains of gradual reduction in vision in the pseudophakic eye. There is no history of diabetes or hypertension. Ocular examination is normal apart from the presence of posterior capsule opacification.

❶: What is posterior capsule opacification? Explain this condition to the patient, including its treatment.

General Medicine 1 - A 78-year-old man who lives alone is brought to A&E after being found on the floor at home by his neighbour. He is confused and disorientated. His head is bruised and he appears unkempt. He is immobile and there is cog-wheel rigidity in all his limbs. There are no focal neurological signs. Reflexes are hard to elicit as a result of the muscle stiffness. Plantar responses are down-going. There are signs of a lower respiratory tract infection. Temperature is 38.2°C, BP 102/60 mmHg, HR 94/min.

❶: What is the differential diagnosis? ❷: What features in the history support the diagnosis? ❸: What additional features in the history would you seek to support the potential diagnoses? ❹: What other features would you look for on clinical examination? ❺: What investigations would you perform? ❻: What treatment options are available?

General Medicine 2 - A 22-year-old white woman presented to her GP with progressive swelling of her legs of 2 weeks' duration. She was otherwise asymptomatic. On examination, she had oedema up to both knees and her blood pressure (BP) was 120/70 mmHg. Urinalysis revealed +++ proteins but no blood; serum creatinine was 92 μmol/L and serum albumin 1.8 g/dL.

❶: What is the likely diagnosis? ❷: How would you investigate this case? ❸: How would you manage the case? ❹: What is the prognosis?

General Medicine 2 - A 58-year-old South Asian man has recently been found to have a raised serum creatinine of 156 μmol/L (reference range: 60-120 μmol/L). He has been on antihypertensive treatment for 10 years and a smoker for 40 years. He is on nifedipine, atenolol and simvastatin. His serum creatinine was 121 μmol/L a year ago.

❶: What is the likely diagnosis? ❷: How would you investigate this case? ❸: How would you manage the case? ❹: What is the prognosis?

General Medicine 2 - A 34-year-old man of African-Caribbean origin has been referred by his GP for severe hypertension. On examination, his BP is 240/140 mmHg, he is slightly confused, there is mild pedal oedema, there are a few basal crackles and neurological examination is normal except for bilateral papilloedema. Urine analysis shows blood ++, protein ++. His Hb is 10.2 g/dL and U&Es show Na+ 136 mmol/L, K+ 3.0 mmol/L, urea 14 mmol/L and creatinine 206 μmol/L (GFR 54 mL/min).

❶: What is the likely diagnosis? ❷: How would you investigate this case? ❸: What would be the initial management? ❹: What would be the long-term management?

General Medicine 2 - A 60-year-old man, previously fit and well, presented to the accident and emergency department (A&E) with a 1-week history of diarrhoea and vomiting. He was unwell and clammy, his skin turgor was poor, and his pulse and BP were 120/min and 80/60 mmHg, respectively. Initial investigations showed serum Na+ 126 mmol/L, K+ 3.0 mmol/L, urea 28 mmol/L and creatinine 306 μmol/L (calculated GFR 22.5 mL/min).

❶: What is the likely diagnosis? ❷: How would you investigate this case? ❸: What would be the initial management? ❹: What would be the long-term management?

General Medicine 3 - A previously well 32-year-old woman presents to the GP surgery with a 24-hour history of abdominal pain and vomiting. On direct questioning, she admits to weight loss over the previous 2 weeks, associated with thirst, polydipsia and polyuria. A fingerstick glucose is 22 mmol/L and urine dipstick testing shows +++ ketones.

❶: What is the likely diagnosis? ❷: What investigations would be performed? ❸: How would the diagnosis be confirmed? ❹: What would be the initial management? ❺: What are the potential complications? ❻: What issues need to be addressed when the patient has fully recovered?

General Medicine 3 - An 18-year-old man who is known to have type 1 diabetes is found unconscious in his front garden. He had been playing football earlier in the day but had not complained of symptoms at that time. The diabetes was treated with four injections of insulin a day and his control was regarded as excellent.

❶: What is the likely diagnosis? ❷: What investigations would be performed? ❸: How would the diagnosis be confirmed? ❹: What would be the initial management? ❺: What are the potential complications? ❻: What issues need to be addressed when the patient has fully recovered?

General Medicine 2 - A 73-year-old woman describes a 3-week history of stiffness in her proximal arms and thighs, which have become increasingly painful. The pain is especially bad at night and first thing in the morning. In the last 2 days she has noticed tenderness in the scalp when combing her hair and headache over the same site. Today she awoke to realize that she had no sight in her right eye.

❶: What is the likely differential diagnosis? ❷: How would you investigate this case? ❸: How would you confirm the likely diagnosis? ❹: How would you initially manage the case? ❺: What are the principles of long-term management? ❻: What is the prognosis?

General Medicine 3 - A 72-year-old woman describes an acute onset of mid-back pain. She has not had anything like this before and presents to the accident and emergency department (A&E). There was no obvious precipitant for her back problem. A radiograph reveals a wedge fracture of T8.

❶: What is the likely differential diagnosis? ❷: How would you investigate this case? ❸: How would you confirm the likely diagnosis? ❹: How would you initially manage the case? ❺: What are the principles of long-term management? ❻: What is the prognosis? ❼: How can I prevent fractures in the future?

General Medicine 1 - A 26-year-old man with fever, headache and confusion. A 26-year-old executive presents with a 1-week history of headache and 2 days of confusion. He has a temperature of 39°C. He returned 3 weeks previously from a 2-week safari in Kenya. Full blood count (FBC) and biochemical profile are normal; computed tomography (CT) shows no abnormality.

❶: What is the likely differential diagnosis? ❷: What in the given history supports the diagnosis? ❸: What additional features in the history would you seek to support a particular diagnosis? ❹: What clinical examination would you perform and why? ❺: What investigations would be most helpful and why? ❻: What treatment options are appropriate?

General Medicine 1 - A 36-year-old woman with fever, headache and a rash. A 36-year-old female university lecturer presents with 3 days of fever, headache and a rash that she describes as 'a return of her suntan'. Four weeks earlier she returned from a 2-week holiday in Kenya. She is on no medication. She has several 1-cm diameter glands in her neck, pharyngitis and a maculopapular rash on her trunk. The temperature is 38°C.

❶: What is the likely differential diagnosis? ❷: What in the given history supports the diagnosis? ❸: What additional features in the history would you seek to support a particular diagnosis? ❹: What clinical examination would you perform and why? ❺: What investigations would be most helpful and why? ❻: What treatment options are appropriate?

General Medicine 1 - A 50-year-old man presents with a 12-month history of gradualonset tremor in his right hand. It is present at rest and he feels that it is aggravated by stress. His handwriting is deteriorating and getting smaller and he feels that his movements are generally slower. Otherwise his health is good.

❶: What is the likely differential diagnosis? ❷: What in the given history supports the diagnosis? ❸: What additional features in the history would you seek to support a particular diagnosis? ❹: What clinical examination would you perform and why? ❺: What investigations would be most helpful and why? ❻: What treatment options are appropriate?

General Medicine 1 - A 50-year-old, previously well man presents with a 5-month history of progressive weakness in his upper and lower limbs. The symptoms initially affected the hands, the right more than the left. He comments that the muscles in his hands have got 'thinner' and he has noticed some 'flickering' of the muscles in his arms. There are no sensory symptoms.

❶: What is the likely differential diagnosis? ❷: What in the given history supports the diagnosis? ❸: What additional features in the history would you seek to support a particular diagnosis? ❹: What clinical examination would you perform and why? ❺: What investigations would be most helpful and why? ❻: What treatment options are appropriate?

General Medicine 3 - A 60-year-old man is brought to see you accompanied by his wife. Together they describe that he has had progressively worsening vomiting for some months. He has lost a stone in weight. He finds that the vomiting is worse after a full meal, but liquids trouble him less than solid food. In the past he has suffered from indigestion and has taken over-the-counter remedies for this.

❶: What is the likely differential diagnosis? ❷: What in the given history supports the diagnosis? ❸: What additional features in the history would you seek to support a particular diagnosis? ❹: What clinical examination would you perform and why? ❺: What investigations would be most helpful and why? ❻: What treatment options are appropriate? ❼: The tablets for my ulcer were wonderful. Should I continue to take them?

General Medicine 2 - A 35-year-old mother attends the medical outpatient clinic complaining of breathlessness with wheeze. This can occur at rest and has been getting worse over the last 2 years. Between episodes of breathlessness she feels better but does not feel that she ever returns to 'normal'. Occasionally she awakens with shortness of breath and coughing in the early hours of the morning.

❶: What is the likely differential diagnosis? ❷: What issues in the given history support the diagnosis? ❸: What additional features of the history would you seek to support a particular diagnosis? ❹: What clinical examination would you perform and why? ❺: What investigations would be most helpful and why? ❻: What treatment options are appropriate?

General Medicine 2 - A 35-year-old woman is brought to the accident and emergency department (A&E) complaining of acute onset of chest pain the preceding evening and collapsing that afternoon. She is shocked and dyspnoeic. This pain is described as 'sharp', located posteriorly on the left hemithorax, and worse on inspiration and coughing. On the morning of admission there was some haemoptysis. She has recently been in hospital for treatment and investigation of recurrent miscarriage.

❶: What is the likely differential diagnosis? ❷: What issues in the given history support the diagnosis? ❸: What additional features of the history would you seek to support a particular diagnosis? ❹: What clinical examination would you perform and why? ❺: What investigations would be most helpful and why? ❻: What treatment options are appropriate?

General Medicine 2 - A 73-year-old man attends his GP complaining of left-sided, 'sharp' chest pain. The pain is worse on inspiration, does not radiate and has appeared in the last 2 days. He has not felt well for 1 week with uncontrollable shivering bouts and sweats. He has had a cough with red-tinged sputum for 3 days. He smokes 20 cigarettes a day, and has angina complicated by a myocardial infarction (MI) 2 years previously. He has recently felt increasingly short of breath with effort.

❶: What is the likely differential diagnosis? ❷: What issues in the given history support the diagnosis? ❸: What additional features of the history would you seek to support a particular diagnosis? ❹: What clinical examination would you perform and why? ❺: What investigations would be most helpful and why? ❻: What treatment options are appropriate?

General Medicine 3 - A 31-year-old man with type 1 diabetes has joined your practice. He was diagnosed aged 12 years and, having attended the clinic regularly as a child, defaulted from the adult clinic, attending only when he needed replacement equipment (injection devices, etc.). He takes no other medication and has had no treatment for complications. BP: 160/80 mmHg; dipstick urine testing: +++ glucose, ++ albumin, + blood, but no ketones; visual acuity (VA): 6/6 in both eyes.

❶: What questions would you ask the patient and why? ❷: What investigations would you request? ❸: What examination would you perform? ❹: What are the differential diagnoses related to diabetes?

General Medicine 3 - A 56-year-old man is diagnosed as having type 2 diabetes by his local optometrist. He attends your practice and during diabetic review complains of 'pins and needles' affecting both his feet and his right hand. The symptoms are worse at night when he is in bed. Discomfort in his hand often disturbs his sleep. There is no discomfort in the feet during the day and exercise tolerance has been unaffected.

❶: What questions would you ask the patient and why? ❷: What investigations would you request? ❸: What examination would you perform? ❹: What are the differential diagnoses related to diabetes?

General Medicine 3 - A 26-year-old woman presents with upper abdominal pain and vomiting. Previously she had experienced episodes of right upper quadrant colicky pain that had lasted several hours at a time. On one of these occasions she had experienced a brief episode of jaundice accompanied by dark urine. However, the present episode is different. The pain has lasted for 2 days and appears to be worsening and, this time, it is non-colicky.

❶: What specific questions would you ask the patient? ❷: What is the most likely diagnosis? ❸: What examination would you perform? ❹: What would be the initial management? ❺: What investigations would you request to confirm a diagnosis? ❻: What other issues should be addressed? ❼: Why do I have this?


संबंधित स्टडी सेट्स

Macroeconomics: Chapter 6 Launchpad

View Set

Microbiology - Ch 13 (Microbe-Human interaction Infection and Disease)

View Set

SBOK Guide Certified Scrum Master

View Set

Rosh Review Pulmonology Fall 2017

View Set

Frontend Technical Interview Questions

View Set